Vous êtes sur la page 1sur 240

Concepts in Calculus II

UNIVERSITY PRESS OF FLORIDA


Florida A&M University, Tallahassee
Florida Atlantic University, Boca Raton
Florida Gulf Coast University, Ft. Myers
Florida International University, Miami
Florida State University, Tallahassee
New College of Florida, Sarasota
University of Central Florida, Orlando
University of Florida, Gainesville
University of North Florida, Jacksonville
University of South Florida, Tampa
University of West Florida, Pensacola

Orange Grove Texts Plus

Concepts in Calculus II
Bona

Miklos
and Sergei Shabanov
University of Florida Department of
Mathematics

University Press of Florida


Gainesville Tallahassee Tampa Boca Raton
Pensacola Orlando Miami Jacksonville Ft. Myers Sarasota

Copyright 2012 by the University of Florida Board of Trustees on behalf of the University of
Florida Department of Mathematics
This work is licensed under a modified Creative Commons Attribution-Noncommercial-No
Derivative Works 3.0 Unported License. To view a copy of this license, visit http://
creativecommons.org/licenses/by-nc-nd/3.0/. You are free to electronically copy, distribute, and
transmit this work if you attribute authorship. However, all printing rights are reserved by the
University Press of Florida (http://www.upf.com). Please contact UPF for information about
how to obtain copies of the work for print distribution. You must attribute the work in the
manner specified by the author or licensor (but not in any way that suggests that they endorse
you or your use of the work). For any reuse or distribution, you must make clear to others the
license terms of this work. Any of the above conditions can be waived if you get permission from
the University Press of Florida. Nothing in this license impairs or restricts the authors moral
rights.
ISBN 978-1-61610-161-9
Orange Grove Texts Plus is an imprint of the University Press of Florida, which is the scholarly
publishing agency for the State University System of Florida, comprising Florida A&M
University, Florida Atlantic University, Florida Gulf Coast University, Florida International
University, Florida State University, New College of Florida, University of Central Florida,
University of Florida, University of North Florida, University of South Florida, and University of
West Florida.
University Press of Florida
15 Northwest 15th Street
Gainesville, FL 32611-2079
http://www.upf.com

Contents
Chapter 6. Applications of Integration
36. The Area Between Curves
37. Volumes
38. Cylindrical Shells
39. Work and Hydrostatic Force
40. Average Value of a Function

1
1
8
17
22
28

Chapter 7. Methods of Integration


41. Integration by Parts
42. Trigonometric Integrals
43. Trigonometric Substitution
44. Integrating Rational Functions
45. Strategy of Integration
46. Integration Using Tables and Software Packages
47. Approximate Integration
48. Improper Integrals

33
33
36
41
45
51
54
59
67

Chapter 8. Sequences and Series


49. Innite Sequences
50. Special Sequences
51. Series
52. Series of Nonnegative Terms
53. Comparison Tests
54. Alternating Series
55. Ratio and Root Tests
56. Rearrangements
57. Power Series
58. Representation of Functions as Power Series
59. Taylor Series

77
77
85
92
99
106
111
117
126
133
139
147

Chapter 9. Further Applications of Integration


60. Arc Length

157
157

Chapter and section numbering continues from the previous volume in the series,
Concepts in Calculus I.

vi

61.
62.
63.
64.

CONTENTS

Surface Area
Applications to Physics and Engineering
Applications to Economics and the Life Sciences
Probability

162
168
176
182

Chapter 10. Planar Curves


65. Parametric Curves
66. Calculus with Parametric Curves
67. Polar Coordinates
68. Parametric Curves: The Arc Length and Surface Area
69. Areas and Arc Lengths in Polar Coordinates
70. Conic Sections

193
193
201
207
214
221
225

CHAPTER 6

Applications of Integration

36. The Area Between Curves


36.1. The Basic Problem. In the previous chapter, we learned that if

the function f satises f (x) 0 for all real numbers x in the interval
[a, b], then the area of the domain whose borders are the graph of f ,
the
 b horizontal axis, and the vertical lines x = a and x = b is equal to
f (x) dx. If there is no danger of confusion as to what a and b are,
a
then this fact is sometimes informally expressed by the sentence the
integral of f is equal to the area of the domain that is under the graph
of f .
What can we say about the area of the domain between two curves?
There are several ways to ask this question. The easiest version, discussed by the following theorem, diers from the previous situation
only in that the horizontal line is replaced by another function g.
Theorem 6.1. Let f and g be two functions such that, for all real
numbers x [a, b], the inequality f (x) g(x) holds. Then the domain
whose borders are the graph of f , the graph of g, and the vertical lines
x = a and x = b has area
 b
(f (x) g(x)) dx.
A=
a

See Figure 6.1 for an illustration of the content of Theorem 6.1.


The reader is invited to explain why this theorem is a direct consequence of the fact that we recalled in the rst paragraph of this section.
The reader is also invited to explain why the theorem holds even if f
and g take negative values.
Example 6.1. Compute the area A(D) of the domain D whose
borders are the graph of the function f (x) = x3 +1, the function g(x) =
x2 + 2, and the vertical lines x = 2 and x = 3. See Figure 6.2 for an
illustration of this specic example.
Solution: In order to see that Theorem 6.1 is applicable, we must
rst show that, for all x [2, 3], the inequality f (x) g(x) holds.
1

6. APPLICATIONS OF INTEGRATION

Figure 6.1. Area enclosed by f (x) and g(x) between


x = a and x = b.

Figure 6.2. Area enclosed by the graphs of f (x) = x3 +


1 and g(x) = x2 + 2 between x = 2 and x = 3.

This is not dicult, since we only need to show that if x [2, 3], then
f (x) g(x), that is,
x3 + 1 x2 + 2,
x3 x2 1,
x2 (x 1) 1,
and this is clearly true since x 2, so x2 4, and x 1 1, forcing
x2 (x 1) 4.

36. THE AREA BETWEEN CURVES

Therefore, Theorem 6.1 applies, and we have




(x3 x2 1) dx =

(x3 +1)(x2 +2) dx =

A(D) =
2

=8

 3
x
x


x 

4
3
4

11
.
12

36.2. Intersecting Curves. Sometimes, the points a and b are deter-

mined by the curves themselves, and not given in advance. In that


case, we have to compute them before we can apply Theorem 6.1.
Example 6.2. Find the area A(D) of the domain D whose borders
are the graphs of the functions f (x) = x2 + 3x + 5 and g(x) = 2x2 +
7x + 8. See Figure 6.3 for an illustration.
Solution: Let us nd the points in which the graphs of f and g intersect. In these points, we have
x2 + 3x + 5 = 2x2 + 7x + 8,
0 = x2 + 4x + 3,
0 = (x + 3)(x + 1).
That is, the two curves intersect in two points, and these points have
horizontal coordinates a = 3 and b = 1. Furthermore, if x

Figure 6.3. Area enclosed by the graphs of f (x) and


g(x) between x = 3 and x = 1.

6. APPLICATIONS OF INTEGRATION

[3, 1], that is, if x is between those two intersection points, then
f (x) g(x) = (x2 + 3x + 5) (2x2 + 7x + 8)
= (x2 + 4x + 3)
= (x + 3)(x + 1)
0,
since x + 3 0 and x + 1 0. Therefore, if x [3, 1], then
f (x) g(x), and Theorem 6.1 applies. So we have

A(D) =

 3
 1

x

(f (x)g(x)) dx = (x2 +4x+3) dx = 2x2 3x 

3
3

1
=1 .
3

2
The situation becomes slightly more complicated if f g does not
hold throughout the entire interval [a, b]. For instance, it could happen
that f (x) g(x) at the beginning of the interval [a, b], and then, from a
given point on, g(x) f (x). In that case, we split [a, b] up into smaller
intervals so that on each of these smaller intervals, either f (x) g(x)
or g(x) f (x) holds. Then we can apply Theorem 6.1 to each of
these intervals. As on some of these intervals f (x) g(x) holds, while
on some others g(x) f (x) holds, theapplication of Theorem 6.1 will
sometimes
involve the computation of (f (x)g(x)) dx and sometimes

(g(x) f (x)) dx. The following theorem formalizes this idea.
Theorem 6.2. Let f and g be two functions. Then the area of the
domain whose borders are the graph of f , the graph of g, the vertical
line x = a and the vertical line x = b is equal to
 b
|f (x) g(x)| dx.
a

Note that Theorem 6.1 is a special case of Theorem 6.2, namely,


the special case when f (x) g(x) = |f (x) g(x)| for all x [a, b].
Example 6.3. Let f (x) = x3 + 3x2 + 2x and let g(x) = x3 + x2 .
Compute the area A(D) of the domain whose borders are the graphs of
f and g and the vertical lines x = 2 and x = 1. See Figure 6.4 for
an illustration.

36. THE AREA BETWEEN CURVES

Figure 6.4. Graphs of f (x) and g(x) on [2, 1].


Solution: In order to use Theorem 6.2, we rst need to compute
|f (x) g(x)|. We have
f (x) g(x) = (x3 + 3x2 + 2x) (x3 + x2 ),
2x2 + 2x = 0,
2x(x + 1) = 0.
That is, there are only two points where these two curves intersect,
namely, at x = 1 and x = 0. If x 1 or if x 0, then f (x)g(x) =
2x(x+1) > 0, so |f (x)g(x)| = f (x)g(x) = 2x2 +2x. If 1 < x < 0,
then f (x) g(x) < 0, so |f (x) g(x)| = g(x) f (x) = 2x2 2x.
Figure 6.5 shows the behavior of the function |f (x) g(x)|.
We can now directly apply Theorem 6.2. We get
 1
|f (x) g(x)| dx
A(D) =
2
1


=

+


(g(x) f (x)) dx

0
1

(f (x) g(x)) dx +

(f (x) g(x)) dx
 0
2
(2x + 2x) dx +
(2x2 2x) dx
1

(2x2 + 2x) dx

+
0


=

 1 
 0
 1
 3
3
2x
2x
2x



+ x2  +
x2  +
+ x2  .



3
3
3
3

6. APPLICATIONS OF INTEGRATION

Figure 6.5. Graph of |f (x) g(x)| on [2, 1].

36.3. Curves Failing the Vertical Line Test. Sometimes we want to com-

pute the area between two curves that do not pass the vertical line
test; that is, they contain two or more points on the same vertical line.
Such curves are not graphs of functions of the variable x. If they pass
the horizontal line test, that is, if they do not contain two points on
the same horizontal line, then they can be viewed as functions of y.
We can then change the roles of x and y in Theorems 6.1 and 6.2 and
proceed as in the earlier examples of this section.
Example 6.4. Compute the area A(D) of the domain between the
vertical line x = 4 and the curve given by the equation y 2 = x. See
Figure 6.6 for an illustration.
Solution: Neither curve satises the vertical line test, but both satisfy
the horizontal line test. Therefore, we set f (y) = 4 and g(y) = y 2 . It is

Figure 6.6. Graph of y 2 = x and x = 4.

36. THE AREA BETWEEN CURVES

clear that the two curves intersect at the points given by y = 2 and
y = 2. Between these two curves, the value of f (y) is larger. Therefore,
Theorem 6.1 applies (with the roles of x and y reversed). So we have
 2
A(D) =
(f (x) g(x)) dx
2
2


=


=

4 y 2 dy

y3
4y
3

16
= 16
3
2
= 10 .
3

 2




Note that the geometric meaning of reversing the roles of x and y


is simply reecting all curves through the x = y line. That reection
does not change the area of any domain, so one can expect analogous
methods of computing areas before and after that reection.
36.4. Exercises.

(1) Find the area of the domain whose borders are the vertical line
x = 0, the vertical line x = 2, and the graphs of the functions
f (x) = x2 + 3 and g(x) = sin x.
(2) Find the area of the domain whose borders are the vertical line
x = 2, the vertical line x = 4, and the graphs of the functions
f (x) = x2 + 3 and g(x) = x.
(3) Find the area of the domain between the horizontal axis and
the graph of the function f (x) = x2 + 6x.
(4a) Find the area of the domain between the horizontal axis and
the graph of the function f (x) = 18 2x2 .
(4b) Find the area of the domain between the horizontal axis and
the graph of the function f (x) = 16 x4 .
(5) Find the area of the domain whose borders are the vertical line
x = 1, the vertical line x = 3, and the graphs of the functions
f (x) = x3 and g(x) = ex .
(6) Find the area of the domain whose borders are the vertical
line x = 1, the vertical line x = 1, and the graphs of the
functions f (x) = x3 and g(x) = 2.

6. APPLICATIONS OF INTEGRATION

(7) Find the area of the domain between the graphs of the functions f (x) = ex and g(x) = x + 1 and the vertical lines x = 0
and x = 1. How do we know that between those two vertical
lines, the graph of f is above the graph of g?
(8) Find the area of the domain between the graphs of the functions f (x) = x2 + 2 and g(x) = 4x 1.
(9) Find the area of the domain between the graphs of the functions f (x) = x3 3x2 and g(x) = x2 .
(10) Find the area of the domain between the graphs of the functions f (x) = x2 and g(x) = x1/5 .
(11) Find the area of the domain between the graphs of the functions f (x) = x3 and g(x) = x4 .
(12) Find the area of the domain
between the graphs of the functions f (x) = x and g(x) = x.
(13) Compute the area between the curves given by the equations
x = 5y and x = y 2 + 6.
(14) Compute the area between the curves y = x 2 and y =
(x 2)3 .
(15) Compute the area between the y axis and the curve x = y 2
7y + 6.
(16) Compute the area between the y axis and the curve x = y 3
11y 2 + 6y 6.
(17) Compute the area between the curves y = x2 and y = 32 x2 .
(18) Compute the area between the three curves f (x) = x, g(x) =
x, and h(x) = 4.
(19) Compute the area between the curves y = |x| and y = x2 12.
(20) Compute the area between the curves y = |x| and y = 20 x2 .

37. Volumes
37.1. Extending the Definition of Volumes. If a solid S can be built up

using unit cubes, then we can simply say that the volume V (S) of S is
the number of unit cubes used to build S. However, if the borders of
S are not planes, then this method will have to be modied. A ball or
a cone is an example of this.
So we would like to dene the notion of volume so that it is applicable to a large class of solids, not just to those solids that are bordered
by planes. This denition should agree with our intuition. It should
also be in accordance with the fact that we can approximate all solids
with a collection of very small cubes; therefore, V (S) must be close to
the number of unit cubes used in the approximation.

37. VOLUMES

With these goals in mind, we recall that we already dened the area
of a domain in the plane whose borders are the graphs of continuous
functions. Building on that denition, we say that the volume of a
prism is its base area times its height. More formally, let S be a solid
whose base and cover are identical copies of the plate P , located at
distance h from each other, on two parallel planes that are at distance
h from each other. Then we dene the volume of S to be
V (S) = A(P )h,
where A(P ) is the area of P . See Figure 6.7 for an illustration. In
particular, the volume of a cylinder whose base is a circle of radius r
and whose height is h is r2 h.
Now let S be any solid located between the planes given by the
equations x = a and x = b. In order to dene and compute the volume
V (S) of S, we cut S into n parts by the planes x = xi for i = 0, 1, . . . , n,
where
a = x0 < x1 < < xn = b.
Let n be large. Then the part Si of S that is between the planes
x = xi1 and x = xi is well approximated by a prism Pi described as
follows. The height of Pi is x = (b a)/n, the base plate and cover
plate of Pi are on the planes x = xi1 and x = xi , respectively, and the
base and cover plates of Pi are congruent to the intersection Ti of S and
the plane x = xi for some point xi [xi1 , xi ]. We can assume that xi
is the midpoint of [xi1 , xi ], but that will turn out to be insignicant.
Note that Pi has volume A(Ti ) x.

Figure 6.7. The volume of S is dened by V (S) = A(P )h.

10

6. APPLICATIONS OF INTEGRATION

Figure 6.8. The volume of the ball B approximated by cylinders.


If n is large, then the union of the prisms Pi approximates S well,
so V (S) should be dened in a way that assures that V (S) is close to
n
n


V (Pi ) =
A(Ti ) x.
(6.1)
i=1

i=1

As n goes to innity, the Riemann sum on the right-hand side of


(6.1) has a limit. We dene that limit to be the volume V (S) of S, so
n

A(Ti ) x.
V (S) = lim
n

i=1

On the other hand, by the denition of the denite integral, we have


 b
n

lim
A(Ti ) x =
A(t) dt,
n

i=1

where A(t) is the area of the intersection of S and the plane x = t.


Therefore, this integral is equal to the volume V (S) of the solid S.
That proves the following theorem, which will be our main tool in this
section.
Theorem 6.3. Let S be a solid located between the planes x = a
and x = b, and let A(t) be the area of the intersection of S and the
plane x = t. Then the volume V (S) of S satises the equation
 b
A(t) dt.
V (S) =
a

Example 6.5. Compute the volume of the ball B whose center is


at the origin and whose radius is r.
Solution: For any given t [r, r], the intersection of the plane
x = t and the ball B is a circle. Let Ct denote this circle. By the

37. VOLUMES

11

Pythagorean theorem, Ct has radius r2 t2 , and therefore, the area


of Ct is A(t) = (r2 t2 ). See Figure 6.8 for an illustration.
Now we can use Theorem 6.3 to compute the volume of B. We get




r2 t2 dt
r

 r
1

= r2 t t3 

3
r

V (B) =

2
= 2r3 r3
3
4 3
= r .
3
2
There is nothing magical about the x axis as far as Theorem 6.3 is
concerned. The argument that yielded that theorem can be repeated
for the y axis instead of the x axis, yielding the following theorem.
Theorem 6.4. Let S be a solid located between the planes y = a
and y = b, and let B(t) be the area of the intersection of S and the
plane y = t. Then the volume V (S) of S satises the equation

V (S) =

B(t) dt.
a

Example 6.6. Let S be the right circular cone whose symmetry


axis is the y axis, whose apex is at y = h, and whose base is a circle
in the plane y = 0 with its center at the origin and with radius r. Find
the volume of S.
Solution: The cone S is between the planes y = 0 and y = h, and
B(t) of Theorem 6.4 is easier to compute than A(t) of Theorem 6.3, so
we use the former.
The intersection of the plane y = t and S is a circle. The radius rt
of this circle, by similar triangles, satises
ht
rt
=
,
r
h

12

6. APPLICATIONS OF INTEGRATION

showing that rt = r(h t)/h. Therefore, B(t) = r2 (h t)2 /h2 , and


Theorem 6.4 implies


(r2 (h t)2 /h2 ) dt


0

r2 h 2
= 2
(h 2ht + t2 ) dt
h 0

 h
2
3
r
t 
= 2 h2 t ht2 +

h
3 

V (S) =

r2 h3
= 2
h
3
1 2
= hr .
3
See Figure 6.9 for an illustration.

37.2. Annular Rings. In the examples that we have solved so far, the

computation of A(t) or B(t), that is, the computation of the area of


the intersection between a solid and a horizontal or vertical plane, was
not dicult. That computation could be done directly.
There are situations in which the domains whose areas we need to
compute are not convex; that is, visually speaking, there is a hole in
them. This happens particularly often when S is obtained by rotating
a domain D around a line.

Figure 6.9. Right circular cone.

37. VOLUMES

13

Example 6.7. Let D be the domain between the two curves y =


x2 = f (x) and y = 2x = g(x) and let S be the solid obtained by
rotating D about the line x = 3. Find the volume of S.
Solution: The two curves intersect at the points (0, 0) and (2, 4). The
intersection of the horizontal plane y = t with S has the form of an
annular ring, which is sometimes informally called a washer. This is
simply a smaller circle cut out o the middle of a larger circle, so that
the two circles are concentric. If the larger circle has radius r1 and the
smaller circle has radius r2 , then the annular ring has area (r12 r22 ).
This general recipe enables us to compute B(t) in the example at
hand. The points in D satisfy x [0, 2] and y [0, 4]. As 0 x 2,
the inequality x2 2x holds. So, for t [0, 4], the point Pi = (t/2, t)
on the
graph of g(x) = y = 2x is closer to the y axis than the point
Po = ( t, t) on the graph of y = x2 = f (x). (It takes a larger value
of x to get the same value t = y by f than it takes to get the same
value by g.) So the outer circle of the annulus will be given by the
rotated image of the curve of f (the parabola), and the inner circle of
the annulus will be given by the rotated image of the curve of g (the
straight line).
In particular, for xed t, the radii are obtained as the distance of
Po (resp. Pi ) from the axis of rotation, that is, the line x = 3. For
the inner radius, this yields



 t+6
t
,
r2 = 3  =
2
2
while for the outer radius, this yields



r1 = 3 t = t + 3.
Therefore, the area of the annular ring that is the intersection of S
and the plane y = t is given by


2


2
t2
t+6
2
2
0.5
t+3
= 6t 2t .
B(t) = (r1 r2 ) =
2
4
Now we can apply Theorem 6.4 to compute V (S). We obtain
 4
V (S) =
B(t) dt
0
 4
t2
6t0.5 2t dt
=
4
0
 4

3
t

= 4t1.5
t2 

12
77.01.

14

6. APPLICATIONS OF INTEGRATION

37.3. Special Cases of Theorems 6.3 and 6.4. Note that the solids we

discussed so far in this section could be obtained by rotating the graph


of a function around an axis. Such volumes are called volumes of revolution. Indeed, the ball of Example
6.5 can be obtained by rotating
the graph of the function f (x) = r2 x2 (a semicircle) about the x
axis. The cone of Example 6.6 can be obtained by rotating the graph
of the function f (x) = x hr + r = y (a straight line) about the y axis.
For such solids, the areas A(t) and B(t) appearing in Theorems 6.3
and 6.4 are easy to compute, since the intersections appearing in those
theorems will be circles or annular rings. If S is a solid obtained by
rotating a curve about the x axis, then the intersection of the plane
x = t and S is a circle of radius f (x), and hence A(t) = f (x)2 . If S is
a solid obtained by rotating the curve of the function g(y) = x about
the y axis, then the intersection of S and the plane y = t is a circle
of radius g(t), and so B(t) = g(t)2 . This yields the following special
versions of Theorems 6.3 and 6.4.
Theorem 6.5. Let S be a solid between the planes x = a and x = b
obtained by rotating the graph of the function f (x) = y about the x
axis. Then we have

b

f (t)2 dt.

V (S) =
a

Theorem 6.6. Let S be a solid between the planes y = a and y = b


obtained by rotating the graph of the function g(y) = x about the y axis.
Then we have

b

g(t)2 dt.

V (S) =
a

The exercises at the end of this section will provide further examples
for the uses of these theorems.
If the domain to be rotated does not include the entire area between
the curve and and coordinate axis (for instance, because it is a domain
between two curves), then we get annular rings, which we discussed in
the last section.
37.4. A Solid Not Obtained by Revolution. While volumes of revolution

are a very frequent application of Theorems 6.3 and 6.4, they are not
the only applications of those theorems.
Example 6.8. Let S be a pyramid whose base is a square of side
length a and whose height is h. Compute the volume of S.
Solution: The rst step is to place S in a coordinate system so that
Theorem 6.3 can be applied. Let us place the axis of S on the x axis of

37. VOLUMES

15

the coordinate system, so that the center of the base of S is at the origin
and the cusp of S is at x = h. This does not completely determine the
position of S, because S could still rotate around the x axis. However,
such rotations will not change the value of A(t) for any t [0, h], and
so they are insignicant for the computation of V (S).
Now note that, for any t [0, h], the intersection of S and the plane
x = t is a square of side length a(h t)/h (by similar triangles). See
Figure 6.10 for an illustration. So A(t) = a2 (h t)2 /h2 , and Theorem
6.3 implies
 h 2
a (h t)2
dt
V (S) =
h2
0

 h
2
3
t 
a
= 2 h2 t ht2 +

h
3 
0

a2 h
=
.
3

37.5. The Big Picture. Note that the theorems presented in this sec-

tion on integrals conrm our intuition that if a certain function measures a quantity, then, under the appropriate conditions, the integral
of that function measures a quantity that is somehow in a space that
is one dimension higher. For instance, we saw earlier that if f (x) measured the height of a curve at a given horizontal coordinate x, then,

Figure 6.10. Pyramid.

16

6. APPLICATIONS OF INTEGRATION


under the appropriate conditions, f (x) dx measured the area under
the curve. So taking integrals meant moving from one dimension to
two. In this section, the functions A(t) and B(t)
 measured areas
of domains in a given plane, while A(t) dt and B(t) dt measured
volumes. So taking integrals meant moving from two dimensions to
three.
37.6. Exercises.

(1) Let T be the triangle whose borders are the coordinate axes
and the line y + 4x = 4. Rotate T about the y axis and
compute the volume of the obtained solid.
(2) Compute the volume of the solid between the planes x = 1
and x = 1 obtained by rotating the graph of the function
f (x) = x2 = y about the x axis.
(3) Compute the volume of the solid between the planes x = 2
and x = 2 obtained by rotating the graph of the function
f (x) = x4 = y about the x axis.
(4) Compute the volume of the solid between the planes x = 1
and x = 1 obtained by rotating the graph of the function
f (x) = x3 + 2x = y about the x axis.
(5) Compute the volume of the solid between the planes x = 3
and x = 3 obtained by rotating the graph of the function
f (x) = 2x2 + 5x = y about the x axis.
(6) Compute the volume of the solid between the planes x = 0
and x = obtained by rotating the graph of the function
f (x) = sin x = y about the x axis.
(7) Compute the volume of the solid between the planes x = 0
and x = /4 obtained by rotating the graph of the function
f (x) = tan x = y about the x axis.
(8) Compute the volume of the solid
obtained by rotating the
domain between the curves y = x and y = x about the line
y = 2.
(9) Compute the volume of the solid obtained by rotating the
domain between the curves y = x4 and y = x about the line
x = 1.
(10) Compute the volume of the solid obtained by rotating about
the x axis the domain between the x axis and the curve y = e2x ,
between the planes x = 2 and x = 4.
(11) Compute the volume of the solid obtained by rotating about
the
x axis the domain between the x axis and the curve y =

3
x, between the planes x = 0 and x = 6.

38. CYLINDRICAL SHELLS

17

(12) Compute the volume of a regular tetrahedron of side length z.


A regular tetrahedron is a solid that has four faces, each of
which is a regular triangle.
(13) Let P be a pyramid whose base is a triangle of area A and
whose height is h. Find and prove a formula for the volume of
P.
(14) Let P be a pyramid whose base is a rectangle of area A and
whose height is h. Find and prove a formula for the volume of
P.
(15) Compute the volume of the solid between the planes x = 0 and
x = 1 obtained by rotating the curve f (x) = ex = y about the
y axis.
(16) Compute the volume of the solid between the planes x = 0
and x = 1 obtained by rotating the curve of the function
f (x) = x(1 x) about the x axis.
(17) Compute the volume of the solid obtained by rotating the
domain between the curves y = x3 and y = x4 about the y
axis.
(18) Let D be the domain that lies above the x axis and on the
right of the y axis and is bordered by the curve y = 9 x2 .
Rotate D about the line x = 1. Compute the volume of the
obtained solid.
(19) Let D be dened as in the preceding exercise. Rotate D about
the line y = 1. Compute the volume of the obtained solid.
(20) Use Theorem 6.6 to deduce the formula for the volume of a
sphere of radius r.
38. Cylindrical Shells
38.1. An Alternative Method to Compute Volumes. In principle, Theo-

rems 6.3 and 6.4 are simple methods to compute the volumes of solids.
In practice, however, the areas A(t) and B(t) that appear in these theorems may be dicult to explicitly evaluate. One situation in which
these areas are often dicult to compute is when the solid in question
is obtained by rotating a domain around some line; that is, it is a solid
of revolution.
As an example, let us try to compute the volume of the solid S
obtained by rotating about the y axis the domain bordered by the lines
x = 0, x = 3, and y = 0 and the graph of the function f (x) = 3x3 x4 .
If we try to solve this problem using Theorems 6.3 or 6.4, we run into
diculties, because A(t) and B(t) will not be easy to compute. For
instance, if we wanted to use Theorem 6.4, then, in order to compute

18

6. APPLICATIONS OF INTEGRATION

Figure 6.11. Single cylindrical shell.


B(t), we would need to describe the intersection of S and the plane
y = t. For this, we would have to nd the x coordinates of the points
of that intersection; that is, we would need to nd all real numbers
x [0, 3] for which y = 3x3 x4 = t. This is a fourth-degree equation
for x, which is very dicult and cumbersome to solve. If we wanted
to use Theorem 6.3, then, in order to compute A(t), we would need
to describe the intersection of the plane x = t and S, which is not
straightforward to do.
In situations like this, that is, when the application of Theorems
6.3 and 6.4 leads to technical diculties, it often helps to use another
method called the method of cylindrical shells. A cylindrical shell C
is simply a cylinder C1 of which a smaller cylinder C2 is removed, so
that C1 and C2 have the same symmetry axis. See Figure 6.11 for an
illustration. If C2 is just a little bit smaller than C1 , then C looks like
a shell, explaining the name cylindrical shell.
If C1 and C2 both have height h and Ci has radius ri , then the
volume of C can be computed as
V (C) = V (C1 ) V (C2 )
= hr12 hr22
= h(r12 r22 ).

38. CYLINDRICAL SHELLS

19

Note that the last form of V (C) can be rearranged as


2(r1 + r2 )
.
2
This way of writing V (C) might seem contrived at rst sight. However, it has the following motivation. Note that r1 r2 is the width
of C, while h is its height. Finally, if we atten C out in the plane,
2
, since
it will become a brick with side lengths h, r1 r2 , and, 2 r1 +r
2
the length of the missing side is equal to the circumference of a circle
whose radius is the average of the radii of C1 and C2 .
In other words, (6.2) says that the volume of a cylindrical shell is
equal to the product of its height, width, and length (if the latter is
interpreted properly).
We are now in a position to use cylindrical shells to compute volumes. Let S be a solid that is obtained by rotating the domain D,
which lies below the curve of f (x) = y from x = a to x = b, about the
y axis. In order to estimate V (S), we cut [a, b] into n intervals of equal
length using points
(6.2)

V (C) = h (r1 r2 )

a = x0 < x1 < < xn1 < xn = b.


For each integer i [1, n], we will take a cylindrical shell Si , which
will roughly cover the part of S that is obtained by rotating the part
of D that is between the lines x = xi1 and x = xi about the y axis.
More precisely, this shell will be obtained by removing the cylinder Ci,2
from the cylinder Ci,1 , where Ci,1 and Ci,2 are both cylinders whose
symmetry axis is the y axis and whose height is f (xi ) for the midpoint
xi of the interval [xi1 , xi ]. The radius of Ci,1 is f (xi ), and the radius
of Ci,2 is f (xi1 ).
If we set x = (b a)/n, then (6.2) implies that
V (Si ) = f (xi ) x 2xi ,
since xi is the midpoint of the interval [xi1 , xi ]. Summing the last
displayed equation over all i, we get
(6.3)

V (S)

n


f (xi ) x 2xi ,

i=1

since the union of the shells Si has roughly identical volume to S.


As n goes to innity, this approximation gets better and better,
and the Riemann sum on the right-hand side of (6.3) converges to the
corresponding denite integral. Hence, we have proved the following
theorem.

20

6. APPLICATIONS OF INTEGRATION

Figure 6.12. (a) The curve of y = 3x3 x4 and (b) the


solid obtained by its rotation.
Theorem 6.7. The volume V (S) of the solid obtained by rotating
the domain D whose borders are the curve of f (x) = y, the lines x = a
and x = b, and the horizontal axis y = 0 about the y axis is equal to
 b
V (S) =
2xf (x) dx.
a

Example 6.9. Compute the volume of the solid S obtained by rotating about the y axis the domain bordered by the lines x = 0, x = 3,
and y = 0 and the graph of the function f (x) = 3x3 x4 .
Solution: By Theorem 6.7, we have
 3
V (S) = 2
x(3x3 x4 ) dx
0
 3
(3x4 x5 ) dx
= 2
0
 5
 3
3x
x6 
= 2


5
6 

0
729 729
= 2

5
6
= 152.677.
2
The axis around which we rotate a domain does not have to be
a coordinate axis in order for the method of cylindrical shells to be
applicable. We can apply the method as long as we can decompose the
solid in question into cylindrical shells whose height and radius we can
compute.

38. CYLINDRICAL SHELLS

21

Figure 6.13. (a) The curve of y = x2 /2 and (b) the


solid obtained by its rotation.
Example 6.10. Let S be the solid obtained by rotating the domain
whose borders are the horizontal axis, the vertical lines x = 0 and
x = 2, and the graph of the function f (x) = 2x x2 about the vertical
line x = 3. Compute V (S).
Solution: We can decompose S into cylindrical shells whose center
is on the vertical line x = 3. The shell containing the point x of
the horizontal axis will have height f (x) = 2x x2 and radius 3 x.
Therefore, we have
 2
V (S) =
2(2x x2 )(3 x) dx
0
 2
 3

x 5x2 + 6x dx
= 2
0
 4
 2
x
5x3

= 2

+ 3x2 

4
3
0

8
= 2
3
16.755.
2
38.2. Exercises.

In (1)(8), use the method presented in this section to compute the


volume of the solid obtained by rotating the domain between the given
curves about the y axis.

22

6. APPLICATIONS OF INTEGRATION

(1)
(2)
(3)
(4)
(5)
(6)
(7)
(8)

y
y
y
y
y
y
y
y

= f (x) = x3 , x = 1, y = 0.
= f (x) = x12 , x = 2, x = 3, y = 0.
= f (x) = x,
g(x) = x, x = 2.

3
= f (x) = x, g(x) = x3 , y = 0, y = 1.
= f (x) = x4 , x = 0, y = 1.
= f (x) = 1 x2 , x + y = 1.
= f (x) = ex , x = 0, y = 0, y = 1.
= f (x) = e2x , x = 0, x = 1, y = e2 .

In (9)(16), use the best available method to compute the volume of


the solid obtained by rotating the domain between the given curves
about the given axis.
(9)
(10)
(11)
(12)
(13)
(14)
(15)
(16)
(17)
(18)
(19)

(20)

= f (x) = x, y = g(x) = x, y = 2, about x = 1.


= f (x) = x, y = g(x) = x, y = 2, about x = 3.
= f (x) = x, y = g(x) = x, y = 2, about the x axis.
= f (x) = 6 x, y = g(x) = x, y = 6, about the y axis.
= f (x) = 6 x, y = g(x) = x, y = 6, about the line y = 6.
= f (x) = 6 x, y = g(x) = x, y = 6, about the line y = 7.
= f (x) = (x2 + 4x 21), y = 0, about the y axis.
= f (x) = x + (1/x), y = 4, about x = 1.
1
Describe a solid whose volume is given by 0 2x cos x dx.
1
Describe a solid whose volume is given by 2 0 x4x
2 +1 dx.
Use a computer software package to compute the volume of
the solid obtained by rotating the region between the curves
2
y = ex and y = ex about the y axis.
Use a computer software package to compute the volume of
the solid obtained by rotating the region between the curves
y = sin x and y = sin(x2 ) and the lines x = 0 and x = 1 about
the y axis.

y
y
y
y
y
y
y
y

39. Work and Hydrostatic Force


39.1. Work Moving a Pointlike Object. In physics, the word work has

a more specic meaning than in everyday life. Work in physics means


that a force is extended to move an object a certain distance.
The force F moving an object is computed by the formula
F =m

d2 s
,
dt2

which is called Newtons second law. Here m is the mass of the object,
2
while a = ddt2s is its acceleration. So Newtons second law says that the

39. WORK AND HYDROSTATIC FORCE

23

mass of an object is in direct proportion to the force needed to move


it at constant acceleration.
If a constant force F is exerted while an object moves distance d,
then the work done by that constant force is computed by the formula
W = F d.
Note that in the metric system, distance is measured in meters (m),
time is measured in seconds (s), and therefore acceleration is measured
in m/s2 . Mass is measured in kilograms (kg), so force is measured in
kg sm2 , which are called newtons (N), and, nally, work is measured in
N m, which are also called joules (J). One joule is the work that is
done when a force equal to 1 newton moves an object a distance of 1
meter.
Example 6.11. How much work is needed to lift a child of 20 kg to
a height of 0.5 meters? Use the fact that gravitation causes downward
acceleration of g = 9.8 m/s2 .
Solution: In order to lift the child, one needs to overcome the downward acceleration caused by gravity. This means that an upward force
of
m
m g = 20 kg 9.8 2 = 198 N
s
has to be exerted across a distance of d = 0.5 meters. This yields
W = F d = 198 N 0.5 m = 99 J.
So the work needed is 99 J.
2
If the force exerted is not constant across the entire distance, but
the distance can be split up into a few parts so that the force is constant
on each part, then we can compute the work done by the force on each
part just as in the previous example, and then we can add the obtained
amounts to get the total amount of work done across the entire distance.
If the force exerted changes according to a continuous function f (d),
then we can approximate the work done using the idea of the previous
paragraph and then use integration to compute the total work done by
the force as follows.
Let a and b be real numbers and let us assume that an object is
moving from a to b, and the force moving the object at a given point x
is equal to f (x), where f is a continuous function. In order to compute
the work done across the entire distance, let us split the interval [a, b]
into n equal intervals, using points
a = x0 < x1 < < xn1 < xn = b,

24

6. APPLICATIONS OF INTEGRATION

and set x = (ab)/n. Then the work done by the force on the interval
Ii = [xi1 , xi ] is about f (xi ) x, where xi is some sample point in Ii .
Indeed, f is continuous, so if n is large and therefore Ii is short, then
f does not change much on that interval, so the shape of the domain
under the curve of f and above Ii is roughly a rectangle. This means
that the total work done by the force on [a, b] is close to
(6.4)

n


f (xi ) x.

i=1

As n gets larger, this approximation gets better, and so we dene the


total work done by the force across the interval [a, b] as the limit of the
sum in (6.4) as n goes to innity. On the other hand, that sum is a
b
Riemann sum, so its limit, as n , is the denite integral a f (x) dx.
In other words, we have proved the following theorem.
Theorem 6.8. Let a and b be real numbers. If an object is moved
from a to b by a force that is equal to f (x) at point x, where f is
continuous on [a, b], then the total work done by the force on [a, b] is
 b
W =
f (x) dx.
a

Example 6.12. The force needed to extend a given spring x centimeters over its natural length is given by the function f (x) = 70x.
How much work is needed to extend the spring 10 cm over its natural
length?
Solution: By Theorem 6.8, we have
 0.1
70x dx
W =
0
0.1
2
= 35x 
0

= 0.35 J.
So 12.25 J of work is needed to stretch the spring 10 cm over its natural
length.
2
We point out that the law of physics that says that the force needed
to extend a spring by x units over its natural length is equal to kx is
called Hookes law, and k is called the spring constant.
39.2. Hydrostatic Force. Let us say that we want to pump water out

of a tank that has the shape of the southern half of a ball of radius
1 (m). How much work is needed to do that? This question is more

39. WORK AND HYDROSTATIC FORCE

25

*
i

*
i

Figure 6.14. The tank in a coordinate system, and its


layer at height z.
complex than the previous one since deeper layers of the hemisphere
are smaller, and water in those layers has to travel farther in order to
reach the top of tank.
Therefore, we will cut the tank up into small layers and estimate
the amount of work needed to pump out each layer of water.
Let x = 0 denote the bottom of the tank and let x = 1 denote
the center of the top circle of the tank. Cut the tank into i horizontal
layers by planes that are at heights
0 = x0 < x1 < < xn = 1.
Here xi xi1 = 1/n = x for all i. Let Li denote the ith layer.
The shape of this layer is close to a cylinder of height x. Each water
particle in this layer has to be pumped at a distance roughly equal
to 1 xi , where xi is a point in [xi1 , xi ]. The square of the radius
of the cylinder approximating Li is, by the Pythagorean theorem, 1
(1 xi )2 = 2xi (xi )2 , and therefore the volume of Li is close to
Vi = x(2xi (xi )2 ). See Figure 6.14 for an illustration.
One cubic meter of water has a mass of 1000 kg, so the density of
water is = 1000 kg/m3 . Therefore, the mass of the water in Li is
close to mi = Vi . In order to pump this water out of the tank, the
downward acceleration caused by gravitation, that is, mi g, has to be
overcome, across a distance of 1 xi . Therefore, the work needed to
pump out the water in Li is about mi g(1 xi ), and the work needed
to pump out all the water in the tank is approximated by
(6.5)

n

i=1

mi g(1

xi )

= g

n

i=1

x(2xi (xi )2 )(1 xi ).

26

6. APPLICATIONS OF INTEGRATION

As n grows, the expression displayed in (6.5) approximates the


needed work better and better. We dene the total work needed (to
pump all the water out of the tank) to be the limit of the sum shown
in (6.5) as n goes to innity. As that sum is a Riemann sum, its limit,
as n goes to innity, is the denite integral
 1
(2x x2 )(1 x) dx.
W = g
0

As the integrand is a polynomial, this integral is very easy to compute.


We get that
 1
(x3 3x2 + 2x) dx
W = g
0
 1
 4
x

= g
x3 + x2 

4
0

= g/4
= 7696.675 J.
So it takes almost 7700 J of work to pump out all the water from the
tank.
39.3. Exercises.

(1) How much work is done when a book of mass 2 kg is lifted 1.5
meters from its original location?
(2) How much work is done when a suitcase of mass 15 kg is lifted
0.5 meters from its original location?
(3) A suitcase of mass 20 kg is lifted 0.6 meters and put on a chair.
Some objects are removed, and then the suitcase, which now
has a mass of 12 kg, is lifted an additional 0.4 meters. How
much work was done during the total ascent of the suitcase?
(4) If it takes 10 J of work to lift an object 2 meters, how much
work does it take to lift that object an additional 3 meters?
(5) An elevator and the passengers entering it on the rst oor
have a total mass of 2000 kg. The distance between two adjacent oors served by this elevator is 6 meters. How much work
is done if the elevator rst stops on the third oor, where two
additional people of mass 75 kg each enter, and then the elevator stops on the sixth oor?
(6) If it takes 20 J of work to stretch a spring 20 cm over its natural
length, how much work does it take to stretch that spring an
additional 5 cm?

39. WORK AND HYDROSTATIC FORCE

27

(7) If it takes 30 J of work to stretch a spring 10 cm over its natural


length, how much work does it take to stretch that spring an
additional 2 cm?
(8) Let us assume that 8 J of work is needed to stretch a spring
from 20 cm to 21 cm, and 10 J of additional work is needed
to stretch that same spring from 21 cm to 22 cm. What is the
natural length of the spring?
(9) A rope of mass 5 kg and length 30 m is lying on the ground.
How much work is needed to lift one end of that rope to a
height of 30 meters?
(10) We have n copies of a book, with a mass of 2 kg each. How
much work is needed to arrange these copies in a tower if each
copy is 0.08 meters thick?
(11) How much work is needed to pump out all the water from a
tank of the shape of a cube of side length 1? (The length is
measured in meters.)
(12) How much work is needed to pump out all the water from a
tank of the shape of an inverted cone of height 10 whose top
circle has radius 2?
(13) How much work is needed to pump out all the water from a
tank of the shape of an inverted cone of height 25 whose top
circle has radius 5?
(14) Consider the tank of the previous exercise. How much work
is needed if we want to pump out half the water from this
tank? You can use a software package to solve the high-degree
polynomial equation encountered while solving this exercise.
(15) How much work is needed to pump out all the water from a
tank of the shape of a cylinder of height 20 whose base circle
has radius 30?
(16) How much work is needed to pump out all the water from a
tank of the shape of an inverted pyramid of height 15 whose
top plate is a square of side length 10?
(17) Consider the tank of the previous exercise. How much work
is needed if we want to pump out half the water from this
tank? You can use a software package to solve the high-degree
polynomial equation encountered while solving this exercise.
(18) How much work is needed to pump out all the water from a
tank of the shape of a regular tetrahedron of side length 1?
(19) Consider the tank of the previous exercise. How much work
is needed if we want to pump out seven-eighths of the water
from this tank?

28

6. APPLICATIONS OF INTEGRATION

(20) Consider the tank discussed in Section 39.2. How much work
is needed if we want to pump out half the water from this
tank? You can use a software package to solve the high-degree
polynomial equation encountered while solving this exercise.
40. Average Value of a Function
The concept of average is a simple one as long as we take the average
of a nite number of values, such as the average price of a house in a
given neighborhood or the average daily high temperature in a given
city in a given month. If a1 , a2 , . . . , an are real numbers, then
(6.6)

A = (a1 + a2 + + an )/n

is their average. However, what can we say about the average value
of a function over a given interval [a, b]? We will clearly need a new
denition for that since there are innitely many real numbers in [a, b],
so summing all of them and then dividing their sum by the number of
summands is not an option.
Here is an intuitive way of extending the denition of average to
the values taken by a function over an interval. It follows from (6.6)
that A is the only real number with the property that if we replace
each of a1 , a2 , . . . , an by A, then the sum (a1 + a2 + + an ) does not
change. This observation suggests the following denition.
b
Definition 6.1. Let f be a function such that a f (x) dx exists.
Then the average value of f on the interval [a, b] is the real number
b
f (x) dx
c= a
.
ba
Indeed, c is the only real number with the property that
 b if we replace
f by the constant function f (x) = c, then the integral a f (x) dx does
not change.
A more systematic approach
 b is the following. As we saw when we
rst learned about integrals, a f (x) dx can be approximated in the
following way. Split [a, b] into n equal intervals and choose a point xi
in the ith such interval. Take a rectangle of height f (xi ) over the ith
interval. The average value of the n values of f taken at the points xi
is, of course,
f (x1 ) + f (x2 ) + + f (xn )
An =
.
n

40. AVERAGE VALUE OF A FUNCTION

29

Figure 6.15. The average value of sin x on [0, ].


On the other hand, the total area of the n rectangles we have just
dened is
ba
Rn =
(f (x1 ) + f (x2 ) + + f (xn )).
n
Comparing the last two displayed equations, we see that
Rn
(6.7)
An =
.
ba
If n goes to innity, then the n rectangles will approximate the domain
under
the graph of f , and so the right-hand side of (6.7) will converge

b

f (x) dx

to a ba , while the left-hand side will approximate the average value


of f on [a, b].
Example 6.13. What is the average value A of sin x on the interval
[0, ]?
Solution: We have

A=
=

sin x dx


cos x
0

1 (1)
=

2
= .

See Figure 6.15 for an illustration.

30

6. APPLICATIONS OF INTEGRATION

It is worth pointing out that a continuous function f will actually


take its average value on each interval. This is the content of the
following theorem.
Theorem 6.9. [Mean Value Theorem] Let f be a continuous function on [a, b] and let c be the average value of f on [a, b]. Then there
exists a real number x [a, b] such that f (x) = c.
Proof. It suces to show that if m is the minimum of f on [a, b]
and M is the maximum of f on [a, b], then m c M , and our claim
follows from the intermediate value theorem.
We know that
 b
f (x) dx M (b a)
m(b a)
a

for obvious geometric reasons. Now divide all three terms by b a to


get m c M .
2
Example 6.14. There is a real number x [0, ] such that sin x =
2/.
Solution: This follows from the previous example and Theorem 6.9.
2
40.1. Exercises.

(1) Find the average value of xn on [0, 1].


(2) Compare the results of the previous exercise for n = 1, n = 2,
and n = 3. Explain the pattern that you detect.
(3) Find the average value of tan x on [0, /4].
(4) Find the average value of x2x+1 on [0, 2].
(5) Find the average value of ex
on [0, 1].
x
(6) Find the average value of e ex + 1 on [0, 3].
(7) Find the average value of 1/(1 x2 ) on [2,3].
(8) Find the average value of x cos(x2 ) on [0, ].
(9) Find the average value of cos x on [/2, /2]. Try to deduce
your answer from the result of Example 6.13 without additional computation.
(10) Which is larger, the average value of sin x or the average value
of cos x, if both averages are taken on the interval [0, /2]?
(11) Which is larger, the average value of sin x on [0, ] or the
average value of sin x on [14, 17]? Can you nd an answer
that does not involve computation?

40. AVERAGE VALUE OF A FUNCTION

31

(12) Prove that if f (x) g(x) for all x [a, b], and f and g are
continuous functions on [a, b], then the average value of f on
[a, b] is at least as large as the average value of g on [a, b].
(13) Which is larger, the average value of f (x) = x2 on [2, 3] or the
average value of g(x) = x2 on [1/3, 1/2]?
(14) A bicyclist covered a certain distance at an average velocity of
25 miles per hour. Prove that there was a moment at which the
instantaneous velocity of the bicyclist was 25 miles per hour.
(15) Starting from a standstill, a car needed 10 seconds to reach a
traveling velocity of 20 meters per second. Prove that there is
a moment at which the instantaneous acceleration of the car
is 2 m/s2 .
4
(16) Let us assume that 0 f (x) dx = 10. Prove that there is a
number x [0, 4] such that f (4) = 2.5.
(17) Let f be an odd function. What is the relation between the average value of f on [a, b] and the average value of f on [a, b]?
(18) Let f be an odd function. Compute the average value of f for
any interval [R, R].
(19) Let f be an even function. What is the relation between the
average value of f on [R, R] and the average value of f on
[0, R]?
(20) Let a be a xed real number and let f be a continuous function.
Dene g(x) as the average value of f on the interval [a, x].
Compute g(x) for f (x) = 3, f (x) = 2x + 7, and f (x) = x2 +
2x + 1.

CHAPTER 7

Methods of Integration
41. Integration by Parts
41.1. Method of Integration by Parts. Let u and v be two dierentiable

functions of the variable x. We used the simple product rule


(7.1)

(uv) = u v + uv 

to compute the derivative of the product of these two functions. Is


there a similar rule for computing the integral of the product of two
functions? In general, the answer is no. There is no rule that provides
the integral of the product of two functions that would work in every
case. However, there are many cases in which a relatively simple way
of reversing the product rule of dierentiation will give us the answer
we are trying to obtain.
Indeed, integrating both sides of the product rule (7.1) of dierentiation (7.1) with respect to x, we get the identity



u(x)v(x) = (u (x)v(x)) dx + (u(x)v  (x)) dx
or, after rearrangement,



(7.2)
(u (x)v(x)) dx = u(x)v(x) (u(x)v  (x)) dx.
Formula (7.2) is very useful if we want to compute the integral of
the product of two functions, one of which can play the role of u and
the other
one of which can play the role of v. If wecan compute u,


and (uv ), then formula (7.2) enables us to compute (u v) as well. If
we cannot carry out one or both of these computations, then formula
(7.2) will not help.

Example 7.1. Compute xex dx.
Solution: We set u (x) = ex and v(x) = x. Then formula (7.2) is easy
to apply, since v(x) = x and v  (x) = 1. Therefore, (7.2) implies that


x
x
xe dx = e x ex 1 dx = ex x ex + C = ex (x 1) + C.
2
33

34

7. METHODS OF INTEGRATION

The reader is encouraged to verify that the obtained solution is


correct by computing the derivative of ex (x 1) and checking that it
is indeed equal to ex x.
At this point, the reader may be asking how we knew that we
needed to set u (x) = ex and v(x) = x, and not the other way around.
The answer is that the other distribution of roles, that is, u (x) = x
and v(x) = ex would not have helped. Indeed, if we had
u
 chosen

and
 x v2 in that way, we would have needed to compute (uv ) dx =
(e x )/2 dx. That would have been more complex than
the original



problem. We should always choose u and v so that (uv ) dx is easy
to compute. That usually means selecting v so that it becomes simpler
when dierentiated, and to select u so that u does not get much
more complex when integrated (or at least one of these two desirable
outcomes occur).

Example 7.2. Compute x cos x dx.
Solution: We set u (x) = cos x and v(x) = x, which means that
u(x) = sin x and v  (x) = 1. So formula (7.2) implies


x cos x dx = x sin x sin x dx = x sin x + cos x + C.
2
The technique of integration we have just explained is called integration by parts.
41.2. Advanced Examples. Sometimes the integrand does not seem to

be a product, but it can be transformed in to one. The following is a


classic example.

Example 7.3. Compute ln x dx.
Solution: The crucial observation is that writing ln x = 1 ln x helps.
Let u (x) = 1 and v(x) = ln x. Then u(x) = x and v  (x) = 1/x, so,
crucially, u(x)v  (x) = 1. Therefore, formula (7.2) yields



ln x dx = 1 ln x dx = x ln x 1 dx = x ln x x + C.
2
Sometimes integration by parts leads to an equation or a system of
equations that needs to be solved in order to get the solution to our
problem.

Example 7.4. Compute ex cos x.

41. INTEGRATION BY PARTS

35

Solution: We set u (x) = ex and v(x) = cos x. Then u(x) = ex and


v  (x) = sin x, and formula (7.2) yields


x
x
(7.3)
e cos x dx = e cos x + ex sin x dx.

So we could solve our problem if we could compute the integral ex sin
x dx. We can do that by applying the technique of integration by parts
again, setting u (x) = ex and v(x) = sin x. We obtain


x
x
(7.4)
e sin x dx = e sin x ex cos x dx.
Finally, note that (7.3) and
 (7.4) is a system of equations with unfor
knowns ex cos x dx and ex cos x dx. We can solve this
 system,
x
instance, by adding these two equations and noting that e sin x cancels. We get the equation


x
x
e cos x dx = e (cos x + sin x) ex cos x dx
or

ex
e cos x dx = (cos x + sin x) + C.
2
x

Note that substituting the


 obtained expression for
(7.4), we get a formula for ex sin x dx, namely,

ex
ex sin x dx = (sin x cos x) + C.
2

2
e cos x dx into
x

41.3. Definite Integrals. If we evaluate both sides of formula (7.2) from

a to b and we apply the fundamental theorem of calculus, we get the


identity
 b
b  b


(u v) dx = (uv)
(uv  ) dx.
(7.5)
a

Example 7.5. Evaluate

2
1

ln x dx.

Solution: As we saw in Example 7.3, we can set u(x) = x and v(x) =


ln x. Then u (x) = 1, v  (x) = 1/x, and formula (7.5) yields
 2
2  2

ln x = (x ln x)
1 dx = 2 ln 2 1.
1

36

7. METHODS OF INTEGRATION

41.4. Exercises.

(1)
(2)
(3)
(4)
(5)
(6)
(7)
(8)
(9)
(10)
(11)
(12)
(13)
(14)
(15)
(16)
(17)
(18)
(19)
(20)


Compute  x sin x dx.
Compute  x2 ex dx.
Compute  x2 cos x dx.
Compute  x ln x dx.
Compute  x2 ln x .
Compute  e2x sin x dx.
Compute  xex dx.
2
Compute  (x
+ cos x) dx.
Compute  x ln x dx.
Compute  x tan1 (x) dx.
Compute  x sin1 (x) dx.
Compute (ln x)2 dx.
2
Evaluate 1 x cos 2x dx.
1
Evaluate 0 x2 sin x dx.
1
Evaluate 0 xex dx.
1
Evaluate 0 xe2x dx.
3
Evaluate 0 x3 ex dx.
3
Evaluate 2 x ln x dx.
2
Evaluate 1 (ln x)2 dx.

Use integration by parts to compute (cos x)2 dx.

42. Trigonometric Integrals


42.1. Powers of sin and cos. In this section, we consider functions of the

form f (x) = sinm x cosn x and discuss techniques for their integration.
It seems natural to rst consider the cases when m or n is 0, that is,
when f is just a power of sin or cos. Even these special cases will break
up into further subcases. The easiest subcase is when the exponents
are even numbers. In that case, we can use the trigonometric identities
(7.6)

cos 2x = 2 cos2 x 1

and
(7.7)

sin 2x = 2 sin x cos x

to eliminate high powers of trigonometric functions in the integrand.



Example 7.6. Compute cos4 x dx.

42. TRIGONOMETRIC INTEGRALS

37

2x
Solution: Using (7.6), we get that cos2 x = 1+cos
, and so
2

2
1 + cos 2x
1 cos 2x cos2 2x
4
cos x =
= +
+
.
2
4
2
4

Applying (7.6) again, with 2x replacing x, we get that cos2 2x =


so the previous displayed equation turns into

1+cos 4x
,
2

3 cos 2x cos 4x
+
+
.
8
2
8
Having eliminated the powers of cos, the integration is easy to carry
out as follows:
cos4 x =

 


3 cos 2x cos 4x
cos x dx =
+
+
dx
8
2
8
3x sin 2x sin 4x
+
+
+ C.
=
8
4
32
4

2
The computation is more complex if the integrand is an odd power
of sin or cos. In that case, we separate one factor and convert the rest
into the other trigonometric function, using the rule cos2 x + sin2 x = 1.

Example 7.7. Compute sin3 x dx.
Solution: We have
sin3 x = sin x sin2 x = sin x (1 cos2 x) = sin x sin x cos2 x.
The advantage of this form is that it makes integration by substitution easy. Indeed, set u = cos x, then du/dx = sin x, and so


u3
cos3 x
2
sin x cos x dx = u2 du =
+C =
+ C.
3
3
Comparing
the two displayed equations of this solution and noting that

sin x dx = cos x, we get

cos3 x
sin3 x = cos x +
+ C.
3
2
The methods shown above can be used to compute the integral of
products of powers
cos x. In other words, the method allows
 ofmsin x and
n
us to compute cos x sin x dx as shown below.

Example 7.8. Compute cos2 x sin3 x dx.

38

7. METHODS OF INTEGRATION

Solution: Just as in Example 7.7, we separate one sin x factor. This


accomplishes two things. It allows us to convert the remaining even
number of sin x factors to cos x factors, and it allows us to integrate by
substitution.
Indeed,


2
3
cos x sin x dx = cos2 x sin2 x sin x dx

= cos2 x(1 cos2 x) sin x

= cos2 x sin x cos4 x sin x

= u2 du + u4 du
u 3 u5
+
+C
3
5
cos3 x cos5 x
=
+
+ C,
3
5
where we used the substitution u = cos x.
2
We can always proceed this way if at least one of m and n in the
integrand cosm x sinn x is odd. Indeed, in that case, after separating
one factor from that odd power, an even power remains, and that can
be converted to the other trigonometric function using the identity
sin2 x + cos2 x = 1. If both m and n are even, then we can use that
identity right away.

Example 7.9. Compute cos4 x sin2 x dx.
=

Solution: We have


4
2
cos x sin x dx = cos4 x(1 cos2 x) dx


4
= cos x cos6 x dx.

4
Now note that
 we6computed cos xdx in Example 7.6. You are asked
to compute cos x in Exercise 42.4.4. The dierence of these two
results then provides the solution of the present example.
2
42.2. Powers of tan and sec. When integrating a product of the form

tanm x secn x, we will use the identity sec2 x = tan2 x + 1 and the differentiation rules (tan x) = sec2 x and (sec x) = sec x tan x.
There are two easy cases, namely, when m is odd (and n is at least
1) and when n 2 is even.

42. TRIGONOMETRIC INTEGRALS

39

In the rst case, that is, when m is odd and n 1, we separate one
factor of tan x sec x and express the remaining factors in terms of sec x
by the identity 1 + sec2 x = tan2 x. Then we substitute u = sec x,
= tan x sec x.
which leads to du
dx

Example 7.10. Compute tan3 x sec x dx.
Solution: Following the strategy explained above, we have


3
tan x sec x dx = tan2 x tan x sec x dx

= (1 + sec2 x) tan x sec x dx

= (1 + u2 ) du
u3
+C
3
sec3 x
= sec x +
+ C.
3
= u +

2
In the second case, that is, when n 2 is even, we separate one
factor of sec2 x, express the remaining factors in terms of tan x using
the identity sec2 x = 1 + tan2 x, and substitute u = tan x, which leads
= sec2 x.
to du
dx

Example 7.11. Compute sec4 x dx.
Solution: We have



4

sec x dx =

sec2 x sec2 x dx

= (1 + tan2 x) sec2 x dx
= (1 + u2 ) du
u3
+C
3
tan3 x
= tan x +
+ C.
3

=u+

2
If we are not in these two easy cases, then there is no recipe that
will always work. We then need to have a separate strategy for each
problem. We will show examples of that in Exercises 42.4.13, 42.4.14,
42.4.15, and 42.4.16.

40

7. METHODS OF INTEGRATION

42.3. Some Other Trigonometric


Integrals.
in
 If our goal is to compute


tegrals of the form cos mx sin nx dx, cos mx cos nx dx, and sin mx
sin nx dx, then we can often make use of the following identities:
(7.8)

sin a cos b =

1
1
sin(a b) + sin(a + b),
2
2

(7.9)

cos a cos b =

1
1
cos(a b) + cos(a + b),
2
2

1
1
cos(a b) cos(a + b).
2
2

Example 7.12. Compute cos 3x cos 5x dx.

(7.10)

sin a sin b =

Solution: Using (7.9) with a = 3x and b = 5x and noting that


cos 2x = cos 2x, we get that cos 3x cos 5x = 12 cos 2x + 12 cos 8x, and
so


 
1
1
cos 3x cos 5x dx =
cos 2x + cos 8x dx
2
2
1
1
= sin 2x +
sin 8x + C.
4
16
2
42.4. Exercises.


Compute  cos3 x dx.
Compute  sin5 x dx.
Compute  cos5 x dx.
Compute  cos6 x dx.
Compute  sin4 x dx.
Compute  sin3 x cos2 x dx.
Compute  sin x cos3 x dx.
Compute  sin3 x cos3 x dx.
Compute  sin2 x cos2 x dx.
Compute  sin4 x cos4 x dx.
Compute  tan2 x sec4 x dx.
Compute  tan3 x sec5 x dx.
Compute tan5 x dx by separating one factor of tan2 x in the
integrand and expressing it in terms of sec2 x.
(14) Compute  tan3 x dx using integration by parts.
(15) Compute sec3 x dx using integration by parts, with u (x) =
sec2 x and v(x) = sec x.
(1)
(2)
(3)
(4)
(5)
(6)
(7)
(8)
(9)
(10)
(11)
(12)
(13)

43. TRIGONOMETRIC SUBSTITUTION

(16) Compute
7.11.)
(17) Compute
(18) Compute
(19) Compute
(20) Compute

41

sec5 x dx. (Hint: Recall the result of Example


 sin x cos x dx.
 sin 2x cos 4x dx.
 sin 3x sin 7x dx.
cos 2x cos 4x dx.

43. Trigonometric Substitution


43.1. Reversing the Technique of Substitutions. Let us assume that we

want to compute the area of a circle by viewing one-fourth of that circle


as the domain under a curve. Let r be the radius of the circle, and let
us place the center of the circle at the origin. Then the northeastern
quarter of the circle, shown inFigure 7.1, is just the domain under the
graph of the function f (x) = r2 x2 , where x ranges from 0 to r. In
other words, we need to compute the integral
 r
r2 x2 dx.
(7.11)
0

In Chapter 5, we presented the technique of integration by substitution. This technique worked in situations when the best way to compute an integral was to dene a simple function of x, such as y(x) = x2 ,
and then continue the integration in terms of that new variable y.
In order to compute the integral in (7.11), we use the reverse of
the strategy mentioned in the previous paragraph. We dene another
variable y so that x is a simple function f of y. It is important to
dene f and y so that f is one-to-one, since that assures that f (y) = x
is equivalent to f 1 (x) = y.
In computing the integral in (7.11), we can set x = r sin y. Then
dx/dy = r cos y, and the limits of integration are y = 0 and y = /2.

Figure 7.1. The northeastern quadrant of the unit circle.

42

7. METHODS OF INTEGRATION

This yields
 r


r2

x2

/2

dx =

=r


r2 r2 sin2 y r cos y dy

/2


1 sin2 y dy


=r

/2

cos2 y dy
0

/2
y + sin 2y 
=r

2
0

= r2 .
4

Note that we could write cos y for 1 sin2 y, since 0 y /2,
and in that interval, cos y is nonnegative.
The preceding computation resulted in nding a denite integral.
We point out that by converting the indenite integral
y + sin 2y
back to a function of x using the rule x = r sin y, we
r2
2
get the corresponding indenite integral, that is


x 1
r2
sin1
+ x r2 x2 + C.
r2 x2 dx =
2
r
2
2

The result that we are going to compute in the next example will be
useful in the next section, when we will learn a technique to integrate
rational functions.

Example 7.13. Compute the integral (1+x1 2 )2 dx.
Solution: We use the substitution x = tan y. Then y = tan1 (x), and
so dy/dx = 1/(1 + x2 ), and hence dy = dx/(1 + x2 ). This yields


1
1
dx =
dy
2
2
(1 + x )
1 + x2

1
=
dy
1 + tan2 y

= cos2 y dy
=

y sin 2y
+
+C
2
4

43. TRIGONOMETRIC SUBSTITUTION

43

y sin y cos y
+
+C
2
2
1
1
x
= tan1 (x) + 2
+ C.
2
2 x +1
The last step is justied since
x
tan y
(7.12)
=
= tan y cos2 y = sin y cos y.
2
2
x +1
1 + tan y
=

2
Figure 7.2 illustrates this trigonometric argument.

Example 7.14. Compute the integral x12 1 .
Solution: The denominator reminds us of the trigonometric
identity
2
2
tan y = sec y 1, and so, if y [0, /2), then tan y = sec2 y 1.
Therefore, we use the substitution x = sec y. Then dx/dy = tan y sec y.
Hence, we have


1
1


=
dx
x2 1
sec2 y 1

1
=
dx
tan y

tan y sec y
=
dy
tan y

= sec y dy
= ln| sec y + tan y| + C

= ln|x + x2 1| + C.
2
Figure 7.3 illustrates this trigonometric argument.

Figure 7.2. Some expressions from (7.12).

44

7. METHODS OF INTEGRATION

Figure 7.3. Some expressions occuring in the solution


of Example 7.14.
43.2. Summary of the Most Frequently Used Trigonometric Substitutions.

The three examples that we have seen so far in this section show the
three most frequently used reverse substitutions. That is,

(i) To compute
r2 x2 dx, use the reverse substitution x =
r sin y.
(ii) To compute integrals involving (r2 + x2 ) under a root sign or
in the denominator of a fraction, use the reverse substitution
x = r tan y. 
(iii) To compute
x2 r2 dx, use the reverse substitution x =
r sec y.
Finally, a word of caution. The availability of the method of reverse
substitution does not mean that this method is always the best one to
compute an integral that contains a square root sign. Some of the
following exercises can be solved by another method faster (and, no,
we are not revealing which ones).
43.3. Exercises.

(1) Use the method presented in this section to compute the area
of an ellipse determined by the equation
x2 y 2
+ 2 = 1.
a2
b

2
(2) Compute
1 4x dx.
 x3

(3) Compute
2 +16 dx.
 3x
(4) Compute 
x 25 x2 dx.
(5) Compute  4 36x2 dx.
(6) Compute  1 + x2 dx.
1
(7) Compute 4+x
2 dx.
 x
(8) Compute x5 dx.

(9) Compute x2 1x2 4 dx.

44. INTEGRATING RATIONAL FUNCTIONS

45


2
(10) Compute  x
x 4 dx.
x2 2x dx.
(11) Compute
 x2
(12) Compute 9+x2 dx.

(13) Compute 4x12 25 dx.

1
(14) Compute x2 4x+13
dx.

1
(15) Compute (x2 +4x+5)
2 dx.

(16) Compute
2x x2 dx.
 x2
(17) Compute 6xx2 dx.

(18) Compute  e2x 16 dx.
(19) Compute ex 4 e2x dx.
(20) Find the average value of x12 +9 on the interval [0, 3].
44. Integrating Rational Functions
44.1. Introduction. Recall that a rational function is the ratio of two

polynomials, such as
R(x) =

3x + 5
P (x)
= 2
.
Q(x)
2x + 4x + 9

Integrating rational functions is relatively simple, because most of these


functions can be obtained as sums of even simpler functions. If the
degree of P (x) is at least as large as the degree of Q(x), then we can
divide P (x) by Q(x), getting a polynomial as a quotient, and possibly
a remainder. That is, if the degree of P is at least as large as the
degree of Q, then there exist polynomials P1 (x) and P2 (x) such that
the degree of P2 (x) is less than the degree of Q(x) and
R(x) =

P1 Q( x) + P2 (x)
P (x)
P2 (x)
=
= P1 (x) +
.
Q(x)
Q(x)
Q(x)

As P1 (x) is a polynomial, it is easy to integrate. Therefore, the di2 (x)


culty of integrating R(x) lies in integrating RQ(x)
, which is a rational
function whose denominator is of higher degree than its numerator.
For this reason, in the rest of this section, we focus on integrating
rational functions with that property, that is, when the degree of the
denominator is higher than the degree of the numerator.
Example 7.15. Let R(x) =
using long division, we get

x3 +2x+1
.
x2 x+1

Then dividing P (x) by Q(x)

P (x) = (x + 1)(x2 x + 1) + 2x,

46

7. METHODS OF INTEGRATION

so
P (x)
2x
=x+1+ 2
,
Q(x)
x x+1
2x
.
and integrating Q(x) boils down to integrating x2 x+1
R(x) =

44.2. Breaking Up the Denominator. In order to decide how to break

up a rational function R(x) into the sum of simpler terms, we analyze


the denominator Q(x) of R(x). A theorem in complex analysis, sometimes called the fundamental theorem of algebra, implies that if q(x)
is a polynomial whose coecients are real numbers, then q(x) can be
written as a product of polynomials that are of degree 1 or 2.
This decomposition, or factorization, of Q(x) will determine the
way in which we break up our rational function into the sum of simpler
terms. There are several cases to distinguish, based on the factorization
of Q(x).
44.2.1. Distinct Linear Factors. The easiest case is when Q(x) factors

into the product of polynomials of degree 1, and each of these terms


occurs only once.

1
dx.
Example 7.16. Compute x2 +3x+2
Solution: Note that x2 +3x+2 = (x+1)(x+2). Using that observation,
we are looking for real numbers A and B such that
1
A
B
(7.13)
=
+
2
x + 3x + 2
x+1 x+2
as functions, that is, such that (7.13) holds for all real numbers x.
Multiplying both sides by x2 + 3x + 2, we get
(7.14)

1 = A(x + 2) + B(x + 1).

If (7.14) holds for all real numbers x, it must hold for x = 1 and
x = 2 as well. However, if x = 1, then (7.14) reduces to 1 = A,
and if x = 2, then (7.14) reduces to 1 = B. So we conclude that
A = 1 and B = 1 are the numbers we wanted to nd. It is now easy
to compute the requested integral as follows:



1
1
1
dx =
dx
dx
2
x + 3x + 2
x+1
x+2
= ln(x + 1) ln(x + 2) + C.
2
The above method can always be applied if Q(x) factors into a
product of linear polynomials, each of which occurs only once. In

44. INTEGRATING RATIONAL FUNCTIONS

47

particular, if Q(x) decomposes as a(x a1 )(x a2 ) (x ak ), then


we can decompose R(x) into a sum of the form
A2
Ak
A1
+
+ +
.
x a1 x a2
x ak
After determining the numbers Ai , we can integrate each of the above
k summands.
44.2.2. Repeated Linear Factors. The next case is when Q(x) factors

into linear terms, but some of these terms occur more than once.

2x+7
Example 7.17. Compute (x+1)
2 (x1) dx.
Solution: Just as in the previous case, we decompose the integrand
into a sum of simpler fractions. We are looking for real numbers A, B,
and C such that
2x + 7
A
B
C
=
+
.
+
2
2
(x + 1) (x 1)
x + 1 (x + 1)
x1
Multiplying both sides by the denominator of the left-hand side, we
get
2x + 7 = A(x + 1)(x 1) + B(x 1) + C(x + 1)2 .
Substituting x = 1 in the last displayed equation yields 9 = 4C, so
C = 2.25. Substituting x = 1 yields 5 = 2B, so B = 2.5. Finally,
the coecient of x2 on the left-hand side is 0, while on the right-hand
side, it is A + C. So A + C = 0, yielding A = 2.25.
Now we are in a position to compute the requested integral.


2.25
2.25
2.5
dx +
dx +
dx
2
x+1
(x + 1)
x1



2.25
2.5
2.25
dx +
dx +
dx
=
2
x+1
(x + 1)
x1
2.5
= 2.25 ln(x + 1) +
+ 2.25 ln(x 1).
x+1
2
k
In general, if a term (x+a) occurs in Q(x), then the partial fraction
decomposition of R(x) will contain one term with denominator (x + a)i
for each i {1, 2, . . . , k}. For instance, if Q(x) = (x+2)3 (x+5)2 (x10),
then R(x) will have a partial fraction decomposition of the form


2x + 7
dx =
(x + 1)2 (x 1)

A3
A4
A6
A2
A5
A1
+
+
+
+
+
.
2
3
2
x + 2 (x + 2)
(x + 2)
x + 5 (x + 5)
x 10

48

7. METHODS OF INTEGRATION

44.2.3. Distinct Quadratic Factors. The third case is when the factor-

ization of Q(x) contains some quadratic factors that are irreducible


(i.e., they are not the product of two linear polynomials with real coecients), but none of these irreducible quadratic factors occurs more
than once. In that case, after obtaining the partial fraction decomposition of R(x), we may have to resort to the formulas

1
dx = tan1 x + C
2
x +1
and



1
1
1 x
+ C.
dx
=
tan
x 2 + a2
a
a

4x+2
Example 7.18. Compute the integral x3 +x
2 +x+1 dx.
Solution: It is easy to notice that setting x = 1 turns the denominator to 0; hence, the denominator is divisible by x + 1. Dividing the
denominator by x + 1, we get x2 + 1, so the denominator factors as
(x + 1)(x2 + 1). The factor x2 + 1 is irreducible (it is not divisible by
xb for any real number b, since no real number b satises the equation
b2 + 1 = 0).
Therefore, we are looking for real numbers A, B, and C such that
A
B
Cx
4x + 2
=
+ 2
+ 2
.
(7.15)
3
2
x +x +x+1
x+1 x +1 x +1
The reader is invited to verify that the third summand of the righthand side is necessary; that is, if the summand xCx
2 +1 is removed, then
no pair of real numbers (A, B) will satisfy (7.15).
In order to nd the correct values of A, B, and C, multiply both
sides of (7.15) by (x + 1) (x2 + 1) and rearrange, to get
4x + 2 = (A + C)x2 + (B + C)x + A + B.
The coecient of x2 is 0 on the left-hand side, so it has to be 0 on the
right-hand side. Therefore, A + C = 0. Similarly, the coecient of x is
4 on the left-hand side, so it has to be 4 on the right-hand side, forcing
B + C = 4. Similarly, the constant terms of the two sides have to be
equal, and, consequently, A + B = 2. Solving this system of equations,
we get A = 1, B = 3, and C = 1. Therefore,




1
1
x
4x + 2
dx =
dx + 3
dx +
dx
3
2
2
2
x +x +x+1
x+1
x +1
x +1
1
= ln(x + 1) + 3 tan1 x + ln(x2 + 1).
2
2

44. INTEGRATING RATIONAL FUNCTIONS

49

In general, if x2 +ax+b is a quadratic factor in Q(x), then the partial


E
and
fraction decomposition will contain a summand of the form x2 +ax+b
Fx
a summand of the form x2 +ax+b . Again, the latter is necessary, since
x
will not equal one of the form
a rational fraction of the form x2E+F
+ax+b
E
for any choice of E if F = 0.
x2 +ax+b
44.2.4. Repeated Quadratic Factors. Finally, it can happen that the

factorization Q(x) contains irreducible quadratic factors, some of which


occur more than once.
 3 2 +3x+7
dx.
Example 7.19. Compute the integral x x+2x
4 +2x2 +1
Solution: It is easy to see that the denominator factors as (x2 + 1)2 .
Hence, we are looking for real numbers A, B, C, and D such that
A
Bx
C
Dx
x3 + 2x2 + 3x + 7
= 2
+ 2
+ 2
+ 2
.
4
2
2
x + 2x + 1
x + 1 x + 1 (x + 1)
(x + 1)2
Multiplying both sides by x4 + 2x2 + 1 and rearranging, we get
x3 + 2x2 + 3x + 7 = Bx3 + Ax2 + (B + D)x + (A + C).
For each k, the coecients of xk must be the same on both sides.
Hence, A = 2 and B = 1, so C = 5 and D = 2.
Now we can compute the requested integral using the preceding
partial fraction decomposition as follows:


x3 + 2x2 + 3x + 7
dx
x4 + 2x2 + 1

 
2
2x
x
5
dx
=
+
+
+
x2 + 1 x2 + 1 (x2 + 1)2 (x2 + 1)2
5
1
5x
1
+ tan1 x 2
= 2 tan1 x + ln(x2 + 1) +
2
2
2(x + 1) 2
x +1
1 5x 2
1
.
= ln(x2 + 1) + 4.5 tan1 x + 2
2
2 x +1

1
Here we used the formula for (x2 +1)
2 that we computed in the last
section, in Example 7.13.
2
By now, the reader must know what the general version of the
technique of the preceding example is. If the factorization of Q(x)
contains (x2 + ax + b)k , then, for each integer i such that 1 i k,
the partial fraction decomposition of R(x) will contain a summand of
Ei
Fi x
the form (x2 +ax+b)
i and a summand of the form (x2 +ax+b)i .

50

7. METHODS OF INTEGRATION

44.3. Rationalizing Substitutions. There are situations when a function

that is not a rational function can be turned into one by an appropriate


substitution, and then it can be integrated by the methods presented
in this section. The most frequent scenario in which this happens is
when the integrand contains roots, but if those roots are replaced by
another variable, we get a rational function in that other variable.
 x
dx.
Example 7.20. Compute x+1

1
Solution: We use the substitution x = y. Then dy/dx = 21 x = 2y
.
This leads to



x
y

2y dy
dx =
y+1
x+1

2y 2
=
dy
y+1

2
dy
= 2(y 1) +
y+1
= y 2 2y + 2 ln(y + 1)

= x 2 x + 2 ln( x + 1).
2
Note that the computation would have been very similar if the
integrand
contained some

other root of x. Indeed, if the integrand


contained r x instead of x, then we would have substituted y = r x,
and that would have turned the integrand into a rational function of
y. Indeed, y = x1/r implies
dy
x(1/r)1
=
,
dx
r
dy
y 1r
=
,
dx
r
and therefore
dx = ry r1 dy.
In other words, dx is a equal to dy times a polynomial function of y,
so, indeed, the integrand will be a rational function of y.
44.4. Exercises.

(1) Compute
(2) Compute
(3) Compute
(4) Compute
(5) Compute







5
dx.
x2 +5x+4
x+3
dx.
x2 6x+5
2
x +4x+1
dx.
x3 6x2 +11x6
3
dx.
x3 +x2 x1
2x+7
dx.
x3 x2 x+1

45. STRATEGY OF INTEGRATION

(6) Compute
(7) Compute
(8) Compute
(9) Compute
(10) Compute
(11) Compute
(12) Compute
(13) Compute
(14) Compute
(15) Compute
(16) Compute
(17) Compute
(18) Compute
(19) Compute
















51

1
dx.
x4 +5x2 +4
1
dx.
x4 +4x2 +4
1
dx.
1x4
3x+7
dx.
x4 1
2
x +4
dx.
x3 x
x+3
dx.
x3 +1
1
dx.
x3 1

x+1

dx.
x3

3x

3 x+2 dx.
1
dx.
1+ x
ex
dx.
e2x +1
x
e
dx.
e2x 1
x
e
dx.
e2x 4ex +3
sin x
dx.
cos x+cos2 x
1
dx.
10 sin x+8


(20) Compute
(Hint: Use the substitution u =
2u
tan(x/2) and observe that this implies that sin x = 1+u
2 .)
Note that the substitution described above is called the Weierstrass substitution, named after the German mathematician
Karl Weierstrass. It is a powerful tool for computing the integrals of functions of the type f g, where f is a trigonometric
function and g is a rational function.
45. Strategy of Integration

We presented various integration techniques in this chapter and


in some preceding chapters. The most general ones were integration
by parts and integration by substitution. The most frequently studied
special cases were related to trigonometric functions and their inverses.
Reverse substitution came up in some special cases. We also discussed
the integration of rational functions, using the technique of partial
fractions.
In short, we have learned a decent number of methods. For this
very reason, it is sometimes not obvious which method we should use
when trying to integrate a function. While there is no general rule, in
this section we will provide a few guidelines.
So let f be a function that is not equal to one of the functions
whose integral we either know ohand or have a deterministic method

52

7. METHODS OF INTEGRATION

to compute. That is, f is not a polynomial, f is not a rational function,


f is not the function f (x) = ax or f (x) = loga x for some positive real
number a, and f is not one of the basic trigonometric functions like
sin x or tan x. Let us also assume that simple algebra will not help,
that is, that f cannot be transformed into one of these elementary
functions by simple algebraic transformations. Then how do we decide
which method to use?
45.1. Substitution. The method that needs the least amount of work,

when it is available, is a simple substitution, so it is reasonable to


try to use that method rst. There is a particularly good chance for
this approach to work when f is the composition of two functions, one
of which has a constant derivative, or when f is of the form f (x) =
(x), and so f (x) dx = h(x). In the
h (g(x))g  (x), since then f (x) = dh
dx
language of substitutions, this means that substituting y = g(x) will
work, since




dy



f dx = h (g(x))g (x) dx = h (y) dx = h (y) dy.
dx
In other words, the integral of the composite function f is turned into
something simpler, the integral of the function h.
Example 7.21. Let f (x) = sin2x. Then, using the substitution
y = 2x, we get that sin 2x dx = 12 sin y dy = 12 cos y + C = 12 cos
2x + C.
Example 7.22. Let f (x) = x23x+1 . Then we set y = x2 + 1, so
dy/dx = 2x. This leads to


3x
3x
dx
=
dx
x2 + 1
y

1
3
dy
=
2
y
3
= ln y + C
2
3
= ln(x2 + 1) + C.
2

The reader should compute the integral sinn x cos x dx at this
point.
45.2. Integration by Parts. If f is the product of two functions, but

substitution does not seem to help, then integration by parts is the


logical next step. This technique is particularly useful when one of
the two functions whose product is f is made signicantly simpler by
dierentiation.

45. STRATEGY OF INTEGRATION

Example 7.23. Compute the integral

53

xex dx.

Solution: Considering the integrand, we notice that substitution is


unlikely to help, since x and ex are not closely related. On the other
hand, the integrand is a product, and one of the terms, x, is made
simpler by dierentiation. Therefore, we choose the technique of integration by parts, with x = u and v  = ex . Then u = 1, while
v = ex , and we get


x
x
xe dx = xe ex dx
= xex + ex + C
= (1 x)ex + C.

45.3. Radicals. If the two most general methods (substitution and in-

tegration by parts) are not helpful, then it is quite possible that there is
a root sign in the integrand. In that case, there are two specic methods that we can try, reverse substitution and rationalizing substitution.
The easiest way to know when to use each of these two methods is to
remember the relatively few cases in which reverse substitution
 1works
directly. As we have seen, these are the integrals involving r2 +x2 dx,


r2 x2 dx, and
x2 r2 dx. If the integrand does not contain
any of these functions, then it may be simpler to use
a rationalizing

x
dx. The exsubstitution, such as in computing the integral x+4
ercises at the end of this section will ask the reader to decide which
method to use for a few specic examples.
45.4. If Everything Else Fails. If none of our methods work, then it may

be that an unexpected transformation of the integrand may help, at


least with relating the integral to one that is not quite as challenging
to compute.
 sin4 x
2
Example 7.24. Compute tan
2 x cos 2x dx.
Solution: We use the trigonometric identity tan x = sin x/ cos x, to
rewrite the integrand. We get


sin4 x
2
cos 2x dx = sin2 x cos2 x cos2 2x dx
tan2 x

1 2
sin 2x cos2 2x dx
=
4

1
=
sin2 4x dx,
16

54

7. METHODS OF INTEGRATION

which is easy to integrate with the method we learned for powers of


trigonometric functions.
2
It is important to point out that sometimes there is indeed no solution; that is, there exist elementary functions f such that there is no
elementary function F (x) satisfying F  (x) = f (x). Examples of such
2
functions f include ex , ex /x, and ln1x .
45.5. Exercises.

In all of the exercises below, compute the integral.



(1) x2 e5x dx.

(2) xx2 1 dx.

dx.
(3) xx+1
3
 sin+1x
(4) e
cos x dx.
 ex
(5) ex 1 dx.

(6) x12 +4 dx.
 1
dx.
(7) x+4
 1
(8) x ln x dx.

1
dx.
(9) x ln xln(ln
x)
 1
(10) 1x dx.

x
dx.
(11) 1+sin
x
 ecos
3x
(12) ex +1 dx.

(13) (x + cos x)2 dx.

(14) ex sin 3x dx.

(15) x2 ln x dx.

2
(16) 1x
dx.
 x 1
(17) 4x4x2 dx.
 x
dx.
(18) 5x

(19) tan4 x sec2 x dx.
 1
dx.
(20) 1+sin
x
46. Integration Using Tables and Software Packages
46.1. Tables of Integrals. Tables of integrals can be found in many

calculus textbooks and on the Internet. The website www.integraltable.com is a good example, and we will use it as a reference in this
subsection. (When we say the table of integrals, we mean that table.)

46. INTEGRATION USING TABLES AND SOFTWARE PACKAGES

55

No matter how extensive a table of integrals is, it cannot contain


all integrals. It is therefore important to know how to use these tables
to compute integrals that are not contained in the tables in the same
form.
The easiest case is when the integral to be computed is a special
case of a more general integral that is in the table.

Example 7.25. Use the table of integrals to compute ln(x2 +9) dx.
Solution: Looking at the table of integrals, we nd that the integrand
is a special case of integral (45), with a = 3. Using the formula given
in the table for the general case with a = 3, we get the result




2
2
1 x
2x + C.
ln(x + 9) dx = x ln(x + 9) + 6 tan
3
2
Sometimes, we have to resort to integration by substitution to be
able to use the table of integrals.

9 4x2 dx.
Example 7.26. Use the table of integrals to compute
Solution: Taking a look at the
 table of integrals, we nd that formula
a2 x2 dx. In order to be able to use
(30) provides a formula for
that formula, we set y = 2x, which implies dy/dx = 2. Therefore,


1
9 4x2 dx =
9 y 2 dy.
2
Now we can apply formula (30) from the table of integrals, to get




y
9
y
1
9 y 2 dy =
9 y 2 + tan1
+C
2
4
2
9 y2


x
2x
9
1
2

=
+ C.
9 4x + tan
2
2
9 4x2
2
Sometimes we need to carry out some algebraic manipulation before
we can use the technique of substitutions in connection with using the
table of integrals.
 2
dx.
Example 7.27. Use the table of integrals to compute xx2+2x+1
+2x+10
Solution: There is no integral in the table of integrals that would immediately stand out as one that is very similar to this one. The crucial
observation is that the substitution y = x+1 signicantly
simplies our

2
y
integrand. That substitution leads to the integral 2 2 dy, which,
y +3

56

7. METHODS OF INTEGRATION

in turn, can be directly found in the table of integrals as item (36).


Substituting x back into the obtained formula, we get

x2 + 2x + 1

dx
x2 + 2x + 10

x2 + 2x + 10 9 

=
ln x + 1 + x2 + 2x + 10 + C.
2
2
2
46.2. Software Packages. Computer software packages such as Maple

and Mathematica are very useful tools of integration. These packages


will compute the denite or indenite integrals of a large class of functions, then they will present the results in a form that is usually, but
not always, in the form the user expected. In this section, we show a
few examples of these unexpected results and explain how to interpret
them.
To start with a very basic example, type
int(x^2+3x,x);
into Maple. We get the answer 13 x3 + 32 x2 . This is the correct answer
having constant term 0. Experimenting with other functions, we note
that Maple always answers in this way, that is, without the constant
C at the end. It is important not to forget this if we will be using the
obtained function in some further computation.
Maple does not always provide the simplest form for the integral
that it computes.
For instance, if we ask Maple to compute the indef
inite integral x(x2 + 3)6 dx, we get the output
1 14 3 12 27 10 135 8 405 6 729 4 729 2
x + x + x +
x +
x +
x +
x.
14
2
2
2
2
2
2

However, it is very easy to compute x(x2 + 3)6 dx by hand, using
the substitution u = x2 . That substitution leads to the same solu1
(x2 + 3)7 . If we want
tion, but in a much simpler form, namely, 14
to verify that this result indeed agrees with the one given by Maple
and displayed in (7.16), we can ask Maple to expand the expression
1
(x2 + 3)7 using the expand command. We see that the exH = 14
panded expression indeed agrees with the one given in (7.16), up to
the constant terms at the end.
There are other commands like expand that, are useful if we want
to transform the output of an integration software package. The commands rationalize and simplify are examples of these.

(7.16)

46. INTEGRATION USING TABLES AND SOFTWARE PACKAGES

57

There is often more than one way to express integrals involving


hyperbolic functions.
 1A striking example is the following. If we ask
Maple to compute 1x2 dx by typing
int(1/(1-x^2),x);
Maple returns the answer tanh1 x. This is very surprising, since it is
not dicult to integrate the integrand as a rational function, with no
hyperbolic functions involved. Indeed,
0.5
0.5
1
=

,
2
1x
x+1 x1
and so


x+1
1
1
1
dx = ln(x + 1) ln(x 1) = ln
(7.17)
.
2
1x
2
2
x1
The result obtained by Maple actually agrees with the result given
in (7.17), even if that is not obvious. Indeed, if y = tanh1 x, then,
by denition, x = (ey ey )/(ey + ey ). Solving this equation
for

y is not completely trivial, but at the end, it yields y = ln x+1


.
x1
y
(Hint: Multiply both the numerator and the denominator by e to get
x = (e2y 1)/(e2y + 1).)
Finally, when computing denite integrals, Maple sometimes answers by using the acronyms
of some rare functions. For instance, if
1
2
we want to compute 0 sin (x ) dx by typing
int(exp(sin(x^2)),x=0..1).
then we get the answer
(1/2)*FresnelS(sqrt(2)/sqrt(Pi))*sqrt(2)*sqrt(Pi).
Here FresnelS refers to the Fresnel sine integral, a concept beyond
the scope of this book. If we simply want to know a numerical value
2
1
for 0 ex dx, we can type
evalf(int(sin(x^2),x=0..1))
instead. Maple outputs 0.4596976941 as the answer.
46.3. Exercises.


(1) Use the table of integrals to compute  x ln(3x
 + 5) dx.
(2) Use the table of integrals to compute  x cos 2 x dx.
(3) Use the table of integrals to compute x+x+7
dx.
4x+5
 x2
(4) Use the table of integrals to compute x6 +16 dx.

3
(5) Use the table of integrals to compute (4x8x2 +9) dx.

(6) Use the table of integrals to compute
8x2 + 3 dx.

58

7. METHODS OF INTEGRATION


1
(7) The table of integrals provides a formula for ax2 +bx+c
dx.
Describe the values of a, b, and c for which that formula will
not work and compute the integral in those exceptional cases.
(8) The table of integrals provides a formula for

ln(ax2 + bx + c) dx.

(9)

(10)

(11)

(12)

(13)
(14)

(15)

(16)

(17)

(18)

Describe the values of a, b, and c for which that formula will


not work and compute the integral in those exceptional
 cases.
favorite
package to compute xex dx,
 3 software
Use2 your
x
x
x e dx, and x e dx. Do you see a pattern? Try to guess
what x4 ex dx is, then verify your guess by using your software package again.

Use
your
favorite
software
package
to
compute
ln x dx,


x ln x dx, and x2 ln x dx. Do you see a pattern? Try to
guess what x3 ln x dx is, then verify your guess by using your
software package again.

favorite software package to compute ln x dx,
Use your
(ln x)2 dx, and (ln x)3 dx. Do you see a pattern? Try to
guess what (ln x)4 dx is, then verify your guess by using your
software package again.

1
1
and
let
g(x)
=
. Clearly, f (x) = g(x)
Let f (x) = 1+x
2
1+x2
for all x for which these functions are dened. Compute the
integral of both functions with Maple. If the results seem
dierent, show
that they are in fact equal.

2
Compute x(x + 1)8 dx rst by Maple, then by substitution.
Which answer
 is simpler?
Compute x x2 + 4x + 29 dx using a software package. If
the result appears to be dierent from what you nd in the
table of integrals,
explain why the two results are equivalent.

Compute tan2 x sec4 x dx. If the result appears to be dierent from what you get using the methods of Section 42, explain
why the two
are equivalent.
 results
2
Compute sin x cos4 x dx. If the result appears to be dierent
from what you get using the methods of Section 42, explain
why the two results are equivalent. 
1
Use a software package to compute 0 sinn x dx for n = 3,
n = 4, and n = 5. (The case of n = 2 was discussed in the
text.) Explain the trend you detect. 
1
Use a software package to compute 0 cosn x dx for n = 2,
n = 3, n = 4, and n = 5. Explain the trend you detect.

47. APPROXIMATE INTEGRATION

59

(19) Use a software package to compute the value of the expression


 106 x2 /2
1
e
dx.
106
(20) Use a software package to compute the value of the expression
 6 x4
1 10
dx.
0
x6 +1
47. Approximate Integration
Sometimes
it is not possible to nd the exact value of a denite
b
integral a f (x) dx. It could happen that we cannot nd the antiderivative of f (x) or that the antiderivative of f (x) is not an elementary
function. Or it could happen that f itself is not given by a formula,
but instead, f is given by its graph, which is plotted by a computer
program. In this case, we resort to methods of approximate integration.
47.1. Basic Approximation Methods. The key observation behind the

approximation methods is the fact that if f (x) 0 for x [a, b], then
b
f (x) dx is equal to the area below the graph of the function f on
a
b
that interval. More precisely, a f (x) dx is equal to the area of the
domain bordered by the horizontal axis, the vertical lines x = a and
x = b, and the graph of f .
In order to estimate the area of this domain D, we cut D into small
vertical strips. To do so, we choose real numbers
a = x0 < x1 < < xn = b
and then consider the vertical lines x = xi for all i. There are several
ways in which we can estimate the area of each strip. Let Si be the
strip bordered by the lines x = xi and x = xi+1 .
We could say that the area of Si is roughly equal to the area of
the rectangle whose base is the interval [xi1 , xi ] and whose height is
f (xi1 ). This is called the left-endpoint approximation. Or we could
say that the area of Si is roughly equal to the area of the rectangle
whose base is the interval [xi1 , xi ] and whose height is f (xi ). This is
called the right-endpoint approximation. Or we could use f (
xi ), the
value that f takes at the midpoint xi of the interval [xi1 , xi ]. This is
called the midpoint approximation. Summing over the allowed values
of i (i.e., i ranges from 1 to n), we see that these three methods provide
b
the following three estimates for a f (x) dx.
(i) The left-endpoint approximation shows
 b
n

(7.18)
f (x) dx
(xi xi1 )f (xi1 ).
a

i=1

60

7. METHODS OF INTEGRATION

(ii) The right-endpoint approximation shows


 b
n

f (x) dx
(xi xi1 )f (xi ).
a

i=1

(iii) The midpoint approximation shows


 b
n

f (x) dx
(xi xi1 )f (
xi ).
a

i=1

As the above formulas suggest, it will be particularly easy to work


with these formulas if the points x1 , x2 , . . . , xn1 split the interval [a, b]
into equal parts, since in that case xi xi1 = (b a)/n for all i.
1 2
Example 7.28. Find the approximate value of 0 ex dx.
Solution: Let us use the left-endpoint method with n = 4 and x1 , x2 ,
and x3 splitting [0, 1] into four equal parts. That means that x1 = 1/4,
x2 = 1/2, and x3 = 3/4. Then (7.18) implies
 1

1 0
2
ex dx
e + e1/16 + e1/4 + e9/16 1.2759,
4
0
since xi xi1 = 1/4 for all i.

Figure 7.4. Left-endpoint method.

47. APPROXIMATE INTEGRATION

61

Figure 7.5. Right-endpoint method.


If we use the right-endpoint method, with the same set of points
xi , we get
 1

1  1/16
2
ex dx
+ e1/4 + e9/16 + e 1.7055.
e
4
0
It is not surprising that the second method yields the larger result,
2
since the integrand ex is an increasing function, so f (xi ) > f (xi1 ).
Furthermore, for each point x [xi1 , xi ], we have f (xi1 ) f (x)
f (xi ). So the left-endpoint method underestimates the area of each
strip Si , while the right-endpoint method overestimates
 1 2 it. Therefore,
the area of Dand hence the correct value of 0 ex dxis between
the two values of 1.2759 and 1.7055 computed above.
2
Replacing the value of n = 4 by some larger number will result in
a more precise approximation (and more work). Using the midpoint
method will result in an approximation A that is closer to the actual
2
1
value of the integral 0 ex dx, but it is not completely obvious from
1 2
1 2
which side A approximates 0 ex dx, that is, whether A < 0 ex dx or
1 2
A > 0 ex dx.
47.2. More Advanced Approximation Methods.
47.2.1. Trapezoid Method. If the dierence between f (xi1 ) and f (xi )

is large, then estimating the area of Si by using rectangles could lead

62

7. METHODS OF INTEGRATION

Figure 7.6. Trapezoid method.


to large errors. A more rened approach is to estimate the area of Si
by computing the area of the trapezoid whose vertices are the points
(xi1 , 0), (xi , 0), f (xi ), and f (xi1 ). We know that the area of this
trapezoid is the average length of its parallel sides times the distance
))(xi xi1 )
.
of those parallel sides from each other, that is, (f (xi )+f (xi1
2
Summing over all possible
values of i, we get an estimate for all the
b
area of D, that is, for a f (x) dx. Indeed, we obtain the formula


f (x) dx =
a

n

(f (xi ) + f (xi1 ))(xi xi1 )
i=1

In particular, if the xi are chosen so that they split the interval [a, b]
into n equal parts, then the last displayed equation simplies to


ba
(f (xi ) + f (xi1 ))
2n i=1
n

f (x) dx =
a

ba
(f (a) + 2f (x1 ) + 2f (x2 ) + + 2f (xn1 ) + f (b)) .
2n

Note that f (x0 ) = f (a) and f (xn ) = f (b) occur only once in the
sum in the last line since a and b are each part of only one of the
intervals [xi1 , xi ].
Example 7.29. Use
trapezoid method with n = 4 to nd the
 1 the
x2
approximate value of 0 e dx.

47. APPROXIMATE INTEGRATION

63

Figure 7.7. Trapezoid method.


Solution: We will have x1 = 1/4, x2 = 1/2, and x3 = 3/4, just as in
Example 7.28. This yields
 1

1 0
2
ex dx =
e + 2e1/16 + 2e1/4 + 2e9/16 + e
8
0
= 1.4907.
2
The alert reader may have noticed that the result we obtained is
precisely the average of the left-endpoint and right-endpoint approximations we obtained for the same integral in the previous section.
(This means that it is a better approximation than at least one of the
two earlier ones.) This is not an accident, and in Exercise 47.4.17,
the reader will be asked to prove that, under certain conditions, this
phenomenon will always occur.
47.2.2. Simpsons Method. A similar method is Simpsons method, in

which we use parabolas instead of straight lines for approximation. For


simplicity, let us now assume that the points xi split the interval [a, b]
into n equal parts, that is, xi xi1 = (b a)/n. In order to simplify
the notation, let us set yi = f (xi ).
For any integer i [1, n1], consider the points (xi1 , yi1 ), (xi , yi ),
and (xi+1 , yi+1 ). There is exactly one parabola pi of the form y =
Ax2 + Bx + C that contains these three points. It can then be proved

64

7. METHODS OF INTEGRATION

that the area under that parabolamore precisely, the area of the
domain Pi bordered by the horizontal axis, the vertical lines x = xi1
and x = xi+1 and pi is equal to
(7.19)

ba
(yi1 + 4yi + yi+1 ) .
3n

If we summed (7.19) over all possible values of i, we would not get a


good estimate, since most points of the domain under the curve would
be part of two of the Pi . For instance, a point with a horizontal coordinate between xi and xi+1 is part of both Pi and Pi+1 . Therefore,
we sum the last displayed equation over all even values of i, and, accordingly, we stipulate that n be an even number. This leads to the
following estimate.
Theorem 7.1 (Simpsons Method). Let n be an even positive integer. Then


f (x) dx
a

ba
(y0 + 4y1 + 2y2 + 4y3 + + 2yn2 + 4yn1 + yn ) .
3n

7.30. Use Simpsons method with n = 4 to approximate


 1 Example
2
sin (x ) dx.
0

Figure 7.8. Simpsons method.

47. APPROXIMATE INTEGRATION

65

Solution: We have y0 = 0, y1 = sin(1/16), y2 = sin(1/4), y3 =


sin(9/16), and y4 = sin 1. So Simpsons method yields
 1

1
sin x2 dx
(4 sin(1/16) + 2 sin(1/4) + 4 sin(9/16) + sin 1)
12
0
0.31.
2
Note that this result conrms our intuition in that if x [0, 1], then
sin (x2 ) sin x, and so
 1
 1
 2
sin x dx
sin x = 1 cos 1 0.4597.
0

47.3. Bounds on the Error Term. The error term E of an approximation

is the dierence between the number obtained by the approximation


and the actual value of the quantity that was approximated. (In this
b
section, that actual value is the value of a f (x) dx.) It goes without
saying that the smaller the absolute value of the error term, the better
the approximation is.
The eld of numerical analysis studies the error terms of approximation methods. The techniques of that eld yield various bounds
on error terms. We collected some of these bounds in the following
theorem.
Theorem 7.2. Let f be a twice-dierentiable function on [a, b] such
that |f  (x)| M if x [a, b]. Then the following hold for the approxib
mation methods used to compute a f (x).
(a) If ET is the error term of the trapezoid method, then
M (b a)3
.
12n2
is the error term of the midpoint method, then
|ET |

(b) If EM

M (b a)3
.
24n2
(c) If, for all x [a, b], the number f (4) (x) is dened and is at
most as large as the constant K, and ES is the error term of
Simpsons method, then
K(b a)5
.
|ES |
180n4
Comparing the formulas of parts (a) and (b) of the previous theorem, we can conclude that the worst-case scenario of the midpoint
method is better than the worst-case scenario of the trapezoid method.
|EM |

66

7. METHODS OF INTEGRATION

This, of course, does not mean that the midpoint method is always
better than the trapezoid method.
Example 7.31. Find an upper bound for the approximation obtained in Example 7.29.
2

Solution: We apply part (a) of Theorem 7.2. We have f (x) = ex ,


2
so f  (x) = ex (4x2 + 2). This is an increasing function on [0, 1], so its
maximum is taken at x = 1, showing that |f  (x)| 6e. As in Example
7.29, we chose n = 4, so the previous theorem yields
|ET |

M (b a)3
6e
=
0.085.
12n2
12 16
2

47.4. Exercises.

(1) Use n = 4 and the midpoint method to nd the approximate


2
1
value of 0 ex dx.
(2) Use n = 4 and the left-endpoint method to nd the approxi3
1
mate value of 0 ex dx.
(3) Use n = 4 and the right-endpoint method to nd the approx4
1
imate value of 0 ex dx.
(4) Use n = 6 and
themidpoint method to nd the approximate
1
value of 0 1 + x dx.
(5) Use n = 4 and the trapezoid method to nd the approximate
2
1
value of 0 ex dx.
(6) Use n = 4 and the trapezoid method to nd the approximate
1
value of 0 sin(ex ) dx.
(7) Use n = 4 and the trapezoid method to nd the approximate
2
value of 1 sinx x dx.
(8) Use n = 4 and the trapezoid method to nd the approximate
1 1
value of 0 1+x
5 dx.
(9) Use n = 4 and Simpsons method to nd the approximate
1
value of 0 sin x3 dx.
(10) Use n = 4 and Simpsons method to nd the approximate
2 ln x
value of 1 1+x
2 dx.
(11) Use n = 4 and Simpsons method to nd the approximate
1
value of 0 esin x dx.
(12) Use n = 4 and Simpsons method to nd the approximate
1 x
value of 0 ee dx.

48. IMPROPER INTEGRALS

67

(13) Use your


approximation method with n = 4 to com 1 favorite

pute 0 sin x dx. Then compute the same integral by nding

the antiderivative of sin x and compare the two results you


obtain.
(14) Use your
 1 favorite approximation method with n = 4 to compute 0 e x dx. Then
compute the same integral by nding the

x
antiderivative of e and compare the two results you obtain.
(15) Use your
approximation method with n = 4 to com 1 favorite
2
pute 0 ln(x + 9) dx. Then compute the same integral by
nding the antiderivative of ln(x2 + 9) and compare the two
results you obtain.
(16) What value of n should be used in each of exercises 1, 5, and
9 to get an error term that is less than 106 ?
(17) Let us assume that the points x1 , x2 , . . . , xn1 used for approximate integration split the interval [a, b] into n equal segments.
Prove that the result obtained by the trapezoid method will be
the average of the left-endpoint method and the right-endpoint
method.
(18) How large is the error term of the approximation in exercise 1
in the worst case?
(19) How large is the error term of the approximation in exercise 7
in the worst case?
(20) Find an upper bound for the error term of the approximation
shown in the solution of Example 7.30.
48. Improper Integrals
In our studies of integration, we have not dealt with denite integrals over innite intervals, nor did we integrate functions over an
interval if the function was not dened in every point of that interval.
In this section, we will consider denite integrals of these kinds, which
are called improper integrals.
48.1. Infinite Intervals. For nite intervals, we have identied

b

f (x) dx
with the area of the domain limited by the graph of f , the vertical lines
x = a and x = b, and the horizontal axis. We learned that, by the funb
damental theorem of calculus, the equality a f (x) dx = F (b) F (a)
holds, where F is an antiderivative of f .
Now let us consider the integral of f over the innite interval [a, ).
b
Recalling
that a f (x) dx is equal to a certain area, we intuitively want

f (x) dx to equal the area of the domain bordered by the line x = a,
a
a

68

7. METHODS OF INTEGRATION

Figure 7.9. Area under the curve y = f (x) from x = a


to x = b.

the horizontal axis, and the graph of f . Note that this area may be a
nite number, even if it is not squeezed between two vertical lines. One
example of this is when f (x) = 0 if x >
 N for some real number N .
Time has come to formally dene a f (x) dx.
b
Definition 7.1. Let f be a function. If the integral a f (x) dx
b
exists for all b > a and limb a f (x) dx
 = L exists as a (nite) real
f (x) dx is convergent, and
number, then
we
say
that
the
integral
a

we write a f (x) dx = L.
b
f (x) dx does not exist or is innite, then we say that
If
lim
b
a

f (x) dx is divergent.
a
Note that if F is an antiderivative of f , then

b

f (x) dx = lim (F (b) F (a))

lim

= ( lim F (b)) F (a).


b


Therefore, the integral a f (x) dx is convergent if and only if
limb F (b) exists and is nite.
Example 7.32. Let f (x) = x2 . Compute


1

f (x) dx.

48. IMPROPER INTEGRALS

69

Figure 7.10. Area under the curve y = f (x) from 1 to .


Solution: We have


x
1

x2 dx
1
b

= lim x1 

dx = lim

1
= lim + 1
b
b
= 1.

In particular, 1 f (x) dx is convergent.
2
Encouraged by the simple solution of the last example, we are going

to compute the more general integral 1 xr for any real number r.



Example 7.33. Let f (x) = xr . Compute 1 f (x) dx.
Solution: Let us rst assume that r =
1. Then we have
 b

r
x dx = lim
xr dx
b
1
1

 b
1 r+1 
= lim
x
.
b

r+1
1

If r > 1, then r + 1 > 0 and limxxr+1 = , so the limit in the

last displayed row is innite, and hence 1 xr dx is divergent.


If r < 1, then r + 1 < 0 and limx xr+1 = 0, so the limit in the

1
last displayed row is equal to r+1
, and hence 1 xr dx is convergent.

70

7. METHODS OF INTEGRATION


If r = 1, then we need to compute 1 xr dx dierently, since, in

a+1
that case, xa dx = xa+1 . Instead, we have


x1 dx
1
b

= lim ln x

dx = lim

= lim ln b = .
b

Therefore, 1 x1 dx is divergent.
2
Note that the results of the previous example prove the following
important theorem.
Theorem 7.3. Let r be a real number.

(i) If r 1, then 1 xr dx is divergent.

(ii) If r < 1, then 1 xr dx is convergent.


The following denition is not very surprising. It is the counterpart
of Denition 7.1.
Definition 7.2. Let f be a function and let b be a real number
b
such that, for all real numbers a < b, the integral a f (x) dx exists. If
L = lima f (x) dx exists as a (nite) real number, then we say that
b
b
the integral f (x) dx is convergent, and we write f (x) dx = L.
f (x) dx is innite or if it does not exist, then we say that
If lim
 b a
f (x) dx is divergent.

The following denition makes it clear how and when we can dene
an integral on the entire line of real numbers.
Definition 7.3. Let f be a function and let m be a real number
m

such that both f (x) dx and m f (x) dx are convergent. Then we



say that the integral f (x) dx is convergent and that
 m


f (x) dx =
f (x) dx +
f (x) dx.

Otherwise, we say that

f (x) dx is divergent.

See Figure 7.11 for an illustration.



Example 7.34. Compute ex dx.

48. IMPROPER INTEGRALS

Figure 7.11.

m

f (x) dx is blue, while

Figure 7.12.

0


m

71

f (x) dx is orange.

ex dx.

Solution:
We set m = 0 and apply
We get that
 0 Denition
7.3.
 x
x
x
e dx is convergent if both of e dx and 0 e dx are con
vergent. However,
 0
0
x
x 
e dx = lim e  = 1 +

is divergent and therefore so is ex .


2
 0 x
Figure 7.12 shows the domain whose area is equal to e dx.
The reader could ask how we knew that we needed to select 0, and
not some other real number, for the role of m, that is, to split the
real number line into two parts. The answer is that we did not, and
other choices of m would have given the same result since the integrand

72

7. METHODS OF INTEGRATION

converges to innity as x goes to negative innity. We chose m = 0


because it was convenient to do so.
Note that all improper integrals discussed in this section are called
Type 1 improper integrals.
48.2. Vertical Asymptotes. Sometimes we may want to compute the

integral of a function f on a nite interval [a, b] so that in some point


c [a, b], the function f has a vertical asymptote. An example is the
function f (x) = 1/(x2 4) on the interval [1, 3]. In this case, we use
b
the technique of limits to formally dene a f (x) dx, as we did in the
previous section.
Definition 7.4. Let f be a function that is continuous on [a, b],
except for one point c [a, b]. Then we set


(7.20)
a

f (x) dx = lim

f (x) dx

tc

and

(7.21)
c

f (x) dx = lim+
tc

f (x) dx.
t

Furthermore, if both of the two limits displayed above exist and are
nite, we set


f (x) dx =
a

f (x) dx +
a

f (x) dx.
c

Note that if the only point c in which f is not continuous is one


of the endpoints of [a, b], then we only have to compute one of (7.20)
and (7.21), since the other integral is taken over a trivial interval and
is hence zero.
Example 7.35. Compute

1
0

x1/2 dx.

Solution: As the only point in [0, 1] in which f (x) = x1/2 is not


continuous is 0, we use formula (7.21) with c = 0 and b = 1.

48. IMPROPER INTEGRALS

Figure 7.13.
We get

1/2

1
0

73

x1/2 dx.

x1/2 dx
t
1


= lim+ 2x1/2 

t0

dx = lim+
t0

t
1/2

= 2 lim+ t
t0

=20
= 2.

1

x1/2 dx is convergent.
4
Example 7.36. Compute 1 x2 dx.

So the integral

Solution: We apply Denition 7.4 since the interval [1, 4] has one
point, c = 0, where the integrand is not continuous. Therefore,
 0
 4
 4
2
2
x dx =
x dx +
x2 dx
1

= lim
t0

0
t

x
1

dx + lim+
t0

x2 dx

t
4




= lim x1  + lim+ x1 


t0
t0

=+
= .

74

7. METHODS OF INTEGRATION

Figure 7.14.

4
1

x2 dx.

So the integral in question is divergent.


2
4
Figure 7.14 shows the domain whose area is equal to 0 x2 dx and
the correct way of breaking that interval up to two parts.
Note that we would have reached the wrong conclusion if we had
disregarded the fact that x2 is not continuous at x = 0 and tried to
apply the fundamental theorem of calculus. Indeed, in that case, we
4
would have obtained the wrong result: x1  = 1
1 = 54 . This
4
1

result is incorrect, and the incorrect step was to apply the fundamental
theorem of calculus for a function that is not continuous in the entire
interval of integration.
The integrals that we have discussed in this section are called Type
2 improper integrals.
48.3. Further Remarks.
48.3.1. Improper Integrals of Mixed Type. There are some integrals that

are improper for two reasons. They are taken over an innite interval,
and that interval contains a point in which the function is not continuous. In that case, we split up the interval of integration so that now
we have two integrals, one of which is of Type 1 and the other of which
is Type 2.
 1
Example 7.37. Compute 0 (x2)
2 dx.
Solution: We break up the interval [0, ) to the union of the two
intervals [0, 2] and [2, ), getting
 2


1
1
1
dx
=
dx
+
dx.
2
(x 2)2
(x 2)2
0
0 (x 2)
2

48. IMPROPER INTEGRALS

Figure 7.15.

Figure 7.16.

75

1
dx.
(x2)2

1
dx.
x2 ln x

The rst term on the right-hand side is an improper integral of Type


2, and the second term on the right-hand side is an improper integral
of Type 1. We can compute both by the methods presented earlier in
this section.
2
48.3.2. Comparison Test. Comparison tests for improper integrals work

very similarly to those for proper integrals.


Theorem 7.4. Let us assume that, for all x a, the chain of
inequalities 0 f (x) g(x) holds.


(i) If a f (x) dx is divergent, then so is a g(x) dx.

(ii) If a g(x) dx is convergent, then so is a f (x) dx.



Example 7.38. Show that 3 x2 1ln x dx is convergent.

76

7. METHODS OF INTEGRATION

Solution: If x 3, then ln x > 1, so x2 ln x > x2 , and therefore


7.16 for an illustration. On
the integrand is less than x12 . See
 Figure
2
the other hand, we know that 3 1/x dx is convergent, so our claim
follows from the comparison test.
2
48.4. Exercises.

(1)
(2)
(3)
(4)
(5)
(6)
(7)
(8)
(9)
(10)
(11)
(12)
(13)
(14)
(15)
(16)
(17)
(18)

(19)
(20)


Is 1 sin x dx convergent or divergent?

Is 0.5 x1.5 dx convergent?

Is x2 dx convergent?

Is 0 xex dx convergent?

2
Is xex dx convergent?

Is 0 ex1+x dx convergent?

Is 2 x ln1 x dx convergent?
4
1
Is 0 x2 8x+7
dx convergent?
 ax
dx, where a is a xed nonnegative real numCompute 0 e
ber.
3
Compute 0 ln x dx.
3
Compute 0 (ln x)2 dx.
3 x
dx.
Compute 0 ln
 1 x1
Compute 3 x+3 dx.

Compute 0 x21+1 dx.
 x
Compute 0 e2xe +9 dx.
1 1
Compute 0 1x
2 dx.

is convergent?
Is there a real number a such that 0 xa dx

Prove that if f is an even function, and 0 f(x) dx is con

vergent, then f (x) dx is convergent and f (x) dx =



2 0 f (x) dx.
State and solve the analogous version of the previous exercise
for the case of odd functions f .
Show an example of a function f for which
 R

f (x) dx = lim
f (x) dx.

CHAPTER 8

Sequences and Series


49. Infinite Sequences
A sequence can be thought of as an ordered list of numbers a1 , a2 ,
..., an , an+1 , .... The subscript n indicates the position of a number an in
the sequence; for example, a1 is the rst element, an is the nth element,
and so on.
Definition 8.1 (Sequence). A sequence is a function f dened on
the set of all positive integers; that is, it is a rule that assigns a number
to each positive integer. If f (n) = an for n = 1, 2, ..., it is customary
to denote the range of f by the symbol {an } or {an }
1 .
So a sequence can be dened by specifying the rule an = f (n) to
calculate the nth term from an integer n. For example,
 n   1 2 3 
n

, , , ... ,
=
an =
n+1
n+1 1
2 3 4
 (1)n  
(1)n
1
1 1 
an =
= 1, , , , ... ,

n
n
2
3 4
1
2
3
an = q n {q n }
=
{1,
q,
q
,
q
,
...}.
0
Sequences can also be dened recursively, that is, by a relation that allows us to nd an if am , m < n, are known. For example, the Fibonacci
sequence {fn } is dened by the recurrence relation
f1 = f2 = 1 , fn = fn1 +fn2 , n 3 {fn } = {1, 1, 2, 3, 5, 8, 13, ...}
Graphic representation of sequences. A sequence can be pictured sim-

ilarly to the graph of a function by plotting points (x, y) = (n, an ),


n = 1, 2, ..., on the xy plane. For example, the sequence an = n/(n + 1)
is the set of points on the graph y = x/(x + 1) corresponding to all
positive integer values of x, that is, x = 1, 2, ....
49.1. Limit of a Sequence. The sequence an = n/(n + 1) has the prop-

erty that the values an approach 1 as n becomes larger. Indeed, the


dierence
n
1
1 an = 1
=
n+1
n+1
77

78

8. SEQUENCES AND SERIES

Figure 8.1. Set of points on the graph y = x/(x +


1) corresponding to integer values x = n. For large x,
x/(x+1) approaches 1 from below, and hence n/(n+1) =
1/(1+1/n) 1 as n . The dierence 1n/(n+1) =
1/(n + 1) can be made smaller than any (small) number
> 0 for all n > N and some integer N .
decreases with increasing n and hence can be made smaller than any
preassigned positive number for all n > N , where N depends on .
For example, put = 102 . Then the condition 1 an < implies that
1/(n + 1) < or 1/ 1 < n or 99 < n. Thus, 1 an < 102 for all
n > 99. If = 104 , then 1 an < 104 for all n > N = 9999. In
other words, no matter how small is, there is only a nite number of
elements of the sequence that lie outside the interval (1 , 1 + ). In
this case, the sequence is said to converge to the limit value 1.
Definition 8.2 (Limit of a Sequence). A sequence {an } has the
limit a if, for every > 0, there is a corresponding integer N such that
|an a| < for all n > N . In this case, the sequence is said to be
convergent, and one writes
lim an = a or an a as n .

If a sequence has no limit, it is called divergent.


One can say that a sequence {an } converges to a number a if and
only if every open interval containing a has all but nitely many of the
elements of {an }.
Theorem 8.1 (Uniqueness of the Limit). The limit of a convergent
sequence is unique:
lim an = a and

lim an = a

a = a .

49. INFINITE SEQUENCES

79

Figure 8.2. Denition of the limit of a sequence. The


dots indicate numerical values an (vertical axis). The
integer n increases from left to right (horizontal axis).
The convergence of an to a number a means that, for
any small > 0, there is an integer N such that all the
numbers an , n > N , lie in the interval (a , a + ).
It is clear that N depends on . Generally, a smaller
requires a larger N .
Proof. Fix > 0. Then, by the denition of the limit, there are
numbers N and N  such that |an a| if n > N and |an a | if
n > N  . Hence, both inequalities hold for n > max(N, N  ) and for all
such n:
0 |a a | = |a an + an a | |an a| + |an a | < 2 .
This inequality shows that the nonnegative number |a a | is smaller
than any preassigned positive number, which means that |a a | = 0
2
or a = a .
Since a sequence is a function dened on all positive integers, there
is a great deal of similarity between the asymptotic behavior of a function f (x) as x and a sequence an = f (n).
Theorem 8.2 (Limits of Sequences and Functions). Let f be a
function on (0, ). Suppose that limx f (x) = a. If an = f (n),
where n is an integer, then limn an = a.
The validity of the theorem follows immediately from the denition
of the limit limx f (x) = a (i.e., given > 0, there is a corresponding
number M such that |f (x) a| < for all x > M ) by noting that the
range of f (x) contains the sequence an = f (n).

80

8. SEQUENCES AND SERIES

Example 8.1. Find the limit of the sequence an = ln n/n if it exists


or show that the sequence is divergent.
Solution: Consider the function f (x) = ln x/x so that an = f (n) for
all positive integers. Hence,
ln n
ln x
1/x
= lim
= lim
= 0,
n n
x x
x 1
arising from ln x/x as x has
where the indeterminate form

been resolved by means of lHospitals rule. Note that lHospitals rule


applies not to sequences but to functions of a real variable.
2
Following the analogy between the limits of sequences and functions, one can select a particular class of divergent sequences.
lim

Definition 8.3 (Innite Limits). The limit limn an = means


that, for every positive number M , there is a corresponding integer N
such that an > M for all n > N . Similarly, the limit limn an =
means that, for every negative number M , there is a corresponding
integer N such that an < M for all n > N .
Example 8.2. Analyze the convergence of the power sequence an =
1/np , where p is real.
Solution: Put f (x) = 1/xp for x > 0. Then an = f (n) and therefore

0 if p > 0,
1
1
1 if p = 0,
lim p = lim p =
n n
x x
if p < 0.
2
Example 8.3. Analyze the convergence of the sequence an = q n ,
n = 0, 1, ..., where q is real.
Solution: Suppose q > 0. Put f (x) = q x = ex ln q . From the properties
of the exponential function, it follows that eax if a = ln q > 0,
eax = 1 if a = ln q = 0, and eax 0 if a = ln q < 0. Therefore,
an if q > 1, an = 1 1 if q = 1, and an 0 if 0 < q < 1. When
q = 0, an = 0. Suppose q < 0. Then q = |q| and an = (1)n |q|n =
(1)n en ln|q| . If |q| < 1, then even and odd terms of the sequence
converge to 0: a2n = e2n ln|q| 0 and a2n1 = e(2n1) ln|q| 0 as
n . When q = 1, the sequence an = (1)n is divergent because
a2n = 1 and a2n1 = 1; that is, the sequence oscillates between 1
and 1 for all n and an does not approach any number. Finally, if
q < 1, the sequence is divergent, too, because a2n = e2n ln|q| but
a2n1 = e(2n1) ln|q| . Moreover, it approaches neither nor

49. INFINITE SEQUENCES

81

as it oscillates taking ever-increasing positive and negative values.


Thus,

0 if q (1, 1),
1 if q = 1,
lim q n =
n

if q > 1,
and the sequence does not converge if q 1.

Example 8.4. Find the limit of the sequence an = (1 + q/n)1/n .


Solution: Let f (x) = (1 + q/x)x for x > |q| to make 1 + q/x positive
for any q. Then

q x
= lim+ (1 + qu)1/u
lim an = lim 1 +
n
x
u0
x
where the substitution x = 1/u has been made. To nd the latter limit,
consider ln f (x) = ln(1 + qu)1/u = ln(1 + qu)/u. Using lHospilals rule,
lim+

u0

ln(1 + qu)
q/(1 + qu)
q
= lim+
= lim+
= q.
u0
u0 1 + qu
u
1

It follows from ln f (x) q that f (x) eq as x and, hence,

q n
= eq
lim 1 +
n
n
for all real q.

49.2. Subsequences. Given a sequence {an }, consider a sequence {nk }

of positive integers such that n1 < n2 < n3 < . Then the sequence
{ank }, k = 1, 2, ..., is called a subsequence of {an }. Recall that a sequence {an } converges to a number a if and only if every open interval
containing a has all but nitely many of the elements of {an }. Therefore, the following theorem holds.
Theorem 8.3. A sequence {an } converges to a if and only if every
subsequence of {an } converges to a.
This necessary and sucient criterion for convergence has already
been used in Example 8.3. The sequence an = (1)n does not converge because it has two subsequences a2n = 1 and a2n1 = 1, which
converge to dierent numbers, 1 = 1.
49.3. Limit Laws for Sequences. The limit laws for functions also hold

for sequences, and their proofs are similar. If {an } and {bn } converge

82

8. SEQUENCES AND SERIES

to numbers a and b, respectively, and c is a constant, then


lim (an + bn ) = lim an + lim bn = a + b,

lim (can ) = c lim an = ca,

lim (an bn ) = lim an lim bn = ab,

limn an
a
an
=
=
if b = 0,
n bn
limn bn
b
lim (an )p = ( lim an )p = ap if p > 0 and an > 0.
lim

It should be emphasized that the convergence of the sequences an and


bn is crucial for these relations to hold. For example, let bn = n and
an = q n where q is a number. Then both the sequences diverge. In
particular, bn and an as n . However, an + bn = q
and, hence, an + bn q as n . Evidently, the relation q =
makes no sense. Furthermore, take c = 0 so that the sequence
can = 0 n = 0 is a constant sequence and, hence, converges to 0. The
relation 0 = 0 would not make any sense either. Similarly, put
an = n and bn = q/n. Then an diverges, an , while the sequence
bn converges bn 0 as n 0. However the sequence of the products
is a constant sequence an bn = q and, hence, converges. The relation
q = 0 makes no sense. If an = pn, q > 0, and bn = n, then
an and bn as n , where the sequence of the ratios
is meaningless.
converges: an /bn = q. The relation q =

The squeeze theorem also applies to sequences (see Fig. 8.3).


Theorem 8.4 (Squeeze Theorem). If cn an bn for n > N and
limn bn = limn cn = a, then limn an = a, where a can also be
+ or .

Example 8.5. Find the limit of an = sin(/ n).

Solution:
Since x sin x x if x 0, one has cn = / n
an / n = bn , where cn 0 and
bn 0 as n (see Example
2
8.2). By the squeeze theorem, sin(/ n) 0 as n .
Theorem 8.5. If limn |an | = 0, then limn an = 0.
This theorem follows directly from the denition of the limit of a
sequence where a = 0. The next result provides a convenient tool to
calculate limits of sequences using continuous functions.
Theorem 8.6. If an a as n and the function f is continuous at a, then
lim f (an ) = f (a).
n

49. INFINITE SEQUENCES

83

Figure 8.3. The squeeze theorem. The dots indicate numerical values (vertical axis) of the sequences bn
(blue), cn (black), and an (red). The integer n increases
from left to right (horizontal axis). The sequences bn and
cn converge to a number a. This means that the dierences |bn a| and |cn a| can be made arbitrarily small for
all n N and some integer N . Since cn an bn , the
dierence |an a| is also arbitrarily small for all n N .
By the denition of the limit, the sequence an must converge to a, too.
This theorem asserts that if a continuous function is applied to the
terms of a convergent sequence, the result is also convergent.
Proof. The continuity of f at a means that limxa f (x) = f (a)
or, by the denition of this limit, for any > 0, there is a corresponding
> 0 such that |f (x) f (a)| < whenever |x a| < . Having found
such , put  = and, by the denition of the limit limn an = a,
for any such  > 0, there is a corresponding integer N such that
|an a| <  = if n > N . Therefore, for any > 0, one can nd a
corresponding integer N such that |f (an ) f (a)| < for all n > N ,
2
which means that limn f (an ) = f (a).
Example 8.6. Find the limit of the sequence an = exp(1/n2 ).
Solution: Consider the sequence bn = 1/n2 . Then
1
lim bn = lim 2 = 0.
n
x x
Put f (x) = ex . Then an = f (bn ). By continuity of the exponential
function,
lim an = exp( lim bn ) = e0 = 1.
n
n
2

84

8. SEQUENCES AND SERIES

49.4. Exercises.

(1) Find a formula for the general term an of the sequences:




1 1 1 1
1
{an } = 1, , , , , , ... ,
3 5 7 9 11
 1 1 1 1

{an } = 1, , , , , ... ,
2 4 8 16
 1 2

3 4
5
{an } = , , , , , ... .
4 7 10 13 16
(2) Let {an } converge to a number a. Prove that the sequences
bn = an+k , where k is a xed positive integer, cn = an2 , and
dn = a2n converge and nd their limits.
(3) Show that the sequence an = (2n 1)/(n + 1) converges to
2. Determine how many terms of an lie outside the interval
(2 , 2 + ) if = 0.1 and = 0.01.
(4) Show that the sequence an = (n2 2(1)n n + 1)/(n2 + 1)
converges to 1. Given 2 > > 0, nd the number of terms
of the sequence that lie outside the interval (1 , 1 + ) as a
function of .
In the following exercises, determine whether the sequence
an converges or diverges for the specied ranges of parameters
(if the range of a parameter is not given, assume that it can
be any real number). If the sequence converges, nd its limit.
(5) an = 2n .
(6) an = 2n (1)n 2n .
(7) an = (3 5n2 )/(1 + n2 ).
(8) an = Pm (n)/Qk (n), where Pm and Qk are polynomials of degree m and
k, respectively.
(9) an = 1/ n2 + n + 1.
(10) an = (n2 + 1)1/3 /(n3 + 1)1/2 .
(11) an = (n
+ 2)3 /(n6 + n2 + 1)1/2 .

(12) an = 3 (n3 + 1)/(8n3 + 4n2 + 2n + 1).
(13) an = [Pm (n)]q /[Qk (n)]p , where q and p are positive numbers
and Pm and Qk are polynomials of degrees m and k, respectively.
(14) an = (2n + 3n )en .
(15) an = (2n + 3n )e2n .
(16) an = (2n +3n +5n )/(2n p+3n +5n p), where q and p are positive
numbers.
(17) an = (2n + 3n + 5n )/(2n p + 3n + 5n q), where q and p
are positive numbers.

50. SPECIAL SEQUENCES

85

(18) an = Pm (pn )/Qk (q n ), where Pm and Qk are polynomials of


degree m and k, respectively, and 0 < p, q < 1.
(19) an = q n sin(pn).
(20) an = tanh(n2 ).
(21) an = tan[n/(2 + 4n)].
(22) an = sin2 [(n2 + 2)/(2n2 + 5)].
(23) an = ln(an)/ln(bn), where a and b are positive numbers.
(24) an = ln(Pm (n))/ ln(Qk (n)), where Pm and Qk are strictly positive polynomials of degree m and k, respectively.
(25) an = n cos(1/n).
(26) an = (ln n)p /n.
(27) an = nq (ln n)p .
tan1 (n2 ).
(28) an =
(29) an = n2 + n n.
(30) an = n sin(p/n).
(31) an = n2 (cos(p/n) 1).
(32) an = n(q1/n 1), where q > 0.
(33) an = n2 n e n2 n.
(34) an = bn sin(p/bn ), where bn as n .
(35) an = (bn tan bn )/(bn )p , where p > 0 and bn 0 as n .
(36) an = f (bn )/g(bn ), where the functions f and g are twice differentiable and have a root b such that f  (b) = g  (b) = 0,
g  (b) = 0, and bn b as n .
(37) Prove that a sequence that is the sum of convergent and divergent sequences diverge.

50. Special Sequences


Theorem 8.7 (Special Sequences). Let p and q be real numbers.
(8.1)
(8.2)
(8.3)
(8.4)
(8.5)

lim

lim

p=1

if

p > 0,

n = 1,

nq
= 0 if
lim
n pn
n!
lim n = 0,
n n
qn
lim
= 0.
n n!

p > 1,

86

8. SEQUENCES AND SERIES

Proof.

(8.1). If p > 1, put an = n p 1. Then an > 0 and, by the binomial


theorem,
p = (1 + an )n = 1 + nan + n(n 1)an /2 + + nann1 + ann .
All terms in the right side are positive. Therefore by retaining only the
rst two terms, a smaller number is obtained:
1 + nan (1 + an )n = p.
It follows from this inequality that
p1
.
n

By the squeeze theorem, an 0 as n and hence n p = an +1 1.


The case p = 1 is trivial. If 0 < p < 1, the result is obtained by taking
reciprocals and using the basic limit laws:
0 < an

lim

1
1

=
=1
p = lim
n n (1/p)
limn n (1/p)

because 1/p > 1.


(8.2). Put an = n n 1. Then an 0 and, by the binomial theorem,
n = (1 + an )n
Hence, for n 2,

n(n 1) 2
an .
2

2
.
n1

By the squeeze theorem, an 0 or n n = an + 1 1 as n 0.


(8.3). Consider the function f (x) = xq ecx , where c > 0. By the
asymptotic property of the exponential function, f (x) 0 as x
for any q; the exponential grows faster than any power function (which
has been proved in Calculus I). Since an = f (n) for c = ln p > 0 if
p > 1, one concludes that
0 an

nq
= lim nq en ln p = lim xq ex ln p = 0.
n pn
n
x
lim

(8.4). The following inequality holds:


n!
1 2 3n
1
1 2 3
n
1
=
=
0 < an .
n
n
n n nn
n n n
n
n
n
By the squeeze theorem, an 0 as n .
(8.5). If q > 0, then there is a positive integer k such that k1 q < k,
an =

50. SPECIAL SEQUENCES

87

Figure 8.4. The sequence in the left panel is monotonically increasing, and the sequence in the right panel is
monotonically decreasing.
that is, k is the smallest positive integer such that q/k < 1. The
following inequality holds:
an =

qn
q
q
q q
q q
q
q
q
qk
=

q k1
q k1 = .
n!
1 2
k1 k
n
k
n1 n
n
n
k

By the squeeze theorem, 0 < an qn 0 as n and an converges


to 0. The case q = 0 is trivial. If q < 0, then |an | = |q n /n!| = |q|n /n!
2
0 as n and hence, by Theorem 8.5, an converges to 0, too.

Example 8.7. Find the limit of an = n nq , where q > 1, if it exists


or show that the sequence diverges.
Solution:
lim

nq = lim ( n n)q = ( lim n n)q = 1q = 1


n

by (8.2) and the basic limit laws.

50.1. Monotonic Sequences.

Definition 8.4 (Monotonic Sequences). A sequence an is said to


be
monotonically increasing if an an+1 ,
monotonically decreasing if an an+1
for all n = 1, 2, ....
The class of monotonic sequences consists of the increasing and the
decreasing sequences.
Example 8.8. Show that the sequence an = n/(n2 + 1) is monotonically decreasing.

88

8. SEQUENCES AND SERIES

Figure 8.5. A bounded sequence. The dots indicate


numerical values of an (vertical axis). The integer n increases from left to right (horizontal axis). All the numbers an lie in the interval: m an M .
Solution: The inequality an an+1 must be established. It is equivalent to the following inequalities obtained by cross-multiplication:
n
n+1
2
(n + 1)(n2 + 1) n[(n + 1)2 + 1]
2
(n + 1) + 1
n +1
n3 + n2 + n + 1 n3 + 2n2 + 2n
1 n2 + n.
The latter inequality is true for n 1. Therefore, an+1 an (in fact,
the strict inequality an+1 < an holds as well), and the sequence is
monotonically decreasing.
2
Definition 8.5 (Bounded Sequence). A sequence is said to be
bounded above if there is a number M such that
an M

for all n 1.

A sequence is said to be bounded below if there is a number m such


that
m an

for all n 1.

A sequence is said to be bounded if it is bounded above and below:


m an M

for all

n 1.

For example, the sequence an = 1/n is bounded: 0 < an 1. The


sequence an = en is bounded below, but not above.

50. SPECIAL SEQUENCES

89

Completeness axiom for the set of real numbers. The completeness ax-

iom for the set of real numbers says that if S is a nonempty set of real
numbers that has an upper bound M (x M for all x S), then S
has a least upper bound. By denition, the number a is a least upper
upper bound of S if, for any > 0, a is not an upper bound of
S. The least upper bound is called the supremum of S and denoted
sup S. Naturally, sup S M for any upper bound M of S. If S has
a lower bound m, then it also has the greatest lower bound, denoted
inf S (the inmum of S). The number inf S is a lower bound of S such
that inf S + is not a lower bound of S for any positive > 0; that is,
m inf S for any lower bound m of S. The completeness axiom is an
expression of the fact that there is no gap or hole in the real number
line.
Theorem 8.8 (Monotonic Sequence Theorem). Suppose {an } is
monotonic. Then {an } converges if and only if it is bounded.
Proof. Suppose an an+1 (the proof is analogous in the other
case). Let S be the range of {an }. If {an } is bounded, let a = sup S
be the least upper bound of S (it exists by the completeness axiom).
Then an a for all n 1. By the denition of sup S, for every > 0,
the interval (a , a] should contain an element of S (otherwise, a
would be an upper bound of S). Therefore there should exist an integer
N such that
a < aN a .
Since {an } increases, all numbers an , n N , lie in the interval (a, a]:
a < an a

|a an | < whenever n N .

By the denition of the limit of a sequence, this inequality shows that


2
{an } converges to a.
The proof is illustrated in Fig. 8.6.
Example 8.9. Investigate the convergence of the sequence dened
by the recurrence relation a1 = 2 and an+1 = 12 (an + 3).
Solution: Let us compute the rst few terms of the sequence a1 = 2,
a2 = 2.5, a3 = 2.75, and so on. The initial terms suggest that the
sequence is monotonically increasing, and one can try to prove this
property an an+1 for all n. A commonly used technique to do so
is mathematical induction. The statement is true for n = 1. Suppose
that the statement is true for n = k (the hypothesis of mathematical
induction). Then, using this hypothesis, one has to prove that the
statement holds for n = k + 1. If the proof goes through, then starting

90

8. SEQUENCES AND SERIES

Figure 8.6. Monotonic sequence theorem. A bounded


monotonic sequence with numerical values indicated by
dots (vertical axis). The integer n increases from left to
right. If a = sup{an } is the least upper bound of all an ,
then, for any number > 0, a is not an upper bound
of the sequence. Since an increases monotonically, there
is an integer N such that all the numbers an , n N , are
greater than a and hence lie in the interval (a , a].
This means that an converges to a.
with n = 1, one can establish the statement for n = 2, n = 3, an so on.
This is the basic idea of mathematical induction. Using the recurrence
relation,
1
1
(ak+1 + 3) > (ak + 3) = ak+2 > ak+1 .
ak+1 > ak =
2
2
Thus, the sequence is indeed monotonically increasing. If it happens to
be bounded, then it converges. Again, mathematical induction turns
out to be helpful. The rst terms suggest that an < 3. This is true for
n = 1. Suppose the inequality is true for n = k. Let us try to prove
that this hypothesis implies that the inequality holds for n = k + 1.
Using the recurrence relation,
1
1
(ak + 3) < (3 + 3) = ak+1 < 3.
ak < 3 =
2
2
Thus, the sequence is monotonic and bounded and hence converges. If
the sequence an converges to a, then so does the sequence an+k for any
integer k (the sequence bn = an+k is a subsequence of the convergent
sequence an and hence bn converges to a by Theorem 8.3). Since the
existence of the limit has been established, the limits of both sides of
the recurrence relation must coincide by the basic limit laws:
1
1
lim an+1 = ( lim an + 3) = a = (a + 3) = a = 3.
n
n
2
2

50. SPECIAL SEQUENCES

Thus, an 3 as n .

91

Example 8.10. Investigate


the convergence
of the sequence dened
by the recurrence relation a1 = 2 and an+1 = 2 + an .
Solution: The rst few terms of the
sequence suggest that the se

quence is increasing: a1 = 2 < a2 = 2 + 2. Let us try to prove


the inequality an < an+1 by induction. Suppose it is true for n = k.
Then, by monotonicity of the square root function and the recurrence
relation,

ak < ak+1 ak < ak+1 2 + ak < 2 + ak+1 ak+1 < ak+2 .


The rst terms of the sequence suggest that a1 < 3 and a2 < 3. Let us
try to prove that an < 3 for all n by induction. Suppose the inequality
holds for n = k. Then, by the recurrence relation,

ak < 3 = 2 + ak < 5 = 2 + ak < 5 < 3 = ak+1 < 3.


Thus, the sequence is monotonic and bounded, and hence it converges.
If its limit is a, then using the basic limit laws

lim an+1 = lim 2 + an = 2 + lim an a = 2 + a a = 2.


n

2
50.2. Exercises.

In (1)(15), nd the limit of the sequence {an } or show that it does


not exist. Assume that the parameters are real numbers.

(1) an = n 2n2 + 3.
(2) an = cos2n (n2 )/n.
(3) an = nrn .
(4) an = np en .
(5) an = n q n .
(6) an = pn n q n .
(7) an = n pn n q n .
(8) an = Pm (n)q n , where Pm is a polynomial of degree m.
(9) an = (2n 1)!!/(2n)n , where (2n 1)!! = 1 3 5 (2n 1).
(10) an = (2n)!!/(2n 1)!!, where (2n)!! = 2 4 (2n).
(11) an = qn /(2n)!!.
n
(12) an =
3n + 5n .
n
(13) an = 1 + pn.
(14) an = n qn2 + pn + 1, where q > 0.

92

8. SEQUENCES AND SERIES


(15) an = n Pm (n), where Pm is positive polynomial of degree
m > 1.
(16) Prove that every convergent sequence is bounded.
In (17)(22), determine whether the sequence is monotonic or not
monotonic. Is the sequence bounded?
(17) an = (2)n .
(18) an = (1)n n.
(19) an = nen .
(20) an = n + n1 .
(21) an = sin(qn)/n, where q is real.
(22) an = (1 + q/n)n , where q is real.
In (23)(27), nd the limit of the sequence or show that it does not
exist.
(23) a1 = 1 and an+1 = 4 an .
(24) a1 = 1 and an+1 = 1/(1 + an ).
(25) a1 = 1 and an+1 = 3 1/an .
(26) a1 = 2 and an+1 = 1/(3 an ).
(27) a1 = 1 and an+1 = 1 + 1/(1 + an ).
(28) The size of an undisturbed sh population has been modeled
by the formula pn+1 = bpn /(a + pn ), where pn is the sh population after n years and a and b are positive constants that
depend on the species and the environment. Suppose that
p0 > 0. Show that pn+1 < (b/a)pn . Then prove that pn 0
if a > b; that is, the population dies out. Finally, show that
pn b a if b > a. Hint: Show that pn is increasing and
bounded, 0 < pn < b a if p0 < b a. If p0 > b a, then pn
is decreasing and bounded, pn > b a.
51. Series
51.1. Basic Definitions and Notation. With a sequence {an }, one can

associate a sequence {sn }, where


sn =

n


ak = a1 + a2 + + a n .

k=1

The symbol
(8.6)


n=1

an = a1 + a2 + a3 +

51. SERIES

93

is called an innite series, or just a series. The numbers sn are called


the partial sums of the series (8.6). The
limits of summation are often
omitted
to denote a series. The symbol an also stands for an innite
series
n=1 an . If {sn } converges to s, then the series is said to converge
and one writes

n


an = s
or
lim
ak = s.
n=1

k=1

The number s is called the sum of the series. If the sequence of partial
sums {sn } diverges, the series is said to diverge. It should be understood that s is the limit of a sequence of sums, and it is not obtained
merely by addition.

For example, the sequence of partial sums for the series (1)n
is s1 = 1, s2 = 1 + 1 = 0, s3 = s2 1 = 1, or, generally,
sn = ((1)n 1)/2. This sequence diverges as it has two subsequences
s2n = 0 and s2n1 = 1, which converge to dierent numbers, 0 = 1.
If one simply uses addition, dierent values for the sum of the series
may be obtained:


an =(a1 + a2 ) + (a3 + a4 ) + (a5 + a6 ) + = 0 + 0 + = 0,
n=1


n=1

an =(a1 + a2 + a3 ) + (a4 + a5 + a6 ) + = 1 1 = ,
an =a1 + (a2 + a3 ) + (a4 + a5 ) + = 1 + 0 + 0 + = 1.

n=1

Generally, by grouping terms in the sum in dierent ways (according


to the associativity of addition), the sum is found to be any integer!
The reader is advised to verify this. Thus, the addition rules cannot
generally be applied to evaluate the sum of a series.
51.2. Geometric Series. Take a piece of rope of length 1 m. Cut it in

half. Keep one half and cut the other half in two pieces of equal length.
Keep doing this, that is, keeping one half and cutting the other half in
two equal-length pieces. The total length of the retained pieces is

1
1
1 1 1
1

1 1
.
+ + + =
1 + + + =
2 4 8
2
2 4
2 n=0 2n
This series must converge. The partial sum sn here is the total length of
retained pieces. The sequence {sn } is monotonically increasing (after
each cut piece of rope is added) and bounded by the total length 1. So

94

8. SEQUENCES AND SERIES

it converges. From the geometry, it is also clear that 1 sn = 1/2n ,


where n is the number of cuts, and hence sn 1 as one would expect
(the total length of the rope). So it is concluded


1
= 2.
2n
n=0
This series is an example of the geometric series:


1 + q + q2 + q3 + =
qn,
n=0

where q is a number. The geometric series does not converge for any
value of q.
Theorem
 n 8.9 (Convergence of a Geometric Series). A geometric
series n=1 q converges if |q| < 1, and, in this case,


1
qn =
,
|q| < 1,
1q
n=0
and the series diverges otherwise.
Proof. If q = 1, the sequence of partial sums obviously diverges.
If q =
1, one has
sn = 1 + q + q 2 + + q n1

qsn = q + q 2 + q 3 + + q n .

Subtracting these equations, one infers


sn qsn = 1 q n

sn =

1 qn
.
1q

Therefore,
1
1
1 qn
=

lim q n .
n
n 1 q
1 q 1 q n
It has been found (Example 8.3) that the sequence an = q n converges
only if |q| < 1, and, in this case, q n 0 as n . If |q| 1, the
geometric series diverges.
2
lim sn = lim

32
27

Example 8.11. Analyze the convergence of the series 4 83 + 16


9
+ .

Solution: The series can be written in the form 4q 0 + 4q 1 + 4q 2 + 4q 3 +


, where q = 2/3. So its partial sums are four times the partial
sum of the geometric series with q = 2/3. Therefore,

2 n


12
4
2 n
4
=4
=
.

2 =
3
3
5
1

(
)
2
3
n=1
n=1

51. SERIES

95

When real numbers are presented in decimal from, one often encounters a situation when a number has a repeated pattern of decimal
places. Take, for example, the number 1.2131313...; that is, the combination 13 repeats itself in all decimal places starting in the second
decimal place.
Example 8.12. Is the number 1.2131313... rational or irrational?
If it is rational, write it as a ratio of integers.
Solution: By denition of the decimal representation,
1.2131313... = 1.2 +
= 1.2 +

13
13
13 
1 n
13
+
+
+

=
1.2
+
103 105 107
103 n=0 100

13 100
12
13
1201
=
+
=
.
3
10 99
10 990
990

51.3. Necessary Condition for a Series to Converge. The following the-

orem follows from the limit laws applied to the sequences of partial
sums.

Theorem
8.10
(Properties
of
Series).
Suppose
that
the
series
an

and
bn are convergent and their
 sums are s and
 t, respectively. Let
c be a number. Then the series (an + bn ) and
can converge and



an +
bn = s + t
(an + bn ) =


an = cs .
(can ) = c
Indeed,
} and {tn } are the sequences of partial sums of the
 if {sn
series
a
and
n
 bn , respectively, then the partial sums of the series

(an + bn ) and (can ) are sn + tn and csn , respectively. By the limit
laws, sn + tn s + t and csn cs.

Note that the convergence
of

the series (an + bn ) does not imply
the convergence of
an and
bn . For example, put an = 1 and
bn = 1. Then sn = n and
 tn = n. The sequence sn + tn = 0
converges to 0. Therefore (an + bn ) = 0. However,
 the sequences

a
1 and
s
n = n and
n =
 tn = n diverge and
 so do the series

bn = (1). The equality (an + bn ) =
an +
bn becomes
meaningless (0 = ). Recall that the basic limits laws hold
only for convergent sequences (Section 49.3). This shows that the rules
of algebra for nite sums are not generally applicable to series. Only
series from a special class of absolutely convergent series, discussed later,
behave pretty much as nite sums.

96

8. SEQUENCES AND SERIES

Every theorem about sequences can be stated in terms of series and


vice versa. The
 sequence {an } can be expressed via the partial sums
of the series
an by putting a1 = s1 and
an = sn sn1 ,

n > 1.

In particular, if the series converges, meaning that sn s as n ,


then also sn1 s. By the basic limit laws, the limit on both sides of
the above recurrence relation may be taken:
lim an = lim (sn sn1 ) = lim sn lim sn1 = s s = 0 .

Thus, for a convergent series


verges to 0.

an , the sequence {an } necessarily con-

Theorem
 8.11 (Necessary Condition for a Series to Converge). If
the series
an converges, then limn an = 0.
The converse is not generally true. The condition limn an = 0 is
not sucient for a series to converge. However, it can still be used as
a test for divergence of a series.
Corollary 8.1 (Test for Divergence of a Series). If the 
limit
0, then the series
an
limn an does not exist or if limn an =
diverges.
Example 8.13. Show that the series

n3 /(3n3 + 1) diverges.

Solution:
1
n3
1
lim an = lim
= = 0,
= lim
n
n 3n3 + 1
n 3 + 13
3
n
so the series diverges.
2
If the necessary condition is satised, the series may converge or
diverge. The sequence of partial sums has to be analyzed.
Example 8.14. Find the sum of the series
or show that it does not exist.

1
n=1 n(n+1)

if it exists

51. SERIES

97

Solution: The necessary condition for convergence is evidently satised. So the sequence of partial sums has to be analyzed for convergence:
n
n



1
1
1
=

sn =
k(k + 1) k=1 k k + 1
k=1

1
1
1 1
1 1
1
= 1
+

+ +

2
2 3
3 4
n n+1
1
1 as n .
=1
n+1
So the sequence {sn } converges to 1 and hence


1
= 1.
n(n
+
1)
n=1
2

This example is a particular case of a telescopic series.


Theorem
8.12 (Convergence of a Telescopic Series). A telescopic

series n=1 (an an+1 ) converges if limn an = a, and, in this case,


(an an+1 ) = a1 a,
n=1

The proof is analogous to the above example and based on the fact
that the sequence of partial sums of a telescopic series sn = a1 an+1
converges to a1 a. The details are left to the reader as an exercise.
51.4. Exercises.

In (1)(4), nd the sequence of partial sums of the series and investigate


its convergence.
(1)n1
1 1 1
(1) 1 + + +
+
2 4 8
2n1


 


1 1
1
1 1
1
(2)
+
+
+
+
+
+ +
2 3
4 9
2n 3n
1 2 3
n
+ + + + n +
2 4 8
2
5
2n 1
3
1
+
(4) + 2 + 3 + +
2 2
2
2n
Hint: Put n = 1 + 1 + + 1 (the sum of n units) in (3) and (4), when
calculating the partial sum, then use partial sums of a geometric series.
(3)

98

8. SEQUENCES AND SERIES

In (5)(9), determine whether the (geometric) series converges or diverges (p > 0 and q > 0). Find the sum of the series if it exists.


n
3n+1
n=0

(5)

(6)


q n + pn
(p + q)n
n=1

(8)


en
3n1
n=1

(9)


(5)n

(7)

n=0

32n


1 + 2n + 3n + 4n 5n

5n

n=0

In (10)(14), determine whether the series converges or diverges by


expressing it as a telescopic series. Find the sum of the series if it
exists.



1
3
(11)
(10)
n2 1
n2 + 3n + 2
n=2
n=1
(12)
(14)


1
n
(13)
ln
n+1
(3n 2)(3n + 1)
n=1
n=1


n+22 n+1+ n
n=1

In (15)(20), determine whether the series converges or diverges. If it


converges, nd its sum. Here p is a positive number, p > 0.
(15)
(18)


n=1

k2
k2 + k + 1
(sin p)n

n=1

(16)


2 3n
n=1

(19)


en
n=1

np

5n
(20)

(17)

n=1

e2n +

n=1


4
n(n + 1)

In (21) and (22), express the number as a ratio of integers


(21) 1.23232323....

(22) 1.53525252....

In (23)(25), nd the values of x for which the series converges. Find


the sum of the series for those values of x.




xn
sinn x
(23)
(24)
(25)
(x 5)n
n
n
2
3
n=1
n=1
n=1
In (26) and (27), solve the equation.
(26)


(1 + x)n = 3
n=2

(27)


n=0

enx = 9

52. SERIES OF NONNEGATIVE TERMS

99


(28) Suppose that
n=1 = a. Let 1 < k1 < k2 < < kn <
be a strictly increasing sequence of integers. Let An = akn +
akn +1 + + akn+1 1 . For example, for k1 = 2, k2 = 5, and
k3 = 7, A1 = a1 , A2 =a2 + a3 + a4 , and A3 = a5 + a6 ,
respectively. Prove that
n=1 An = a. In other words, the
series obtained from a convergent series by grouping its terms
without changing the order of its terms converges and has the
same sum. The converse is not true (see Section 56).
52. Series of Nonnegative Terms
In many applications, the terms of a series decrease monotonically.
It appears that there is a relation between convergence of such series
and convergence of improper integrals over an interval [1, ). This
relation allows one to establish a necessary and sucient condition for
series of nonnegative terms to converge.
52.1. The Integral Test. Suppose f (x) is a positive, continuous, mono-

tonically decreasing function on [1, ) such that f (x) 0 as x .


Suppose also that the improper integral
 a

f (x) dx = lim
f (x) dx = If
a

exists. The value If is the area under


 the graph y = f (x) over the interval [1, ). Consider the series
n=1 f (n). The necessary condition
for convergence is fullled because f (n) 0 as n . To investigate
the convergence of the series, one has to analyze the convergence of its
partial sums:
sn =

n


f (k) = f (1) + f (2) + + f (n).

k=1

Since the function f (x) monotonically decreases and is continuous on


every interval [k, k + 1], it attains its minimal and maximal values
f (k + 1) f (x) f (k) on this interval and therefore (see Fig. 8.7)
 k+1
f (x) dx f (k).
(8.7)
f (k + 1)
k

Let us take the sum over k = 1, 2, ..., n 1 of the left inequality in


(8.7). In the left side, the summation yields f (2) + f (3) + + f (n) =
sn f (1). By moving f (1) to the other side, an upper bound for sn is

100

8. SEQUENCES AND SERIES

obtained:

sn f (1) +

f (x) dx + +
1

f (x) dx = f (1) +
n1

f (x) dx .
1

Similarly, taking the sum over k = 1, 2, ..., n of the right inequality in


(8.7), the lower bound is derived:
 2
 3
 n+1
 n+1
f (x) dx +
f (x) dx + +
f (x) dx =
f (x) dx,
sn
1

Therefore

(8.8)

n+1

f (x) dx sn f (1) +

f (x) for all n 1.


1

This inequality shows that the following theorem holds.


Theorem 8.13 (Integral Test). Suppose f is a continuous, posithe series
tive,
 decreasing function on [1, ) and let an = f (n). Then

n=1 an converges if and only if the improper integral 1 f (x) dx converges. In other words,



f (x) dx converges =
f (n) converges,
1

n=1

f (x) dx
1

diverges

f (n) diverges.

n=1

Proof. If the improper integral converges to a number If , then


by (8.8) the sequence of partial sums is bounded, sn f (1) + If , and
monotonically increases, sn sn + f (n + 1) = sn+1 . Therefore, it is
convergent by Theorem 8.8. If the improper integral diverges, then, for
n+1
any number M > 0, there is an integer N such that 1 f (x) dx M
for all n > N . By the left inequality of (8.8), M sn for all n > N ;
that is, {sn } is a monotonically increasing, unbounded sequence and
hence it diverges.
2

Figure 8.7. Integral test. An illustration of inequality (8.7).

52. SERIES OF NONNEGATIVE TERMS

101

Remark. Suppose that an = f (n), where f (x) is a function on


[1, ), such that it is continuous, positive, and decreasing on [N, ),
where N 1 is an integer. Then



an converges
f (x) dx converges;
(8.9)
N

n=1

that is, the integral test applies even if the sequence an becomes monotonically decreasing only for n N 1. This is easy to understand
by isolating the rst N 1 terms in the series




an = a1 + a2 + + aN 1 +
an = a1 + a2 + + aN 1 +
bn ,
n=1

n=N

n=1


where bn = aN +n1 . Convergence of
bn implies convergence of
an
and vice versa as they dier by a number. Put bn = g(n), where
g(x) = f (x+N 1), which is a continuous, positive, decreasing function
on [1, ), and



g(x) dx =
f (x + N 1) dx =
f (u) du
1

by changing the integration variable u = x + N 1.


52.2. Special Series of Nonnegative Terms.

Theorem 8.14 (Convergence of p-series). The p-series


1
n=1

np

converges if p > 1 and diverges if p 1.


Proof. If p 0, the series diverges because the necessary condition for convergence is not fullled, an if p < 0 and an = 1 = 0 if
p = 0. For p > 0, consider the function f (x) = xp , which is positive,
continuous, and decreasing on [1, ), and



 a
1
1
dx
1

if p = 1,
ap1
= p1
p
ln a
if p = 1.
1 x
So, by the integral test, the series converges if p > 1 because the
improper integral diverges if 0 < p 1 and converges if p > 1 (the
limit a exists only if p > 1).
2
 p
Harmonic series. The series
n diverges for all 0 < p 1 despite
that the necessary condition to 
converge is fullled: an = np 0. In
1
particular, the harmonic series
n=1 n diverges.

102

8. SEQUENCES AND SERIES

np depends
on the value of p > 1; that is, this series denes a function on (1, ).
This function is called Riemanns zeta function.

Riemanns zeta function. The sum of a p-series (p) =

Example3/28.15. Investigate the convergence of the series


.
n=1 (n + 2)
Solution: The series can be written as


n=1

 1
 1
1
1
=
=
1

+
.
3/2
3/2
3/2
(n + 2)3/2
n
2
n
n=3
n=1

The latter series is a p-series that converges for p = 3/2 > 1.

Theorem 8.15. The series


n=2

1
n(ln n)p

converges if p > 1, and it diverges if p 1.


Proof. Consider the function g(x) = x(ln x)p for x > 1. Its derivative reads g  (x) = (ln x)p1 (p + ln x). If p 0, then g  (x) > 0 for
all x > 1 and g(x) increases, while its reciprocal f (x) = 1/g(x) should
decrease. If p < 0, then g  (x) > 0 for all x > ep and hence g(x) increases, while f (x) = 1/g(x) decreases if x > ep > 1. Thus, for any p,
there is an integer N such that the function f (x) = 1/[x(ln x)p ] is continuous, positive, and decreases on [N, ). By the integral test (8.9),
the series in question converges if and only if the improper integral


dx
du
=
p
p
N x(ln x)
ln N u
converges, where the integration variable has been changed, u = ln x,
du = dx/x. This integral diverges if p 1 and converges if p > 1, and
the conclusion of the theorem follows.
2
On the use of the comparison test for improper integrals. The improper

integral of the function f (x) in the integral test often cannot be explicitly evaluated. The comparison test (Theorem 7.4) may be used to
assess the convergence of the improper integral.

Example
8.16.
the convergence of the series an where

Investigate
an = ( n + 1 n 1)/np , p > 0.

52. SERIES OF NONNEGATIVE TERMS

103

Solution: Let

x+1 x1
x+1 x1
x+1+ x1

f (x) =
xp
xp
x+1+ x1
2

= p
>0
x ( x + 1 + x 1)
for x 1. The idea is to nd lower and upper bounds for f (x) as functions such that the convergence of their improper integrals
is
easy to

analyze. It follows from from the obvious inequality x + 1 > x 1


that
2
2

< f (x) < p


p
2x x 1
2x x + 1
Then a lower bound is obtained as a power function by means of x+1 <
2x. Similarly, an upper bound follows from x 1 > x/2 for all x > 2.
Thus,

2
1
< f (x) < p+1/2
x
xp+1/2 2
for all x > 2. By Theorem 7.4 this inequality implies that the improper
integral of f (x) converges if and only if the improper integral of the
power function xp1/2 converges. The latter is the case if p + 1/2 > 1
or p > 1/2. Thus, the series in question converges if p > 1/2 and
diverges otherwise.
2
 np
Example 8.17. Investigate the convergence of the series n 2
where p 1.
p

Solution: Put f (x) = 2x > 0 if x 1. By monotonicity of the


exponential function and by the obvious inequality xp x for all x 1
and p 1, the function f is bounded from above by g(x) = 2x =
ex ln 2 . It follows from the inequality 0 < f (x) g(x) that


1
f (x)dx
g(x)dx =
0<
ln 2
1
1
and, hence, the series in question converges.

52.3. Estimate of the Sum. If a partial sum sn is used


 to estimate the

sum of a convergent series of nonnegative terms


f (n), how good
is such an estimate? The remainder s sn has to be investigated to
answer this question.
Corollary 8.2 (Estimate of Sums). Suppose f is a continuous,
positive, decreasing function on [1, ) and let an = f (n). If the series

104

8. SEQUENCES AND SERIES

an converges to a number s, then




f (x) dx s sn
n+1

f (x) dx,

where {sn } is the sequence of partial sums.


Proof. The rst inequality is obtained by taking the limit n
in (8.8) with the result




f (x) dx
an f (1) +
f (x) dx,
(8.10)
1

n=1

which is a legitimate operation because (8.8) holds for all n and the
series converges (and so does the improper integral by the integral
test). The remainder estimate is obtained by subtracting (8.8) from
(8.10). Note the value of the improper integral does not coincide with
the sum; it only determines an interval (8.10) in which the sum of a
series lies.
2

Example 8.18. Test the series n=1 (n2 + 1)1 for convergence or
divergence. If it converges, estimate its sum.
Solution: Put f (x) = (x2 + 1)1 , which is a continuous, positive,
 decreasing function on [1, ), such that the series in question is
f (n).
Therefore, the integral test applies, and the series converges because

a

dx

1 
= lim tan x = lim tan1 a = = .
2
a
a
x +1
4
2
4
4
1
1
2
By (8.10), its sum lies in the interval 4 s f (1) + 4 = 12 + 4 .

n
for convergence or diExample 8.19. Test the series
n=1 ne
vergence. If it converges, estimate its sum.
Solution: Consider the function f (x) = xex . Since f  (x) = ex
xex = (1 x)ex 0 if x 1, the function decreases on [1,
 ), and
the integral test applies to assess convergence of the series
f (n):


a 
1 1
2

xex dx =
x dex = lim xex  +
ex dx = + = ,
a
e e
e
1
1
1
1
where the integration by parts has been used to evaluate the integral.
The series converges to a number s that lies in the interval 2e1 s
2
f (1) + 2e1 = 3e1 .
Example 8.20. Estimate values of Riemanns zeta function (p).
How many terms does one need to retain in the partial sum sn to approximate (p) so that the error is less than 10N ?

52. SERIES OF NONNEGATIVE TERMS

105

Solution:
zeta function is dened by the sum of the series
 Riemanns
p
n
.
For
p > 1,
(p) =
n=1


dx
x1p a
a1p
1
1
=
lim
=
lim

=
.

p
a 1 p 1
a 1 p
x
1p
p1

Since f (1) = 1, by (8.10),


p
1
(p)
.
p1
p1
By Corollary 8.2,

0 (p + 1) sn

1
dx
=
.
xp+1
pnp

If (p + 1), p > 0, is to be approximated with an error not exceeding


10N , then the remainder should be less than 10N , which yields the
condition on the number of terms: pn1 p < 10N or np > 10N /p or
n > 10N/p /p1/p . The smallest n satisfying this inequality is 1 plus the
integer part of 10N/p /p1/p . For example, take p = 1 and N = 2. Then
2
(2) sn < 0.01 for all n 101.
52.4. Exercises.

In (1)(15), determine whether the series converges or diverges.


(1)
(4)
(7)
(10)


n=1

2 n

ne

n2 e

n=1

1
n9/8

n=1


n=1

(13)

(2)
n

(5)
(8)

n=1


n=2

n 2n 5
3n7/3
n2

(14)

1
n(n + 4)

n=1

(11)


n=1

(3)
(6)

(ln n)
n


e1/n
n=1

(9)

n=1


n=1


n=2

1
2
n 4n + 5


2n + 1
n(n + 1)
n=1

(15)

1

n(n + 1)(n + 2)

1 n ln n
n2
(12)


n=1

n4


tan1 n
n=1

n2 + 1

n
+1

106

8. SEQUENCES AND SERIES

In (16)(26), determine the values of p for which the series is convergent.





n + |p| n |p|
p
(16)
, p>0
(17)
pn + 100
np
n=1
n=1
(18)

exp(np )

(19)

n=1

(21)
(24)

an , a > 1 (20)

n=1

|p|ln n

n=1

1
(22)
n(1 + n2 )p (23)
np en
p
n
ln
n(ln(ln
n))
n=3
n=1
n=1





p
1
p
np n ln n (26)
n(ln n)

(25)
n n+1
n=1
n=1
n=1

(27) How many terms of the series in Theorem 8.15 does one have
to add to approximate its sum with an error less than 10N ?
In (28)(30), estimate the sum of the series and, in (28) and (29), also
determine how many terms has to be added to approximate its sum
with an error less than 0.01.





2n2 + 5
1
n
2
(29)
(30)
(28)
4
2
4
n + 5n + 4
n 1
n=1
n=1
n=2
(31) Show that the sequence
1 1
1
an = 1 + + + + ln n
2 3
n
converges. The limit limn an = is called the Euler number. Hints: (i) Use (8.8) to show that if sn is the partial sum
of the harmonic series, then sn 1 + ln n and hence an 1
(i.e., the sequence {an } is bounded). (ii) Interpret an an+1
as a dierence of areas to show that {an } is monotonic.
53. Comparison Tests
Consider the series


n=1

an =


n=1

n4

1
.
+1

It has terms smaller than the corresponding terms of the convergent


p-series:



1
bn =
n4
n=1
n=1

53. COMPARISON TESTS

107


because an < bn
for all n. If sn is the partial sum for an and tn is the
partial sum for bn , then sn < tn . Since tn converges to a number t, it
is bounded, tn < t, and hence sn < t. The sequence {sn } is monotonic
and bounded, and therefore it converges. This line of arguments admits
a generalization.


bn
Theorem 8.16 (Comparison Test). Suppose that
an and
are series such that an 0 and bn 0 for all n N and some integer
N 1. Then


an converges,
bn converges and an bn for all n N =


an diverges.
bn diverges and an bn for all n N =


Proof. The series
n=1 an and n=N an dier by a number a1 +
a2 + + aN 1 . So convergence of
n=N an implies convergence of


a
and
vice
versa.
Therefore,
it
is
sucient to consider the case
n=1 n
N = 1. The sequences of partial sums {sn }, sn = a1 + a2 + + an ,
and {tn }, tn = b1 + b2 + + bn , are
 monotonically increasing sequences
because an 0 and bn 0. If
bn converges, then tn t as n
and tn t for all n. By the hypothesis an bn for all n 1,
shows that {sn } is monotonic and
and therefore sn tn t, which 
bounded and, hence, converges. If
bn diverges, then tn . From
the hypothesis an bn , it follows that sn tn . Thus, sn as
n .
2
When 
applying the comparison test, the convergence properties of
the series
bn should be easy to establish. In many instances, a good
choice is a geometric series (Theorem 8.9), a p-series (Theorem 8.14),
a telescopic series (Theorem 8.12), and the series in Theorem 8.15.

3
2
Example 8.21. Test the series
n=1 (2n + 1)/(3n + n + 1) for
convergence.
Solution: Since an > 0 is a rational function of n, a convenient choice
of a series in the comparison test is a p-series:
2n + 1
2 1
1 1
2n + 1
0< 3
=
+
.
<
2
3
2
3n + n + 1
3n
3n
3 n3
The series


2 1
1 1
2n + 1
=
+
3n3
3 n=1 n2 3 n=1 n3
n=1
converges as the sum of two convergent p-series.
2


Example 8.22. Test the series n=1 ( n + 1 n) for convergence.

108

8. SEQUENCES AND SERIES

Solution: One has

( n + 1 n)( n + 1 + n)

an = n + 1 n =

n+1+ n
1
1
1
1
= bn .
=

=
n+1+ n
2n + n
1+ 2 n

The p-series
1/ n diverges and so does the series in question by the
comparison test.
2

an and
 Theorem 8.17 (Limit Comparison Test). Suppose that
bn are series with positive terms. Let c = limn (an /bn ).


then
an converges.
If c = 0 and
bn converges,


If 0 < c < , 
then
an convergesif only if
bn converges.
an diverges.
If c = and
bn diverges, then
Proof. If c = 0, then there is an integer N such that an /bn < 1
for all n > 
N by the denition ofthe limit. Hence, an < bn for all
an converges by the comparison
n > N . If
bn converges, then
test. If c (0, ), then, by the denition of the limit, for any number
c > > 0, there is an integer N such that

an
an 

< c + = M for all n > N.
c  < m = c <
bn
bn
Therefore,
mbn < an < M bn

for all n > N.



By the comparison test, convergence of
bn implies convergence
of


bn implies
an due to the
inequality an < M bn . The divergence of
divergence of
an , again by the comparison test as mbn < an . If
>M
c = , then, for any M > 0, there is an integer N such that an /bn 
when n > N . The inequality
a
>
M
b
shows
that
divergence
of
bn
n
n

implies divergence of
an by the comparison test.
2
It is often helpful to investigate the asymptotic behavior of an as
n to identify a suitable bn in the limit comparison test (Recall
the notion of a slant asymptote of a function f (x) in Calculus I).


3/2
+ n)/ n6 + n4 + 1 for
Example 8.23. Test the series
n=1 (2n
convergence.
Solution: Let us nd the asymptotic behavior of an as n . For
large n, the top of the ratio is n3/2 (2 + n1/2 ) 2n3/2 because the term
n1/2 may be neglected as compared to 2. The asymptotic behavior

53. COMPARISON TESTS

109

will be denoted by , in particular, 2n3/2 + n 2n3/2 . The bottom of


the ratio behaves as (n6 )1/2 = n3 . Therefore,
2n3/2 (1 + 21 n )
2n3/2 + n
2n3/2
an =
=

= 2n3/2 = bn
3
6
4
n
n +n +1
n3 1 + n12 + n16
in the asymptotic region n . This shows that
1 + 21 n
an
= lim
lim
n bn
n
1+ 1 +
n2

= 1 = c.
1
n6


By the limit
comparison
test,
the
series
an converges because the

 3/2
p-series
bn = 2 n
converges.
2


5
n
3
n
Example 8.24. Test the series
n=1 (n + 3 )/ n + 5 for convergence.
Solution: Recall that the power function increases more slowly than
the exponential function, that is, np q n 0 as n for any q > 1
and any p. Hence, the asymptotic behavior of an is
n

3n (1 + n5 3n )
3n
3
an = n/2
n/2 =
= bn .
5 (1 + n3 5n )1/2
5
5
This shows that the ratio
 an /bn converges to c = 1 as n . Bythe
limit comparisontest,
an diverges because the geometric series bn
diverges (q = 3/ 5 > 1).
2
53.1. Estimating Sums. Let

n=1 bn be a convergent series of nonnegative terms. Suppose that


 by the
0 an bn for all n. Then
an can
comparison test, the series n=1 an converges. The sum
 of
be estimated
by
comparing
remainders
for
the
series
b
.
Indeed,
n


b
=
t.
Let
{t
}
and
{s
}
be
the
sequences
of
put
an = s and
n
n
n


an , respectively. The remainders satisfy
partial sums for
bn and
the inequality:

s sn = an+1 + an+2 + bn+1 + bn+2 + = t tn .


So the accuracy of the approximation s sn is the same or higher than
that of the approximation t tn . If, for example, one nds that n = N
is sucient for the equality t = tN to be correct within a specied error,
then s = sN is also correct within the same or even smaller error. The
remainder is easy to estimate if bn = f (n) and the function f is simple

enough to integrate, t tn n f (x) dx (Corollary 8.2).

110

8. SEQUENCES AND SERIES

Example 8.25. Determine


how many terms are needed to estimate

1
tan
(n2 )/(n3 + 1) correct within an error
the sum of the series
n=1
not exceeding 104 .
Solution: The function tan1 x is monotonically increasing for x > 0
approaching asymptotically the value /2. Therefore,
an =

tan1 (n2 )
1
1
= bn ,

3
3
n +1
2 n +1
2 n3

n 1.

Hence,


2
dx

4
s sn t tn
10 157.1.
=

10

2 n x3
4n2
2
So the needed accuracy is guaranteed if n = 158 or larger.
2

53.2. Exercises.

In (1)(15), determine whether the series converges or diverges.


(1)
(4)


n=1


n=1

(7)
(10)
(13)

1
2
n + 3n + 2

(2)

n
n5/3 + n1/3 + 1


1 + (1)n
n=1


n=1


n=1

n3/2

+1

1/ n

e
n+1
1
n1+1/n

n+1
2
n + 3n + 2

n=1

(5)

n=2

(3)

n(ln n)
n2 + 1

(6)

n=1


n=2

(14)

( n n 1)n
n=1

cos2 (n)
n2


1 + 2n
(9)
n2 + 2n
n=1

1
n3 + n + 1
n=1

1

sin2
(11)
(12)
n
n=1
(8)

2n
n100 + 2n/2

(15)


n!
nn
n=1


n2
n=1

n!

In (16)(20), determine the values of the parameters for which the


series converges.




q n
sin2 (xn)
p
q
p
(17)
(n + n + 1) (18)
n 1+
(16)
n2
n
n=1
n=1
n=1






p
p
n1
(19)
( n + 1 n)
(20)
( n + 1 n) ln
n+1
n=1
n=1
(21) Assume an = PN (n)/QM (n), where PN and QM are polynomials of degree N and M , respectively, and 
QM (n) > 0 for all
n. Investigate the convergence of the series
an .

54. ALTERNATING SERIES

111

In (22) and (23), determine how many terms one needs in the partial
sum to estimate the sum of the series correct within an error less than
0.001.



sin3 n
2n

(22)
(23)
n + 3n
n3 + n
n=1
n=1
(24) Consider a sequence {an }, where an can take any value from
the set {0, 1, 2, ..., p 1} and p > 1 is an integer. The meaning
of the representation 0.a1 a3 a3 ... of a number with base p is
that
a1 a2 a3
+ 2 + 3 + .
(8.11)
0.a1 a3 a3 ... =
p
p
p
When p = 10, the decimal system is obtained. The binary
representation corresponds to p = 2. The Maya used p = 20
(the number of ngers and toes). The Babylonians used p =
60. Show that the series (8.11)
 always converges.
an converges, then
ln(1 + an )
(25) Show that if an > 0 and
converges, too.
 2
 2
and
bn converge, then the
(26) Prove 
that if the series
a
n

2
series
|an bn | and (an + bn
) converge.
(27) Prove that the convergence
of
an , where an > 0, implies the

an /n.
convergence
of

(28) If
an converges and
if the sequence {bn } is monotonic and
bounded, prove that
an bn converges.

an di(29) Prove that if limn nan = c = 0, then the series
verges.


bn , con(30) Let the series of nonnegative terms,
an and
verge.
 about the convergence of the se What can be said
min(an , bn ), where min(p, q) and
ries
max(an , bn ) and
max(p, q) are the smallest and largest numbers in the pair
(p, q), respectively, and min(p, p) = max(p, p) = p?
54. Alternating Series
Definition 8.6 (Alternating Series). Let {bn } be a sequence of
nonnegative terms. The series

(1)n1 bn = b1 b2 + b3 b4 + b5

n=1

is called an alternating series.

112

8. SEQUENCES AND SERIES

For example, the series


 (1)n1
1 1 1 1
1 + + + =
2 3 4 5
n
n=1

(8.12)

is an alternating series. It is called the alternating harmonic series.


Theorem 8.18 (Alternating Series Test). If a sequence of positive
terms {bn } is monotonically
decreasing and limn bn = 0, then the

n1
alternating series
(1)
bn converges:
n=1
(i) bn+1 bn for all n
(ii) limn bn = 0


(1)n1 bn

converges.

n=1

Proof. The convergence of the sequence of partial sums {sn } is


to be established. Consider a subsequence of even partial sums {s2k }.
One has s2 = b1 b2 0, s4 = s2 + (b3 b4 ) s2 , and, in general,
s2k = s2(k1) + (b2k1 b2k ) s2(k1) s2(k2) s2 0
by the monotonicity of the sequence {bn }. Thus, the subsequence {s2k }
is monotonically increasing. By regrouping the terms in a dierent way,
one can see that
s2k = b1 (b2 b3 ) (b4 b5 ) (b2k2 b2k1 ) b2k b1
because all numbers in parentheses are nonnegative by hypothesis (i),
which shows that {s2k } is also bounded. Therefore, it converges by the
monotonic sequence theorem:
lim s2k = s.

For the subsequence of odd partial sums s2k+1 = s2k + b2k+1 , one infers
by the limit laws and hypothesis (ii) that
lim s2k+1 = lim s2k + lim b2k+1 = s + 0 = s.

The convergence of two particular subsequences of a sequence to the


same number s does not generally guarantee that the sequence converges to s (all its subsequences should converge to s). By denition,
the limits of {s2k } and {s2k+1 } mean that, given any number > 0,
there are positive integers N1 and N2 such that |s2k s| < if k > N1
and |s2k+1 s| < if k > N2 . Put N = max(2N1 , 2N2 + 1). Then
2
|sn s| < for all n > N , which means that sn s as n .
By this test, the alternating harmonic series (8.12) converges because the sequence bn = 1/n is monotonically decreasing and converges
to 0.

54. ALTERNATING SERIES

113

Figure 8.8. Alternating series test. An illustration of


its proof where two subsequences, s2k and s2k1 , of the
sequence sn of partial sums are analyzed for convergence.
Example 8.26. Test the series


n=1

sin(n/2)/n for convergence.

Solution: One has sin(n/2) = 1, 0, 1, 0, 1, ... for n = 1, 2, 3, 4, 5, ...,


respectively, or, in general, for odd n = 2k 1, sin(n/2) = (1)k1 ,
while for even n = 2k, sin(n/2) = sin(k) = 0. Thus, the series in
question is an alternating series:


1
sin(n/2)  (1)n1 
(1)n1 bn , bn =
=
=
.
n
2n 1
2n 1
n=1
n=1
n=1
The sequence {bn } is monotonically decreasing and bn 0 as n .
So the series converges by the alternating series test.
2
Scope of the alternating series test. Theorem 8.18 provides only a su-

cient condition for an alternating series to converge. There are convergent and divergent alternating series that do not satisfy the hypothesis
(i) of Theorem 8.18. As an example of a convergent series, consider the
alternating series



n



(1)n1
1
n1
n1 1 (1)
(1) bn =
(1)
=
+ 2
n2
n2
n
n=1
n=1
n=1

114

8. SEQUENCES AND SERIES

The sequence {bn } is not monotonic because b2k = 0 while b2k+1 > 0.
Nevertheless,
the series
by Theorem 8.10 because the series
 converges

n1
2
2
1/n converge. The former series converges by
(1) /n and
the alternating series test, while the latter is the pseries with p = 2.
On the other hand, consider the alternating series



n



(1)n1 1
n1
n1 1 (1)
(1) bn =
(1)
=
+
n
n
n
n=1
n=1
n=1
Here again the sequence {bn } of nonnegative terms is not monotonic.
If sn is a partial sum of this series, then sn = tn + hn where tn is the
partial sum of the alternating harmonic series, which has been shown
to converge, and hn is the partial sum of the harmonic series, which is
known to diverge. Thus, the sequence {sn } is the sum of convergent
and divergent sequences and, hence, it diverges.
Remark. Hypothesis (i) of Theorem 8.18 may be weakened
(i) bn+1 bn for all n N
for some integer N 1. Indeed,



(1)n1 bn = b1 b2 + b3 bN 1 +
(1)n1 bn
n=1

n=N
N 1

= b1 b2 + b3 bN 1 + (1)



(1)n1 cn ,
n=1


where cn = bn+N 1 . The series (1) bn and (1)n1 cn dier
by a number, and therefore the convergence
of them implies
 of one
n1
the convergence of the other. The series
(1) cn converges by
Theorem 8.18 as cn+1 cn for all n and limn cn = limn bn+N 1 =
0.

n1 p
n /(n + 1) for converExample 8.27. Test the series
n=1 (1)
gence if p < 1.
n1

Solution: Here bn = np /(n + 1) and, for p < 1,


np
np1
= lim np1 = 0.
= lim
n n + 1
n 1 + 1
n
n

lim bn = lim

So hypothesis (ii) of Theorem 8.18 is fullled. However, the monotonicity of {bn } is not obvious. To investigate it, consider the function
f (x) = xp /(x + 1), where x 1. If f (x) monotonically decreases, then
so does the sequence bn = f (n). The condition f  (x) 0 has to be

54. ALTERNATING SERIES

115

veried:
f  (x) =

pxp1 (x + 1) xp
0
(x + 1)2

(p 1)xp + pxp1 0

p x(1 p)

If p 0, this is true as x 1. If 0 < p < 1, then f (x) monotonically decreases for x p/(1 p) and one can always nd an integer
N p/(1p) such that bn+1 < bn for all n > N . So the series converges
for all p < 1.
2
54.1. Estimating Sums of Alternating Series. A partial sum sn of any

convergent alternating series can be used as an approximation of the


total sum s, but this is not of much use unless the accuracy of the
approximation is assessed. The following theorem asserts that the absolute error of the approximation s sn does not exceed the value of
bn+1 .
8.19 (Alternating Series Sum Estimation). If s =
 Theorem
n1
(1) bn is the sum of an alternating series that satises
(i)

0 bn+1 bn for all n

and

(ii)

lim bn = 0,

then |s sn | bn+1 .
Proof. In the proof of the alternating series test, it was found that
the subsequence {s2k } approaches the limit value s from below, s2k s.
On the other hand, the subsequence {s2k1 } approaches the limit value
s from above (see Fig. 8.8). Indeed, s1 = b1 , s3 = s1 b2 + b3 s1
because b3 b2 , and, in general, s2k+1 = s2k1 b2k +b2k+1 s2k1 ; that
is, {s2k+1 } is monotonically decreasing. This shows that the sequence
of partial sums sn oscillates around s so that the sum s always lies
between any two consecutive partial sums sn and sn+1 as depicted in
Figure 8.8. Hence,
|s sn | |sn+1 sn | = bn+1 .
2
Example 8.28. Estimate the number of terms in a partial sum
sn needed to approximate the sum of the alternating harmonic series
correct within the absolute error not exceeding 10N .
Solution: Here, bn = 1/n. Hence, the approximation s sn has the
needed accuracy if |s sn | bn+1 10N or 1/(n + 1) 10N or
2
n 10N 1.

116

8. SEQUENCES AND SERIES

If the monotonicity condition bn+1 bn holds only if n N , the


conclusion of Theorem 8.19 also holds only if n N
. Indeed, in the notation from Remark above Example 8.27, put t = (1)n1 cn , where
cn = bn+N 1 . Let tn be a partial sum for the series (1)n1 cn . Then
s = sN 1 + (1)N 1 t and sn = sN 1 + (1)N 1 tnN +1 for n N .
Therefore,
|s sn | = |t tnN +1 | cnN +2 = bn+1

for all n N.

54.2. Exercises.

In (1) and (2), prove the convergence of the series and nd its sum.
1 1 1
1
1
(1) 1 + + +

+
2 4 8 16 32
3 5 7
(2) 1 + +
2 4 8
In (3)(21), determine whether the series converges or diverges (here p
is real).


(1)n
(3)
ln(n + 3)
n=1

(6)
(9)

(1)n n
(n3/2 + 1)2/3

n=1

(12)
(15)
(18)

n=2


n=1

(7)
p

(1)n
n1+1/n n
(1)n
n+p

n=1


n=2

(10)
(13)

(16)

(ln n)p

(8)

n=1

(1) sin

n=1

n=1

ln n
sin(n/4)
n

n4/5
(1)n n3
2n
(11)

(1)n ( n n 1)n

n=1

(14)


(1)n n2
n=1

(1)n
np
(19)

(17)

n=1


n=1

(21)

n4 + 1

(1)n

n=1

(1)
n (1)n


cos(n/2)
n=1

(5)


(1)n n
n=2

(1) (ln n)
n

n=1

(20)


(1)n n

(4)
n3 + 1
n=1

nn
n!

(1)n (n2 + n + 1)
(2n + 3)2

(1)n sin2 (n/2)


n


(1)n

n
n
n=1

In (22)(24), nd n for which the approximation by partial sums s sn


is correct within an error not exceeding 10N .
(22)


(1)n1

4
n
n=1

(23)


(1)n n
n=1

10n

(24)


(1)n n1/3
n=1

n2/3 + 6

55. RATIO AND ROOT TESTS

117

(25) Prove that the sum of the alternating harmonic series is


(1)n1
= ln 2.
n
n=1
Hint: Show that a partial sum of the alternating harmonic series is
s2n = h2n hn , where hn = an + ln n and the sequence {an } is dened
in Exercise 52.4.16. Then use the result of the latter exercise to prove
that sn ln 2 as n .
55. Ratio and Root Tests
55.1. Absolutely Convergent Series.


Definition 8.7 (Absolute Convergence). A series
an is called

absolutely convergent if the series of absolute values |an | is convergent.

The absolute convergence is stronger than convergence, meaning


that there are convergent series that do not
For
converge absolutely.
n1
example, the alternating harmonic series
an , an = (1) /n, is
convergent, but not absolutely convergent because the series
 of absolute
1/n, which is
values |an | = 1/n is nothing but the harmonic series
divergent (as a pseries with p = 1). On the other hand, the absolute
convergence implies convergence.
Theorem 8.20 (Convergence and Absolute Convergence). Every
absolutely convergent series is convergent.
Proof. For any sequence {an }, the following inequality holds;
0 an + |an | 2|an |


bn , where
because |an | is either an or an . It shows that the series 
2|an | =
bn= an + |an |, converges
 by the comparison test because
converges
if
a
converges
absolutely.
Hence,
the
series
2 |an | 
n

an =
bn |an | converges as the dierence of two convergent
series.
2

Example 8.29. Test the series [sin n 2 cos(2n)]/n3/2 for absolute convergence.
Solution: Making use of the inequality |A + B| |A| + |B| and the
properties that | sin x| 1 and | cos x| 1, one infers
| sin n 2 cos(2n)|
| sin n| + 2| cos(2n)|
3

3/2 .
|an | =
3/2
3/2
n
n
n

The series of absolute values
|an | converges by comparison with the

convergent pseries 3 n3/2 (here p = 3/2 > 1). So the series in
question converges absolutely.
2

118

8. SEQUENCES AND SERIES


Definition 8.8 (Conditional Convergence). A series an is called
conditionally convergent if it is convergent but not absolutely convergent.
Thus, all convergent series are separated into two classes of conditionally convergent and absolutely convergent series. The key difference between properties of absolutely convergent and conditionally
convergent series is studied in Section 56.
55.2. Ratio Test.


Theorem 8.21 (Ratio Test). Given a series
an , suppose the
following limit exists:
a 
 n+1 
lim 
 = c,
n an
where c 0 or c = .

If c < 1, then  an converges absolutely.
If c > 1, then
an diverges.
If c = 1, then the test gives no information.
Proof. If c < 1, then the existence of the limit means that, for
any > 0, there is an integer N such that
a 
a 
 n+1 
 n+1 
< 
 c < = 
 < c + = q < 1 for all n N.
an
an
Note that since c is strictly less than 1, one can always take > 0
small enough so that the number q = c + < 1. In particular, put
n = N + k 1, where k 2. Applying the inequality |an+1 | < q|an |
consecutively k times,
|aN +k | < q|aN +k1 | < q 2 |aN +k2 | < < q k |aN | = |aN |q N q N +k .
This shows that
|an | < q n , = |aN |q N , for all n N.

The series 
|an | converges
 nby comparison withthe convergent geon
an converges absometric series
q = q because q < 1. So
lutely. If c > 1, then there is an integer N such that |an+1 |/|an | > 1 or
|an+1 | > |an | 0 for all n N . Hence, the necessary condition for a
series
to converge, an 0 as n does not hold; that is, the series

an diverges. If c = 1, it is sucient, to give examples of aconvergent
and divergent series for which c = 1. Consider a p-series
np . One
has
a 
np
1
 n+1 
= lim
=1
c = lim 
 = lim
p
n an
n (n + 1)
n (1 + 1/n)p
for any p. But a p-series converges if p > 1 and diverges otherwise. 2
(8.13)

55. RATIO AND ROOT TESTS

119

8.30. Find all values of p and q for which the series


Example
p n
n
q
converges
absolutely.
n=1
Solution: Here an = np q n . One has
a 
(n + 1)p |q|n+1
(1 + 1/n)p
 n+1 
=
|q|
lim
= |q|.
c = lim 
 = lim
n an
n
n
np
|q|n
1
So, for |q| < 1 and any p, the series converges absolutely by the ratio
test. If q = 1, the ratio test is inconclusive,and these
have
be
cases
 to p
p
n = 1/n ,
studied by dierent means. If |q| = 1, then
|an | =
which is a p-series that converges if p > 1 or p < 1. Thus, the series
converges absolutely for all p if |q| < 1 and for p < 1 if q = 1. Note
that, for 1 p < 0 and q = 1, the series conditionally converges
(i.e., it is convergent but not absolutely
convergent). In this case, it is

2 The ratio
a convergent alternating p-series (1)n /np .
test is advantageous to test convergence of series whose terms contain
products of numbers, e.g., factorials n! = 1 2 3 n.

n
Example 8.31. Test the series
n=0 x /n! for convergence where,
by denition, 0! = 1
Solution: By the ratio test
|an+1 |
|x|n+1 n!
|x|
= lim
= lim
=0
n
n |an |
n (n + 1)! |x|
n n + 1
lim

The series converges for all real x. Later it will be shown that the sum
2
of the series is the exponential function ex .
55.3. Root Test.

Theorem 8.22 (Root Test). Given a series


lowing limit exists:

lim n |an | = c,

an , suppose the fol-

where c 0 or c = .

If c < 1, then  an converges absolutely.
If c > 1, then
an diverges.
If c = 1, then the test gives no information.
Proof. If c < 1, then, as in the proof of the ratio test, the existence
of the limit means that there is an integer N and a number q, c < q < 1,
such that

n
|an | < q |an | < q n for all n N.

120

8. SEQUENCES AND SERIES


This shows that the series |an | converges by comparison
with the

an
converges
convergent geometric series
q n , 0 < q < 1. So
absolutely. If c > 1, then there exists an integer N such that n |an | > 1
for all n N , and hence the necessary condition
for convergence,

not
hold.
The
series
a
diverges.
If c = 1,
an 0 as n , does
n

n
p
p
n
p
consider a p-series: n = ( n) 1 = 1 by Theorem 8.14. But
a p-series converges if p > 1 and diverges if p < 1. The root test is
inconclusive.
2

Example 8.32. Test the convergence of the series
an , where
2
2
n
an = [(2n + 5)/(3n + 2)] .
Solution: Here |an | = an , and the absolute convergence is equivalent
to the convergence. One has

2
2n2 + 5
2 + 5/n2
= < 1.
lim n |an | = lim
=
lim
n
n 3n2 + 2
n 3 + 2/n2
3
So the series converges.

55.4. Oscillatory Behavior of Sequences in the Root and Ratio Tests. Con-

sider a sequence dened recursively by a1 = 1/2 and

n
1
|an+1 |
1 
n 
an = cn =
an+1 = sin
= sin
.
2
2
|an |
2
2

An attempt to test the absolute convergence of
an by the ratio test
fails because the sequence cn oscillates between 0 and 1/2 and, hence,
does not
 converge. On the other hand, the absolute convergence of the
series an can easily be established by comparison with the convergent
geometrical series:
1
1
1
|an | |an1 | |an2 | n
2
4
2
where the inequality | sin x| 1 has been used. Similarly, the sequence
used in the root test may also exhibit oscillatory behavior and be nonconvergent. For example,


n n

1
1 
n 
n
= cn = |an | = sin
sin
an =
.
2
2
2
2

The series an converges absolutely by comparison with the geometric
series: |an | 2n .
The ratio and root tests, as stated in Theorems 8.21 and 8.22,
assume the existence of the limit limn cn = c, while the above examples show that there are absolutely convergent series for which this

55. RATIO AND ROOT TESTS

121

hypothesis is not fullled. What can be said about the absolute convergence of a series when this limit does not exist? To answer this question, recall that, in the proof of the ratio or root test, the existence of
limn cn = c < 1 has been used only to establish the boundedness of
the sequence cn q < 1 for all n N , which is sucient for the series

an to converge. But the boundedness property does not imply the
convergence! Evidently, the boundedness condition holds in the above
examples, cn 12 < 1 for all n > 1, while the sequence {cn } does not
converge. Similarly, the existence of the limit value c > 1 has only
n to
been used to show that n |an | 1 or |an | 1 for innitely many
an
conclude that the sequence {an } cannot converge to 0 and hence
diverges. If |an+1 |/|an | 1 for all n N , then again {an } cannot
converge to 0 (by the proof of the ratio test). Thus, the hypothesis of
convergence of {cn } in the root or ratio test can be weakened to obtain
wider scopes of the tests.
Theorem 8.23 (Ratio and Root Tests Rened). Given a series
n
|an |. Then
n an , put cn = |an+1 |/|an | or cn =

an converges.
cn q < 1 for all n N =

n

|an | 1 for innitely many n
=
an diverges
|an+1 |
1 for all n N
|an |

for some integer N .


55.5. Wider Scope of the Root Test. The dierence in the scopes of the

root and ratio tests is elucidated by the following lemma that is proved
in more advanced calculus courses.
Lemma 8.1. If the limit of |an+1 |/|an | exists, then so does the limit

n
of |an | and in this case
(8.14)
If the sequence

lim

|an | = lim

|an+1 |
.
|an |


n
|an | does not converge, neither does |an+1 |/|an |.

The converse is not true. For example, put an = 2n1 (3(1)n ) >
0. Then


n
n

1
1
n 3 (1)
n 3 (1)
n
|an | = lim
=
lim
=
lim
n+1
n
n
n
2
2
2
2

122

8. SEQUENCES AND SERIES

because (3 (1)n )/2 = 1 for even n and (3 (1)n )/2 = 2 for odd

n, while n p 1 for any p > 0. In contrast, the sequence


|an+1 |
2n+1 3 (1)n+1
1 3 + (1)n
= n+2
=
=
|an |
2
3 (1)n
2 3 (1)n

1 , n even
1
, n odd
4

oscillates between 1 and 1/4 and,


hence, does not converge. So the
convergence of the sequence { n |an |}
does not imply the convergence

n
of {|an+1 /an |} and the sequence { |an |} may converge even if the
sequence {|an+1 /an |} does not. Lemma 8.1 shows that the ratio test has
the same predicting power as the root test only if |an+1 |/|an | converges.
The root test as stated in Theorem 8.23 has wider scope, meaning
that whenever the ratio test shows convergence, the root test does, too,
and whenever the root test is inconclusive, the ratio test is, too. The
subtlety to note here is that the converse of the latter statement is not
generally true. The inconclusiveness of the ratio test does not imply the
inconclusiveness of the root test. The assertion can be illustrated with
the following example. Consider a convergent series obtained from the
sum of two geometric series in which the order of summation is changed:


n=1

an =

1 1
1
1
1
1
+ + 2 + 2 + 3 + 3 +
2 3 2
3
2
3

1 1
1 1
1 1
=
+
=
k
k
2 k=0 2
3 k=0 3
2 1

1
2

1 1
3 1

1
3

3
2

where the sum of a geometric series has been used (Theorem 8.9). Now
note that if n = 2k is even, then a2k = (1/3)k , and a2k1 = (1/2)k if
n = 2k 1 is odd. Take the subsequence of ratios for even n = 2k,
c2k = a2k+1 /a2k = (2/3)k /9. It converges to 0 as k . On the other
hand, the subsequence of ratios for odd n = 2k 1 diverges: c2k1 =
(3/2)k as k . So the limit of cn does not exist; moreover, the
ratio test (as in Theorem 8.23) fails miserably to detect the convergence
The series converges by the root
because cn is not even bounded.

test.

Indeed, c2k = 2k a2k = 1/ 2 < 1 and c2k1 = 2k1 a2k1 = 1/ 3 < 1.

Although
(it oscillates between 1/ 3
the sequence cn does not converge
and 1/ 2), it is bounded, cn q = 1/ 2 < 1 for all n, and hence the
series converges by Theorem 8.23. The ratio test appears to be sensitive
to the order of summation, while this is not so for the root test.

55. RATIO AND ROOT TESTS

123

55.6. When the Ratio Test Is Inconclusive.


Theorem 8.24 (De Morgans Test). Let
an be a series in which
|an+1 |/|an | 1 as n . The series converges absolutely if

a 
 n+1 
lim n 
 1 = b < 1.
n
an

The proof of this theorem is left to the reader as an exercise (see


Exercise 55.7.26). Consider the asymptotic behavior of the ratio cn =
|an+1 |/|an | as n . The theorem asserts that if cn behaves as
n (i.e., neglecting terms of order 1/np where
cn 1 + b/n for large
p > 1), then the series
an converges if b < 1.
For a p-series, the ratio test is inconclusive (see the proof of the
ratio test). However, De Morgans test resolves the inconclusiveness.
Indeed, for large n,
np
1
p
=
1 .
cn =
p
p
(n + 1)
(1 + 1/n)
n
where the asymptotic behavior has been found from the linearization
f (x) = (1 + x)p f (0) + f  (0)x = 1 px for small x = 1/n. So
b = p and the series converges if b < 1 or p > 1.
This illustrates a basic technical trick to applying De Morgans test.
Suppose that there is a function f (x) such that |an+1 |/|an | = f (1/n). If
f is dierentiable at x = 0, then its linearization L(x) = f (0) + f  (0)x
is a good approximation in the sense that
f (x) L(x)
f (x) f (0)
= lim
f  (0) = f  (0) f  (0) = 0
lim
x0
x0
x
x
In other words, f (x) = f (0) + f  (0)x + x(x) where (x) 0 as x 0
(the error of the linear approximation decreases to zero faster than x).
Therefore
f  (0) ( n1 )
f  (0) ( n1 )
|an+1 |
= f (1/n) = f (0) +
+
=1+
+
.
|an |
n
n
n
n
By the inconclusiveness of the ratio test, f (0) = 1. Then




|an+1 |
lim n
1 = lim f  (0) + ( n1 ) = f  (0)
n
n
|an |

and the series
an converges absolutely if f  (0) < 1.
Example 8.33. Investigate the convergence of the series


p(p + 1) (p + n 1)
n=1

n!

124

8. SEQUENCES AND SERIES

Solution: The ratio sequence has the form


|p(p + 1) (p + n 1)(p + n)|
n!
|an+1 |
=
|an |
(n + 1)!
|p(p + 1) (p + n 1)|
p
|1 + n |
|n + p|
=
=
n+1
1 + n1
So it converges to 1 and the ratio test is inconclusive. Consider the
function f (x) = (1 + px)/(1 + x). As a ratio of linear functions, it is
dierentiable at x = 0. By the ratio rule
f  (x) =

p(1 + x) (1 + px)
p1
=
2
(1 + x)
(1 + x)2

f  (0) = p 1 .

By De Morgans test, the series converges (absolutely) if f  (0) < 1 or


p 1 < 1 or p < 0.
2
55.7. Exercises.

In (1)(18), use the ratio or root tests to determine whether the series
is absolutely convergent (here p is real).

(1)
(4)


(1000)2n
n=1


n=1

(7)
(8)
(9)
(10)
(13)

(2)

n!
(n!)2
(2n)!

(5)

 n
2
n
3
n=1


2n n!
n=2

nn

(6)

(3)


n!
,
pn
n=1


pn n!
n=1

nn

100 100 101 100 101 102


+
+
+
1
13
135
4 4 7 4 7 10
+
+
+
2 2 6 2 6 10

3
5
2n+1
2)
( 2 2)( 2 2) ( 2
n=1

 (n!)2


n2
1 n2
(11)
(12)
1
+
n
2n2
(2 + n1 )n
n=1
n=1
n=1
n

 n


2n3 + n n
p + np
(14)
, p>0
3n3 + 5
2n + n2
n=1
n=1

p = 0

55. RATIO AND ROOT TESTS

(15)
(17)


n=1


n=1

np
(ln n)n

(16)

125


2 4 (2n)
n=1

pn n!
5 8 (3n + 2)

(18)

n!
n(n1)


n1
n=2

n+1


2
(19) For which integers p > 0 is the series
n=1 (n!) /(pn)! convergent?
(20) Consider a geometric series with q = 1/2 in which the order of
terms is changed by swapping terms in each consecutive pair:
a1 + a 2 + a 3 + =

1
1 1
1
1
1
1
+1+ + +
+
+
+
+ .
2
8 4 32 16 128 64

Test the convergence of this series using the root and ratio
tests.
In (21)(24), use De Morgans test to investigate the convergence of
the series.
 p  p  135 p
(21) 12 + 13
+ 246 +
24
a(a+c)
a(a+c)(a+2c)
a
(22) b + b(b+c) + b(b+c)(b+2c) + , a > 0 , b > 0 , c > 0

p(p+1)(p+n1)
(23)
n=1

n!nq
p

135(2n1)
(24) n=1 246(2n)
n1q

(25) (Estimating Sums). Given a series
an with positive terms,
put cn = an+1
/a
.
Suppose
that
c

c
< 1, that is, the series
n
 n
converges,
an = s. Let sn be a partial sum. Prove that
s sn

an+1
1 cn+1

if {cn } is a decreasing sequence, and


s sn

an+1
1c

if {cn } is an increasing sequence. Hint: Use the geometric


series as in the proof of the ratio test to estimate the remainder
s sn = an+1 + an+2 + .

(26) Prove De Morgans test. Hints:
|an |
 Compare the series
p
with the convergent p-series
bn , where bn = A/n and p =
(1 b)/2 > 1 if b < 1. Show that n(bn+1 /bn 1) b as
n . Next, show that, by choosing the constant A, one
can always make |an | < bn for all n.

126

8. SEQUENCES AND SERIES

56. Rearrangements
Here characteristic distinct properties of conditionally convergent
and absolutely convergent series are established through the concept
of rearrangement.
Definition 8.9 (Rearrangement). Let {kn }, n = 1, 2, ..., be an
integer-valued positive sequence
appears
 in which every positive integer 
an is
only once. Given a series an , put an = akn . The series
called a rearrangement of
an .
The sequence {kn } is obtained by a permutation of terms of the
sequence of all integers {1, 2, 3, 4, ...}. For example,
{1, 2, 3, 4, 5, 6, 7, 8, 9, ...}

{kn } = {1, 3, 2, 5, 7, 4, 9, 11, 6, ...} .

A rearrangement of a series is accordingly obtained by a permutation of


terms in the sum. For a nite sum, a rearrangement (or a permutation)
of its terms does not change the value of the sum. This is not generally
so for convergent series.
Consider an alternating harmonic series:
(8.15)

an =

n=1


(1)n1
n=1

=1

1 1 1 1 1
+ + + .
2 3 4 5 6

The series is convergent but not absolutely convergent (its sum is s =


ln 2; see Exercise 54.2.25). One of its rearrangements reads
(8.16)


n=1

an = 1 +

1 1 1 1 1 1
1
1
+ + + +
+
3 2 5 7 4 9 11 6

in which two positive terms are always followed by one negative. Let
sn and sn be partial sums of (8.15) and (8.16), respectively. Put hn =
1 + 1/2 + + 1/n (a partial sum of the harmonic series). Then
1 1 1
1
1
+ + +

2 3 4
2n 1  2n

1
1
1 1
1
1 1

1 + + + +
= 1 + + + +
3 5
2n 1 2
2 3
n
1 1
1
1
= h2n
hn
2 4
2n 2
= h2n hn

s2n = 1

56. REARRANGEMENTS

127

In a similar fashion,
1 1 1
1
1 1
1

s3n = 1 + + + + +
3 5 7
4n 1 2 4
2n
1 1
1
1
1
1
= h4n
hn = h4n h2n hn
2 4
4n 2
2
2
1
1
= (h4n h2n ) + (h2n hn ) = s4n + s2n .
2
2
Since the sequence {sn } converges to s, its subsequence {s4n } and {s2n }
also converge to s. By basic limit laws for sequences, the sequence {s3n }
converges and by taking the limit n in the above equality,
lim s3n = lim (s4n + 12 s2n ) = limn s4n + 12 limn s2n

= s + s/2 = 3s/2 .
If the rearranged series (8.16) converges, then the subsequence {s3n }
of the sequence of partial sums {sn } must converge to the sum of the
series and the latter must be s = 3s/2. Thus, a rearrangement of the
series may change its sum! This fact is not specic to the example
considered but inherent in all conditionally convergent series. Terms
of a conditionally convergent series occur with dierent signs (positive
and negative). By regrouping positive and negative terms, it will be
proved that the sum of a conditionally convergent series can be made
any number or . The analysis begins by studying the properties
of sums of positive and negative terms of a conditionally convergent
series.
Given a number x, put x = (x |x|)/2. The number x+ = x if
x > 0 and x+ = 0 otherwise. Similarly, x = x if x < 0 and x = 0
otherwise.

 +
Lemma
8.2.
Given
a
series
a
,
consider
two
series
an and
n

=
(a

|a
|)/2
(the
series
of
positive
and
negative
an , where a
n
n
n
terms).
Then
 +


an and
(i) If an converges absolutely, then
 + an converge.

an and
an diverge.
(ii) If
an is conditionally convergent, then



Proof. Let
an = s < and
|an | = t, where t < if
an
converges absolutely and
t
=

if
it
is
conditionally
convergent.
Let


an , sn be partial sums of
an , and tn be
s
n be partial sums
 of
partial sums of
|an |. Since sn s and tn t as n , one infers
that

1
s+
a+
n an = |an |
n sn = tn
=
= s
= (sn tn ) .
+

n
sn + sn = sn
an + a n = a n
2

128

8. SEQUENCES AND SERIES

If
an converges absolutely, then
 limn sn =
 t < and hence
an is condition(s t)/2; that is, both series
an converge. If


ally convergent, then t = and limn sn = (the series


an
diverge).
2

Theorem 8.25 (Riemanns Rearrangement Theorem). Let
an
be a series that converges, but not absolutely. Then,
for
any
c
that
is
a
 
real number or , there exists a rearrangement
an whose sequence
of partial sums {sn } converges to c.

Proof. Let p1 , p2 , .... denote nonnegative terms of
an in the
order in which they occur, and let q1 , q2 , ... denote negative terms of an
in the 
order in which
In the notation
of Lemma 8.2, the
 +they occur. 

an as well as
qn and a
may
only dier by
series
pn and
n

pn and
qn diverge.
zero terms (if some an = 0). So the series
Consider the following rearrangement. Given a number c, take rst
k1 terms pn , such that the number c lies between the partial sums
sk1 = p1 + p2 + + pk1 and sk1 1 ; that is, k1 is dened by the condition sk1 pk1 < c < sk1 or |c sk1 | < pk1 . If c < 0, then skip this
rst step. Next, take rst m1 terms qn where m1 is the smallest integer
such that sk1 +m1 = sk1 + q1 + + qm1 < c; that is, m1 is dened by the
condition sk1 +m1 + qm1 > c > sk1 +m1 or |c sk1 +m1 | < |qm1 |. This can
always be done because partial
sums of
pn can be larger than any
number, while partial sums of
qn can be smaller than any number
owing to the divergence of these series. So
s1 sn sk1 , 1 n k1 ,

where |c sk1 | < pk1 ,

sk1 sn sk1 +m1 , k1 n k1 + m1 ,

where |c sk1 +m1 | < |qk1 |,

Next, take k2 next terms pn , where k2 is the smallest integer such that
sk1 +m1 +k2 > c, and take m2 next terms qn , where m2 is the smallest
integer for which sk1 +m1 +k1 +m2 < c, and so on. At the nth step of the
procedure, let n1 be the integer for which the last term in sn1 is pkn
and let n2 be the integer for which the last term in sn2 is qmn , that
is, n2 = n1 + mn . The partial sums of the constructed rearrangement
oscillate about c, reaching local minima sn1 and local maxima sn2 :
sn1 sn sn2 ,

n1 n n2 ,

|c sn1 | < pkn , |c sn2 | < |qmn |.



By convergence of the series
an , an 0 as n 0. Hence, pn and qn
also converge to 0 and so do the subsequences pkn 0 and qmn 0.
Thus, all local maxima and minima of the sequence of partial sum {sn }
converge to c by (8.17), which shows that sn c. Finally, if c = ,
(8.17)

56. REARRANGEMENTS

129

one can take any divergent sequence cn (or ) and construct a


rearrangement such that sk1 overshoots c1 and sk1 +m1 undershoots c1 ,
sk1 +m1 +k2 overshoots c2 and sk1 +m1 +k2 +m2 undershoots c2 , and so on.
Obviously, this sequence of partial sum diverges.
2
Absolutely convergent series have a drastically dierent property.
Theorem
8.26 (Rearrangement and Absolute Convergence).If a

an
series
an converges absolutely, then every rearrangement of
converges, and they all converge to the same sum
Proof. Let tm = |a1 | + |a2 | + + |am | be a partial sum of the
series of absolute values. The sequence {tm } increases monotonically
converging to a number t by the hypothesis; that is, for any > 0,
there is an integer N such that t tm < for all m > N . Therefore,
n

|ak | = tm tN +1 = |tm t + t tN +1 |
k=N +1

|tm t| + |t tN +1 | < 2.
So, by taking N large enough, the sum of any number of terms |ak |,
k > N , can be made smaller thanany preassigned positive number.
 
an and its rearrangement
an .
Let sn and sn be partial sums of

One can take n > N large enough such that sn contains a1 , a2 ,...,aN
(i.e., the integers 1, 2, ..., N are in the set of integers k1 , k2 , ..., kn in
the notations of Denition 8.9). Then the dierence sn sn contains
only terms ak with k > N (the terms a1 , a2 ,..., aN are cancelled). Let
m be the largest integer amongst {k1 , k2 , ..., kn } such that am is not
cancelled in sn sn . Then the dierence contains some of the terms
aN +1 , aN +2 , ..., am with possibly reversed sings (the terms of sn that
are not cancelled contribute to sn sn with an opposite sign). Applying
the inequality |b c| |b| + |c| to the sum of all terms in sn sn , it is
concluded that
m

|ak | < 2 .
|sn sn |
k=N +1
If sn


for all n exceeding some integer.


s and sn s as n , then

2
|s s| < 2, which shows that s = s because > 0 is arbitrary.
Thus, an absolutely convergent series is much like a nite sum. The
sum does not depend on the order in which the summation is carried
out. In contrast, the sum of a conditionally convergent series depends
on the summation order. This is the characteristic dierence between
these two classes of convergent series.

130

8. SEQUENCES AND SERIES

56.1. Strategy for Testing Series. It would not be wise to apply tests for

convergence in a specic order to nd one that nally works. Instead,


a proper strategy, as with integration, is to classify the series according
to its form. One should also keep in mind that a conclusion about the
convergence of a series can be reached in dierent ways.
1. Necessary condition for convergence. It is is always easier
to check rst
 the condition an 0 as n than it is to investigate
the series
an for convergence. If the condition does not hold, the
series diverges.

2. Special series. A series
an coincides with (or is a combination of or is equivalent to) special series such as a p-series, alternating
p-series, geometric series, telescopic series, and so on. Their convergence properties are known.

3. Series similar to special ones. If a series
an has a form
that is similar to one of the special series, then one of the comparison
tests should be considered. For example, if an is a rational or algebraic function (contains roots of polynomials), then the series should
be compared with a p-series.
4. Alternating series. If an = (1)n bn , bn 0, then the alternating series test is an obvious possibility.
5. Ratio and root tests. Absolute convergence implies convergence. So, if the ratio or root test shows convergence, then the series in
question converges absolutely. If these tests show divergence, then the
series in question may still converge but not absolutely, and a further
investigation
is required. The root test is convenient for series of the

form (bn )n . The ratio test is convenient when an involves the factorial n! or similar products of integers. The root test has a wider scope,
but it is more dicult to use. The ratio test is often inconclusive if an
is a rational or algebraic function (cn = |an+1 |/|an | 1). In this case,
the asymptotic behavior of cn is rather easy to nd, cn 1 + b/n as
n , and then use De Morgans test.
6. Series
of nonnegative terms. If an = f (n) 0 and the

integral 1 f (x) dx is easy to evaluate, then the integral test is eective.
Also, it can
 be used in combination with the
 comparison test: an
f (n) and 1 f (x) dx converges and so is
an , or f (n) an and


f
(x)
dx
diverges
and
so
is
a
.
n
1

Example 8.34. Test the series (n + 1)/(n2 + n + 1) for convergence.
Solution: For large n, the leading terms of the top and bottom of
the ratio are n and n2 , respectively. So an 1/n asymptotically for

56. REARRANGEMENTS

131

large n. The series resembles the harmonic series, which diverges. It is


natural, then, to try to prove the divergence of the series by comparing
it with the harmonic series:
1
n
n
n+1
=
>

.
n2 + n + 1
n2 + n + 1
n2 + n 2 + n2
3n
Thus, the series indeed diverges by comparison with the harmonic series.
2
 n
Example 8.35. Test the series
3 /(2 4 6 (2n)) for convergence.
Solution: Each term an involves a factorial-like product of integers,
which suggests the use of the ratio test:
an+1
2 4 (2n)
3n+1
3
0.
=
=
n
an
2 4 (2n) (2n + 2)
3
2n + 2
So, the series converges.
2


Example 8.36. Test the series
sin(n2 )e n for convergence.
Solution: One has
The series

1

|an | = | sin(n2 )|e

converges by the integral test:






x
u
u 
e
dx = 2
ue du = 2ue  2
1

eu du

2 2
4
= + = <
e e
e
Here the change of variables has been carried out rst, x = u2 , dx =
2udu. Then the integration by parts has been done to evaluate the
improper integral. Hence, the series in question converges by the comparison test.
2
(a partial sum of
Example 8.37. Put hn = 1 + 1/2 + + 1/n 
the harmonic series). Investigate the convergence of
np eqhn , where
p = q.
Solution: The ratio test is inconclusive:
q
an+1
(n + 1)p eqhn+1
(n + 1)p q(hn+1 hn ) (1 + n1 )p n+1
=
=
e
=
e
an
np eqhn
np
1
p
p


q
q
1
1
lim 1 +
e n+1 = lim 1 +
lim e n+1 = 1p e0 = 1 .
n
n
n
n
n
To apply De Morgans test, the asymptotic behavior of an+1 /an has
to be investigated. Put f (x) = (1 + x)p exp(qx/(1 + x)) so that

132

8. SEQUENCES AND SERIES

an+1 /an = f (1/n). The linearization of f (x) at x = 0 may be obtained


by calculating f  (0), which is technically involved given the explicit
form of f (x). Instead, one can linearize the factors in f (x). Let g(x) =
(1 + x)p . Then g  (x) = p(1 + x)p1 and, hence, g(x) g(0) + g  (0)x =
1 + px near x = 0. For small u, equ 1 qu. Setting u = x(1 + x)1
and using the linearization (1 + x)1 1 x, one nds
equ 1 qu 1 qx(1 x) 1 qx
by keeping only terms linear in x. Therefore
f (x) (1 + px)(1 qx) 1 + (p q)x

pq
an+1
1+
.
an
n

Thus, the series converges if p q < 1 or p < q 1.

56.2. Exercises.

In (1)(19), test the series for convergence or divergence (here p is real).

(1)
(4)
(6)


n=1


n=1


n=1

(8)
(11)
(14)
(16)
(18)

n + 2n

n
(2)
(1)
2n + 3
n=1
n

n!
2 5 8 (3n 1)
(2n + 3)n
(3n2 + 1)n/2

(7)

(1)n

tan(n + 1/n) (9)

n=1


n=1

n=2

n!

np

n=2

n=1

n=1


(1)n ln n

(5)

(3)


np p2n

1 3 5 (2n 1)
2 4 6 (2n)

1
(ln n)ln n

(10)


sin(1/n)
n=1



n n2
, p > 0 (12)
( n p2 1)n
n+p
n=1

np


n!
(13)
enp
n=1


npn1

n
,
|p|
<
1
(15)
(
p 1) , p 0
pn (1 1/n)n
n=1

n=1


n=1


n=1

(1)n
p+n
(a

1/n

(17)

(1)n

n=1
1/n

) , a > 0, b > 0

n1
n+1
(19)

100


n!

n n
n=1

57. POWER SERIES

133

In
(21), use the sum of the alternating harmonic series
(20) and
n1
(1)
/n
= ln 2 to nd the sums of its rearrangement.
n=1
1 1 1 1 1
+ + +
3 2 5 7 4
1 1 1 1 1
(21) 1 + +
2 4 3 6 8
In (22) and (23), nd two rearrangements of the conditionally convergent series that converge to + and to .
(20)

(22)

1+


(1)n1
n=1


(1)n1

n
n=1

(23)

(24) Let all positive terms of the alternating harmonic series be divided into groups of p terms in the order in which they appear
in the series and let all negative terms be divided into groups
of q terms in the order in which they appear. Consider the
rearrangement in which the rst group of positive p terms is
followed by the rst group of negative q terms, which, in turn,
is followed by the second group of p positive terms and the
latter is followed by the second group of q negative terms, and
so on. If the sum of the alternating harmonic series is ln 2,
prove that the sum of this rearrangement is ln 2 + 12 ln(p/q).
(25) Let {an }
1 be the sequence of positive roots of the equation
tan x =
increasing order. Test the convergence of the
xin the2
(a
)
.
series
n=1 n

(26) If
an is conditionally convergent, show that
s+
n
= 1,
n s
n
lim

1
(|ak | ak ).
2 k=1
n

s
n =

57. Power Series


Definition 8.10 (Power Series). Given a sequence {cn }, the series

c n xn = c 0 + c 1 x + c 2 x2 + c 3 x3 +

n=0

is called a power series in the variable x. The numbers cn are called


the coecients of the series.
In general, the series will converge or diverge, depending on the
choice of x. The power series always converges for x = 0 to the number c0 .

134

8. SEQUENCES AND SERIES

Example 8.38. For what values of x does the power series


converge?


n=1

xn /n

Solution: By the root test,



n
|x|
n |x |
|x| as n .
=
n
n
n
So the series converges for all 1 < x < 1 and diverges as x > 1
or x < 1. The root test is inconclusive for x = 1. These values
have to be investigated by dierent
means. For x = 1, the power series

becomes the harmonic series
1/n, which is divergent. For
 x = 1,
the power series becomes the alternating harmonic series (1)n /n,
which is convergent. Thus, the power series converges if x [1, 1)
and diverges otherwise.
2
Given a number a, consider a power series in the variable y = x a:



n
cn y =
cn (x a)n .
n=0

n=0

It is also called a power series centered at a or a power series about a.


Let S be the set of all values of x for which a power series in x converges and let Sa be the set of all values of x for which the corresponding
power series in (x a) converges. What is the relation between S and
Sa ? Since the series are obtained from one another by merely shifting
the value of the variable by a number a, x x a, the set Sa is
therefore obtained by adding the number a to every element of S:
x Sa

x a S = Sa = {x | x a S}.

n
For example, the series
n=0 (x 2) /n converges if x 2 [1, 1)
or x [1, 3) and diverges otherwise by Example 8.38. Thus, the problem of nding the set Sa is equivalent to the problem of nding the
set S.
(8.18)

57.1. Power Series as a Function. Suppose that a power series in x

converges on a set S. Then it denes a function on S:



cn xn , x S.
f (x) =
n=0

The set S is called the domain of such a function. Functions dened


by power series are most common in applications. In what follows, it
will be shown that familiar elementary functions such as sin x, cos x,
and exp x, etc can also be represented as power series. There are many
other (special) functions that are dened by power series.

57. POWER SERIES

135

Example 8.39. Find the domain of the Bessel function of order 0


that is dened by the power series


(1)n 2n
x ,
J0 (x) =
2n (n!)2
2
n=0
where, by common convention, 0! = 1.
Solution: Since an = cn x2n contains the factorial, the ratio test is
more convenient:
|cn+1 |
22n (n!)2
x2
|an+1 |
=
0
= x2
= x2 2(n+1)
|an |
|cn |
2
((n + 1)!)2
22 (n + 1)2
as n . So the series converges for all x.
2
Values of a function dened by a power series can be estimated by
partial sums that are polynomials in the variable x:
n

c k xk = c 0 + c 1 x + c 2 x2 + + c n xn .
f (x) fn (x) =
k=0

Thus, partial sums dene a sequence of polynomials that converges to


the function on S, fn (x) f (x) for all x S. The accuracy of
the approximation is determined by the remainder Rn (x) = f (x)
fn (x). The accuracy assessment is discussed in Section 8.59. Since the
remainder Rn (x) is a function on S, the error of the approximation is
not generally uniform; that is, it depends on x. In this regard, recall
also a useful result given in Exercise 55.7.25
57.2. Radius of Convergence. The set S on which a power series is
convergent is an important characteristic and its properties have to be
studied.

8.3 (Properties of a Power Series). (i). If a power series


 Lemma
n
cn x converges when
x = nb = 0, then it converges whenever |x| < |b|.
(ii). If a power series
cn x diverges when x = d = 0, then it diverges
whenever |x| > |d|.
 n
Proof. If
cn b converges, then, by the necessary condition for
convergence, cn bn 0 as n . This means, in particular, that, for
= 1, there exists an integer N such that |cn bn | < = 1 for all n > N .
Thus, for n > N ,
 c bn xn 
 x n  x n

 n
 
 
|cn xn | =  n  = |cn bn |  <   .
b
b
b

n
which shows that
the series
cn x converges by comparison with the
n
geometric series
q , where q = x/b and |x/b| < 1 or |x| < |b|.

136

8. SEQUENCES AND SERIES


Suppose
cn dn diverges. If x is any number such that |x| > |d|,
 that
n
because, by part (i)
of the lemma, the
then
cn x cannot
 converge
n
convergence
of
cn x implies the convergence of
cn dn . Therefore,

n
2
cn x diverges.
This lemma allows us to establish the following description of the
set S.
Theorem8.27 (Convergence Properties of a Power Series). For a
power series
cn xn , there are only three possibilities:
(i) The series converges only when x = 0.
(ii) The series converges for all x.
(iii) There is a positive number R such that the series converges if
|x| < R and diverges if |x| > R.
Proof. Suppose that neither case 1 nor case 2 is true. Then there
are numbers b = 0 and d = 0 such that the power series converges for
x = b and diverges for x = d. By Lemma 8.3, the set of convergence
S lies in the interval |x| |d| for all x S. This shows that |d| is an
upper bound for the set S. By the completeness axiom,
a least
 S has
n
upper bound R = sup S. If |x| > R, then x S, and
cn x diverges.
If |x| < R, then |x| is not an upper bound for S,
and
there exists a

n
number b S such that b > |x|. Since b S,
cn x converges by
Lemma 8.3.
2
Theorem 8.27 shows that a power series converges in a single open
interval (R, R) and diverges outside this interval. The set S may or
may not include the points x = R. This question requires a special
investigation just like in Example 8.38. So the number R is characteristic for convergence properties of a power series.
Definition 8.11 (Radius
of Convergence). The radius of conver
gence of a power series cn xn is a positive number R > 0 such that the
series converges in the open interval (R, R) and diverges outside it.
A power series is said to have a zero radius of convergence, R = 0, if it
converges only when x = 0. A power series is said to have an innite
radius of convergence, R = , if it converges for all values of x.
The ratio or root test can be used to determine the radius of convergence.

57. POWER SERIES

137

Corollary8.3 (Radius of Convergence of a Power Series). Given


a power series
c n xn ,
if
if

|cn+1 |
=
n |cn |

lim n |cn | =
lim

=
=

1
,

1
R= ,

R=

where R = 0 if = and R = if = 0.
Proof. Put an = cn xn in the ratio test (Theorem 8.21). Then
|an+1 |/|an | = |x||cn+1 |/|cn | |x|. The series converges if |x| < 1,
which
shows that
R = 1/. Similarly, using the root test (Theorem
n
8.22), |an | = |x| n |cn | |x| < 1, which shows that R = 1/.
2
Remark. If the sequences in Corollary 8.3 do not converge, then
Theorem 8.23 from Section 55.4 should be applied to an = cn xn to
determine the radius of convergence. Also, one should keep in mind
that the root test has wider scope (recall Lemma 8.1 in Section 55.5)
Once the radius of convergence has been found and 0 < R < ,
the cases x = R have to be investigated by some other means (as the
root or ratio test is inconclusive in this case) to determine the interval
of convergence S of a power series.
Example 8.40. Find the radius
interval of
 ofn convergence and the
n
convergence of the power series
cn x , where cn = (q) / n + 1 and
q > 0.
Solution:
|cn+1 |
q n+1
=
|cn |
n+2

n+1
=q
qn


n+1
=q
n+2

1 + 1/n
q = .
1 + 2/n

n+1=
Therefore, R = 1/ = 1/q. If x = 1/q, then cn xn = (1)n / 
(1)n bn . The sequence bn converges monotonically to 0 so that (1)n bn
converges
by the
= 1/q, then cn xn =
alternating series test.If x 1/2

1/ n + 1 >
1/
2n, n 1. The p-series
1/n diverges (p = 1/2 <
1) so that
1/ n + 1 diverges by the comparison test. Thus, the
interval of convergence is S = [1/q, 1/q).
2
Example 8.41. Find the radius
and the interval of
 2of convergence
convergence of the power series
n (x + 1)n /q n , where q > 0.

Solution: Put y = x + 1. If S is the interval of convergence of cn y n ,
where cn = n2 /q n , then the interval of convergence in question is obtained by adding 1 to all numbers in S according to the rule (8.18).

138

8. SEQUENCES AND SERIES

By Corollary 8.3,

1
1
1
n
n2 = ( lim n n)2 = = .
lim n |cn | = lim
n
n q
q n
q

So R = 1/ = q. If y = q, then cn y n = n2 , and the series n2 diverges
(an = n2 does not converge to 0). If y = q, then cn xn = (1)n n2 ,
and the series diverges because an = (1)n n2 , does not converge to 0.
The series converges only if |y| = |x + 1| < q, and hence the interval of
convergence is x (q 1, q 1) (the interval (q, q) shifted by 1).
2
57.3. Exercises.

In (1)(19), nd the radius of convergence and the interval of convergence of the power series.
(1)

3 n

nx


2n

(2)

n=0

(4)
(6)
(8)
(10)
(12)

n=2

n=0

(16)
(18)

2n

n(x + 1)

n=1


n=1

n=1

2

nn + 1
(1) 3
(x 1)n
n +3
n=1

(7)

(9)

n=1

2 2n

nx
2 4 (2n)
4n
(x + 3)n
n!

xn
1 3 5 (2n 1)


(n!)k n
(11)
x , k > 0 (integer)
(kn)!
n=0
n2


1
1+
(13)
xn
n
n=1


p(p 1) (p n + 1)

n!

n=1

xn
, p > 0,
n
p
n=1


(1)n
n n
n=1

n!

(3)

n
( 2 1)n x2n


1
(5)
(x 2)n , p > 0
p
n
n=1

1
xn
ln n

(4x + 1)n
n2

n=0

(14)

n=1


(1)n

n3

(15)


n=1

(17)


n=1

xn

(19)

xn
a2n + b2n

2 4 (2n)
xn
3 5 (2n + 1)


[3 + (1)n ]n
n=1

xn

(20) Let p < q be real numbers. Give examples of power series


whose intervals of convergence are (p, q), [p, q], (p, q], and [p, q).

58. REPRESENTATION OF FUNCTIONS AS POWER SERIES

139

(21) The Airy function is dened by the power series


x3
x6
x9
+
+
+ .
23 2356 235689
Find its domain.
(22) A function f is dened by the power series
A(x) = 1 +

f (x) = p + qx + px2 + qx3 + px4 + qx5 + ;


that is, its coecients c2k = p and c2k1 = q, where p and q
are real. Find the domain of f and an explicit expression of
f (x) (the sum
 of nthe series).
(23) If f (x) =
cn x , where cn+4 = cn for all n 0, nd the
domain of f and 
a formula forf (x).
bn xn have the radii of conver(24) The power series
cn xn and
and R2 , respectively. What is the radius of convergence R1
gence of (cn + bn )xn ?

(25) Suppose that the radius of convergence
of
cn xn is R. What

kn
is the radius of convergence of
cn x , where k > 0 is an
integer?
58. Representation of Functions as Power Series
Consider a power series
1 x2 + x4 x6 + x8 + =


(1)n x2n .
n=0

It is a geometric series with q = x , and therefore it converges for


all |q| = x2 < 1 or x (1, 1). Using the formula for the sum of a
geometric series, one infers that
2


1
2
4
=
1

x
+
x
+

=
(1)n x2n
1 + x2
n=0

for all

1 < x < 1,

This shows that the function 1/(1 + x2 ) can be represented as a power


series in the open interval (1, 1). Note that this representation is valid
only in the interval of convergence of the power series despite the fact
that the function 1/(1 + x2 ) is dened on the entire real line.
In general, one can construct a representation of a function by a
power series in (x a) for some a. The interval of validity of this
representation depends on the choice of a.
Example 8.42. Find a representation of 1/x as a power series in
(x a), a > 0, and determine the interval of its validity.

140

8. SEQUENCES AND SERIES

Solution: Put y = x a. The function can be rewritten in a form


that resembles the sum of a geometric series:
1
1 
y n  (1)n
1

=
(x a)n ,
=
=
n+1
x
a(1 + y/a)
a n=0
a
a
n=0

x (0, 2a).

The geometric series converges if |q| = | y/a| = |y|/a < 1, and hence
this representation is valid only if a < y < a or a < x a < a or
0 < x < 2a.
2
58.1. Differentiation and Integration
of Power Series. The formula for


the sum of a power series


cn xn is often complicated and, in most
cases, cannot even be found explicitly. How can functions dened by a
power series be dierentiated and integrated? If a function is a nite
sum f (x) = u1 (x) + + un (x), then the derivative is the sum of



derivatives
 f = u1 + + un and, similarly, the integral is the sum of
integrals f dx = u1 dx + + un dx. This is not generally true
for innite sums. As an example, consider a function dened by the
series



sin(nx)
un (x) =
.
f (x) =
2
n
n=1
n=1

By comparison with a p-series, |un


(x)| = | sin(nx)|/n2 1/n2 , this
series converges for all x because
1/n2 converges. If the series is

dierentiated just like a nite
  sum, that is, term-by-term, un (x) =
cos(nx)/n, then the series  un (x) diverges for x = 2k for any integer
k as the harmonic series
1/n. So f  (2k) does not exist. Thus,
although the terms un (x) are
  of
 dierentiable functions in the interval
the series of derivatives
un (x)
convergence of the series
un (x), 
may not converge and hence f (x) = un (x) may not be dierentiable
everywhere in its domain.

un (x) is a power
It appears that if un (x) = cn (x a)n , that is,
series, then the term-by-term dierentiation or integration is justied.
A proof of this assertion is beyond the scope of this course.
Theorem 8.28 
(Dierentiation and Integration of Power Series).
If the power series
cn (x a)n has a nonzero radius of convergence
R > 0, then the function f dened by
f (x) = c0 + c1 (x a) + c2 (x a) + =
2


n=0

cn (x a)n

58. REPRESENTATION OF FUNCTIONS AS POWER SERIES

141

is dierentiable (and therefore continuous) on the interval (aR, a+R)


and



2
ncn (x a)n1 ,
f (x) = c1 + 2c2 (x a) + 3c3 (x a) + =

f (x) dx = C + c0 (x a) + c1

n=1

(x a)
(x a)n+1
cn
+ =C +
.
2
n
+
1
n=0
2

The radii of convergence of these power series are both R.


Thus, for power series, the dierentiation or integration and the
summation can be carried out in any order:
 d
d 
cn (x a)n =
[cn (x a)n ],
dx
dx




n
[cn (x a)n ] dx.
cn (x a) dx =
Remark. Theorem 8.28 states that the radius of convergence of
a power series does not change after dierentiation or integration of
the series. This does not mean that the interval of convergence does
not change. It may happen that the original series converges at an
endpoint, whereas the dierentiated series diverges there.
Example
Find the intervals of convergence for f , f  , and f 
 8.43.
n
if f (x) = n=1 x /n2 .

n
Solution: Here cn = 1/n2 and hence n |cn | = 1/ n2 = (1/ n n)2
1 = . So the radius 
of convergence is R = 1/ = 1. For x = 1,
the series is a p-series
1/n2 that converges (p = 2 > 1). Thus, f (x)
is dened on the closed
x [1, 1]. By Theorem 8.28, the
 interval
n1
n2
x
/n
and
f  (x) =
/n have
derivatives f  (x) =
n=1
n=2 (n1)x

the same radius of convergence R = 1. For x = 1, the series f (1) =

harmonic series that converges, whereas
(1)n1 /n is the alternating


n
the series f (1) = (1) (n 1)/n diverges because the sequence of
its terms does not converge to
0: |(1)n (n 1)/n| = 1 1/n 1 = 0.
For x = 1, the series f  (1) = 1/n is the harmonic series and hence
diverges. The series f  (1) = (n 1)/n also diverges ((n 1)/n does
not converge to 0). Thus, the intervals of convergence for f , f  , and f 
are, respectively, [1, 1], [1, 1), and (1, 1).
2
The term-by-term integration of a power series can be used to obtain a power series representation of antiderivatives.
Example 8.44. Find a power series representation for tan1 x.

142

8. SEQUENCES AND SERIES

Solution:
1

tan

x=

dx
=
1 + x2




2 n

(x )


dx = C +

n=0


n=0

(1)n

x2n+1
.
2n + 1

Since tan 0 = 0, the integration constant C satises the condition


0 = C + 0 or C = 0. The geometric series with q = x2 converges if
|q| < 1 or x2 < 1 or |x| < 1. Hence, the radius of convergence of the
2
series for tan1 x is R = 1.

In particular, the number 1/ 3 is less than the radius of conver


gence of the power series for tan1 x. So the number tan1 (1/ 3)=
/6 can be written as the numerical series by substituting x = 1/ 3
into the power series for tan1 x. This leads to the following representation of the number :


(1)n
=2 3
.
(2n + 1)3n
n=0
58.2. Continuity at endpoints of the interval of convergence. Consider

the geometric series with q = x

(1)n xn =

n=0

1
1+x

By integrating the series term-by-term, the power series for the logarithm function is deduced:



(1)n xn+1
dx
= ln(1 + x) = C +
1+x
n+1
n=0
which holds for |x| < 1 by Theorem 8.28. The constant C is determined
by setting x = 0, which yields C = ln 1 = 0. Thus,
(8.19)

ln(1 + x) =


(1)n1
n=1

xn

This series diverges at x = 1 as a harmonic series, but it converges at


the other endpoint x = 1 as the alternating harmonic series. In Exercise
54.2.25 it has been shown that the sum of the alternating harmonic
series is ln 2. It follows then that the power series representation (8.19)
also holds at x = 1


(1)n1 n
x = ln 2
lim
x1
n
n=1

58. REPRESENTATION OF FUNCTIONS AS POWER SERIES

143

Here x 1 means the left limit (x approaches 1 from the left). This
observation is not specic to the example considered but rather is of
general nature.
8.29 (Abels theorem). If the power series f (x) =
Theorem
n
c
x
,
|x|
<
R, converges at the endpoint x = R, then
n
n=0
f (R) = lim f (x)
xR

Abels theorem asserts that a function dened by a power series is


continuous on the interval of convergence of the power series.
58.3. Power Series and Differential Equations. A power series represen-

tation is often used to solve dierential equations. The relation between


a function f (x), its argument x, and its derivatives f  (x), f  (x), and
so on is called a dierential equation. A function f (x) that satises a
dierential equation is generally dicult to nd in a closed form. A
power series representation turns out to be helpful. Since in
this representation a function is dened by a sequence {cn }, f (x) = cn xn , and
so are its derivatives f (k) (x), a dierential equation imposes conditions
on cn that are solved recursively.
Example 8.45. Find a power series representation of the solution
of the equation f  (x) = f (x) and determine its radius of convergence.


Solution: Put f (x) =
cn xn and hence f  (x) =
ncn xn1 . Then

the equation f = f gives
c1 + 2c2 x + 3c3 x2 + 4c4 x3 + = c0 + c1 x + c2 x2 + c3 x3 + .
By matching the coecients at the monomial terms 1, x, x2 , x3 , and
so on, one nds:
c1
c2
cn1
.
c0 = c1 c2 = , c3 = , ..., cn =
2
3
n
Using the latter relation recursively:
1
1
1
c0
cn1 =
cn2 =
cn3 = = .
n
n(n 1)
n(n 1)(n 2)
n!
 n
So f (x) = c0 n=0 x /n!, where c0 is a constant (the equation is satised for any choice of c0 ). By the ratio test, the series converges for all
x (so R = ). Indeed, cn = 1/n! and cn+1 /cn = 1/(n + 1) 0 =
and hence R = 1/ = .
2
For this simple dierential equation, it is not dicult to nd f (x) =
c0 ex by recalling the properties of the exponential function: (ex ) = ex .
cn =

144

8. SEQUENCES AND SERIES

The condition f (0) = e0 = 1 determines the constant c0 = 1. Thus,


the exponential function has the following power series representation:
 xn
x
x2 x3
e =1+ +
+
+ =
.
1!
2!
3!
n!
n=0

(8.20)

The series converges on the entire real line. In particular, the number
e has the following series representation:
 1
1
1
1
.
e = 1 + + + + =
1! 2! 3!
n!
n=0

58.4. Approximation of Definite Integrals. If an indenite integral of

b
f (x) is dicult to obtain, then the evaluation of the integral a f (x) dx
poses a problem. A power series representation oers a simple 
way to
approximate the value of the integral. Suppose that f (x) =
c n xn
for R < x < R. By Theorem 8.28, for any R < a < b < R,
 b
 an+1
  b
 bn+1

cn
f (x) dx =
cn
xn dx =
cn
n+1
n+1
a
a
n
n
k+1
k+1
 b
 a

ck
ck

.
k
+
1
k
+
1
k=0
k=0
Errors of the approximation of the series sum by nite sums have been
discussed earlier.
Example 8.46. How many terms does one need in the power series
2
approximation of the integral of f (x) = ex over the interval [0, 1] to
make the absolute error smaller than 105 ?

2
Solution: Note rst that the indenite integral ex dx cannot be
expressed in elementary functions! So a direct use of the
fundamental
 x
2
theorem of calculus becomes problematic. However, e dx can be
represented as a power series that converges on the entire real line by
replacing x in (8.20) by (x2 ). One has

 1



(1)k 1 2k
(1)k
x2
e
dx =
x dx =
k!
k!(2k + 1)
0
0
k=0
k=0

n

k=0


(1)k
(1)k bk .
=
k!(2k + 1) k=0
n

To determine n in the nite sum approximation of the series, recall


the alternating series estimation stated in Theorem 8.19, where bn =

58. REPRESENTATION OF FUNCTIONS AS POWER SERIES

1/(n!(2n + 1)):
n
 1


x2
e
dx

0

k=0

145

(1)k 
1
< 105 .
 bn+1 =
k!(2k + 1)
(n + 1)!(2n + 3)

A direct calculation shows that b7 1.32 105 and b8 1.46 106 .


So n = 7 is sucient to approximate the integral with the required
accuracy.
2
58.5. Exercises.

In (1)(3), nd a power series representation for the function and determine the interval of convergence.
1
(1) f (x) =
1 x4

x
(2) f (x) = 2
3x + 2

x10
(3) f (x) =
x1

In (4)(6), use dierentiation to nd a power series representation for


the function and determine the interval of convergence.
1
(4) f (x) =
(1 + x)2

x3
(5) f (x) =
(1 4x2 )2

(6) f (x) =

1
(1 + x4 )3

In (7)(9), use term-by-term dierentiation to nd the sum of the series:


x 3 x5
+
+
3
5
x2 x4
(8) 1 +
+
+
2!
4!
1
13 2 135 3
(9) 1 + x +
x +
x +
2
24
246

(7) x +

Hint: In (9), multiply the derivative by 1 x.


In (10)(12), use integration to nd a power series representation for
the function and determine the radius of convergence.

1 + x2
(12) f (x) = tan1 (3x)
(10) f (x) = ln(1x)x (11) f (x) = ln
1 x2
In (13)(15), use term-by-term integration to nd the sum of the series.
(13) x + 2x2 + 3x3 +
(14) x 4x2 + 9x3 16x4 +
(15) 1 2x + 2 3x2 + 3 4x3 +

146

8. SEQUENCES AND SERIES

In (16)(18), nd a power series representation for the indenite integral and determine the interval of convergence.

 x

e 1x
ln(1 x)
dx (18)
tan1 (x2 ) dx
dx (17)
(16)
x
x2
(19) Find a power series representation for sin x and cos x using
the dierential equation f  + f = 0. Determine the interval of
convergence.
Hint: Show rst that a sin x + b cos x satises the equation for
arbitrary numbers a and b.
(20) Show that the Bessel function of order 0 dened in Example
8.34 satises the dierential equation:
x2 J0 (x) + xJ0 (x) + x2 J0 (x) = 0.
In (21)(23), use dierentiation or integration to nd the sum of the
series.




(1)n xn+1
n1
n2
nx
(22)
n(n 1)x
(23)
(21)
2n (n + 1)
n=1
n=2
n=0
In (24)(26), how many terms does one need in a power series approximation to evaluate the integral with the absolute error not exceeding
106 ?
 1 x
 1/2
 1
dx
e 1
(25)
ln(1 + x4 ) dx
dx
(26)
(24)
8
1
+
x
x
0
0
0
(27) Find the radius of convergence of the hypergeometric series:
1+

a(a + 1)b(b + 1) 2 a(a + 1)(a + 2)b(b + 1)(b + 2) 3


ab
x+
x +
x + ,
1! c
2! c(c + 1)
3! c(c + 1)(c + 2)

where a, b, and c are reals. Use De Morgans test to determine


the interval of convergence.
In (28)(30), use the fraction decomposition to nd the power series
representation of the function and determine the interval of convergence.
x
1
12 5x
(29)
(30)
(28)
2
2
6 5x x
(1 x)(1 + x )
1 + x + x2 + x3
(31) Find the sum of the series


(1)n1
n=1

2n 1

Hint: Use Abels theorem and Example 8.44.

59. TAYLOR SERIES

147


n
2
(32) Show that the series y =
n=0 x /(n!) satises the dieren

tial equation xy + y y = 0.
59. Taylor Series
59.1. Real Analytic Functions. Suppose a function f is represented by

a power series (R > 0):


f (x) = c0 + c1 (x a) + c2 (x a) + =
2

cn (x a)n .

n=0

where |x a| < R. By Theorem 8.28, its derivatives f (k) (x) can obtained by the term-by-term dierentiation of the series, and the resulting series has the same convergence radius R. Evidently, f (a) = c0 .
What is the signicance of the other coecients cn ? The derivative f 
is given by
f  (x) = c1 + 2c2 (x a) + 3c3 (x a)2 + + kck (x a)k1 + ,
which shows that f  (a) = c1 . The second derivative is
f  (x) = 2c2 + 3 2c3 (x a) + + k(k 1)ck (x a)k2 + .
Therefore, f  (a) = 2c2 . After k such steps,
f (k) (x) = k(k 1) 2 1 ck + (k + 1)k(k 1) 2 ck+1 (x a) + ,
and hence f (k) (a) = k!ck or ck = f (k) (a)/k!. This proves the following
theorem.
Theorem 8.30 (Signicance of Power Series Coecients). If f has
a power series representation


cn (x a)n , |x a| < R,
f (x) =
n=0

for some a and R > 0, then its coecients are


cn =

f (n) (a)
.
n!

Definition 8.12 (Real Analytic Functions). A function f on an


open interval I is said to be 
analytic if, for any a I, it has a power
series representation f (x) =
cn (x a)n that converges in some open
interval (a , a + ) I, where > 0.
The class of analytic functions plays a signicant role in applications. Their properties are discussed next.

148

8. SEQUENCES AND SERIES

Theorem 8.31 (Power Series Representation of Analytic Functions). A function f that is analytic on an open interval I has the
power series representation
(8.21)

f (x) =


f (n) (a)
n=0

n!

(x a)n

for any a I that converges in an open subinterval of I that includes a.


This theorem follows from Denition 8.12 and Theorem 8.30.
In Example 8.42 it was found that
1  (1)n
=
(x a)n ,
x n=0 an+1

(8.22)

x (0, 2a).

This shows that the function f (x) = 1/x is analytic for all x > 0
because a can be any positive number; that is, the function has a
power series representation that converges in an open subinterval of
(0, ) containing any a > 0. Similarly, the analyticity of f (x) = 1/x
can be established for all x < 0.
It is important to emphasize that a power series for an analytic
function does not necessarily converge on the entire domain of the
function. But an analytic function can always be represented by a
convergent power series in a neighborhood of every point of its domain.
Equation (8.22) illustrates the point.
Theorem 8.32 (Properties of Analytic Functions).
(i) The sums and products of analytic functions are analytic.
(ii) The reciprocal 1/f of an analytic function f is analytic if f is
nowhere zero.
(iii) The composition f (g(x)) of analytic functions f and g is analytic.
(iv). Analytic functions are dierentiable innitely many times.
A proof of properties (i)(iii) is given in more advanced calculus
courses. Property (iv) follows from Theorem 8.28. Its converse is not
generally true. There are functions that are dierentiable innitely
many times at a point, but they cannot be represented by a power series that converges in an open interval that includes this point. As an
example, consider the function
f (x) = e1/x

if

x = 0 and f (0) = 0.

59. TAYLOR SERIES

149
2

The function is continuous at x = 0 because limx0 e1/x = limu eu


= 0 = f (0). It is dierentiable at x = 0 because
2

f (x) f (0)
e1/x
= lim
= 0.
x0
x0
x
x
The rst equality is the denition of f  (0). The last limit is established
by investigating the left and right limits x 0 with the help of the
substitution x = 1/u as x 0 ; the left and right limits
2
coincide because ueu 0 as u (the exponential function
decreases faster than any power function). In a similar fashion, it can
be proved that f (n) (0) = 0 for all n
(see Exercise 59.5.24). Thus, f (x)
has no power series representation
cn xn in a neighborhood of x = 0
because, if it did, then, by Theorem 8.31 the function should have been
identically 0 in some interval (, ), > 0, (as f (n) (0) = 0 for all n),
which is not true (f (x) = 0 for all x = 0). Hence, the function is not
analytic at x = 0.
f  (0) = lim

59.2. Taylor and Maclaurin Series.

Definition 8.13 (Taylor and Maclaurin Series). The series in


(8.21) is called the Taylor series of a function f at a (or about a,
or centered at a). The special case of the Taylor series when a = 0 is
called the Maclaurin series of a function f .
It is important to know Maclaurin series of elementary functions.
In particular, Maclaurin series for the power function (1 + x)p , where
p is a real number, is called the binomial series. The numbers
 
p(p 1) (p n + 1)
p
=
n
n!
are called binomial coecients.
Theorem 8.33 (Maclaurin series of elementary functions). For any
real x, the following power series representations hold
 xn
x 2 x3
+
+ =
2!
3!
n!
n=1

ex = 1 + x +

 (1)n x2n+1
x3 x 5 x 7
+

+ =
3!
5!
7!
(2n + 1)!
n=0

sin x = x

 (1)n x2n
x2 x4 x6
cos x = 1
+

+ =
2!
4!
6!
(2n)!
n=0

150

8. SEQUENCES AND SERIES

For |x| < 1, the following power series representations hold


 (1)n1
x 2 x3 x4
ln(1 + x) = x
+

+ =
xn
2
3
4
n
n=1
 

p
p(p 1) 2
p
p
xn
(1 + x) = = 1 + x +
x + =
n
1!
2!
n=0

Proof. The Maclaurin series of the exponential function ex and


the logarithm function ln(1 + x) have been already established in Sections 58.3 and 58.2, respectively. Let f (x) = sin x. One has f  (x) =
(sin x) = cos x and f  (x) = (cos x) = sin x. Hence,
f (2n) (x) = (1)n sin x ,

f (2n+1) (x) = (1)n cos x,

and f (2n) (0) = 0, f (2n+1) (0) = (1)n1 . By Theorem 8.30, the non-zero
coecients of the Maclaurin series are
c2n+1

f (2n+1) (0)
(1)n
=
=
(2n + 1)!
(2n + 1)!

n = 0, 1, 2, ...

By the ratio test,


|c2n+3 x2n+3 |
x2 (2n + 1)!
x2
=
lim
=
lim
=0
n |c2n+1 x2n+1 |
n (2n + 3)!
n (2n + 2)(2n + 3)
lim

the power series converges for any x. The Maclaurin series for f (x) =
cos x = (sin x) is obtained by dierentiating the series for sin x termby-term. By Theorem 8.28, it also converges on the entire real line.
Finally, put f (x) = (1 + x)p . The derivatives are f  (x) = p(1 + x)p1 ,
f  (x) = p(p 1)(1 + x)p2 , and, in general,
f (n) (x) = p(p 1) (p n + 1)(1 + x)pn .
So the coecients of the power series coincide with the binomial coefcients introduced earlier:
 
f (n) (0)
p(p 1) (p n + 1)
p
=
=
.
cn =
n
n!
n!
They satisfy the recurrence relation cn+1 = cn (pn)/(n+1). Therefore,
by Corollary 8.3, the radius of convergence R is determined by
|1 np |
|cn+1 |
|p n|
lim
=1
= lim
= lim
n |cn |
n n + 1
n 1 + 1
n

R = 1.
2

59. TAYLOR SERIES

151

59.3. Taylor Series of Analytic Functions. Every analytic function in a

neighborhood of a point is represented by the Taylor series about that


point. If the Taylor series converges on the entire real line, then the
function is analytic everywhere. In particular, the exponential ex and
trigonometric functions sin x and cos x are analytic everywhere. Moreover, the properties of analytic functions stated in Theorem 8.32 allows
us to add, multiply, and make a composition of the Taylor series (on
the common intervals of their convergence) just like ordinary sums to
obtain the Taylor series representation of the sums, products, and compositions of analytic functions. These are extremely useful properties
in applications.
Example 8.47. Find rst four terms of the Taylor series for the
function f (x) = exp(tan1 x) about x = 0.
Solution: Calculation of the derivatives of such a function is rather
tedious. Instead, note that ex and tan1 x are both analytic in a neighborhood of x = 0. So the composition of their Taylor series (see (8.20)
and Example 8.44) gives the sought-after Taylor series. Only monomials 1, x, x2 , and x3 have to be retained when calculating the composition. This implies that it is sucient to retain two leading terms in
the Taylor series tan1 x = x x3 /3 + and four leading terms in
the Taylor series (8.20) of the exponential function:
1
1 2 1
1 3
1
tan x +
tan x +
etan x = 1 + tan1 x +
2
6

2 1

3
x3
x3
x3
= 1+ x
+ + x
+ + x
+ +
3
2
3
6
3

3
1
1
x
+ x2 + x3 +
=1+ x
3
2
6
1 2 1 3
= 1 + x + x x + .
2
6
2
59.4. Approximations by Taylor Polynomials. An analytic function f can

be approximated by a partial sum of the Taylor series:


n

f (k) (a)
(x a)k = Tn (x).
f (x)
k!
k=0
The polynomial Tn (x) is called a Taylor polynomial of degree n about
a. The convergence of the Taylor series guarantees that the remainder
converges to 0:
Rn (x) = f (x) Tn (x) 0 as n for |x a| < R,

152

8. SEQUENCES AND SERIES

where R is the radius of convergence of the Taylor series. The accuracy


of the Taylor polynomial approximation for a function is assessed in
Taylors theorem discussed in Calculus I. Here it is restated in a slightly
dierent form.
Theorem 8.34 (Taylors Theorem). Suppose a function f is analytic near a and let Tn (x) be its Taylor polynomials about a. Then, for
every n and any |x a| < R, where R is the radius of convergence of
the Taylor series for f about a, there exists a point between a and x
such that
f (n+1) ()
(x a)n+1 .
Rn (x) = f (x) Tn (x) =
(n + 1)!
Proof. Given a number x, |x a| < R, let M be a number dened
by
f (x) = Tn (x) + M (x a)n+1 .
Consider the function
g(t) = f (t) Tn (t) M (t a)n+1 ,

where |t a| < R.

Since the (n + 1)th derivative of a polynomial of degree n vanishes


and hence g (n+1) (t) = f (n+1) (t) n!M , the proof will be complete if
one can show that g (n+1) () = 0 for some between x and a because
the latter would imply that M = f (n+1) ()/n!. By the denition of
(k)
Taylor polynomials, f (k) (a) = Tn (a) for k = 0, 1, ..., n, and hence
g(a) = g  (a) = = g (n) (a) = 0. The function g(t) is dierentiable
and g(x) = g(a) = 0 by the choice of M ; therefore, by Rolles theorem,
there is a number t1 between x and a such that g  (t1 ) = 0. Similarly,
the function g  (t) is dierentiable and g  (t1 ) = g  (a) = 0; hence, there is
a number t2 between t1 and a such that g  (t2 ) = 0. After n + 1 steps of
this procedure, one arrives at the conclusion that g (n+1) (tn+1 ) = 0 for
some number tn+1 = between tn and a, that is, between x and a. 2
Corollary 8.4 (Taylors Inequality). If |f (n+1) (x)| Mn for
|x a| d < R, then the remainder of the Taylor series satises
the inequality
Mn
|Rn (x)|
for |x a| d .
|x a|n+1
(n + 1)!
Since in Taylors theorem lies between x and a, one has |f (n+1) ()|
Mn for |x a| d, and the conclusion of the corollary follows. All
derivatives of an analytic function are continuous and, hence, attain
their maximal and minimal values on any closed interval |x a| d.
So Mn = max |f (n+1) (x)| on |x a| d.

59. TAYLOR SERIES

153

Figure 8.9. An illustration of an approximation of


f (x) = sin x (the dashed red curve) by its Taylor polynomials at x = 0 (the solid blue curve). As n increases,
Tn (x) approaches f (x) = sin x. The approximation becomes better in a larger interval for a larger n in accordance with the analysis of Example 8.50.
Example 8.48. Approximate the function ln(1+sin x) by the Taylor
polynomial of degree 3 about the origin.
Solution: First note that the function in question is a composition
of analytic functions. Since ln(1 + u) = u u2 /2 + u3 /3 + , the
polynomial T3 (x) may be obtained by retaining only terms at most
cubic in x, when expanding u = sin x x x3 /6. Hence, neglecting
all monomials of degree 4 and higher

2

3
x3 1
1
x3
x3
ln(1 + sin x) x
+

x
x
6
2
6
3
6
1
1
x x2 + x3 = T3 (x)
2
6
2

154

8. SEQUENCES AND SERIES

Example 8.49. Approximate the function xp by the Taylor polynomial of degree 2 about x = 1.
Solution: If f (x) = xp , then f  (x) = pxp1 and f  (x) = p(p 1)xp2 .
Therefore f (1) = 1, f  (1) = p, and f  (1) = p(p 1). The needed
Taylor polynomial is
f  (1)
(x 1)2
2
p(p 1)
= 1 + p(x 1)
(x 1)2
2

T2 (x) = f (1) + f  (1)(x 1) +

Alternatively, one could use the identity f (x) = xp = (1 + (x 1))p =


2
(1 + u)p and the binomial series in u.
Example 8.50. Find an upper bound on the error of the Taylor
polynomial approximation about x = 0 for the function f (x) = sin x.
In particular, give an upper bound for the error of the approximation
sin x T7 (x) in the interval [1, 1].
Solution: The Maclaurin series for sin x contains only odd powers
of x and so are the Taylor polynomials (f (2n+2) (x) = (1)n+1 sin x
and, hence, f (2n+2) (0) = 0). Therefore T2n+1 (x) = T2n+2 (x) and the
accuracy of the approximation of sin x by T2n+1 (x) is determined by
|f (2n+3) (x)| = |(1)n+1 cos x| 1 = M uniformly for all x and all n.
By Corollary 8.4,

 

|x|2n+3

 

.
sin x T2n+1 (x) = sin x T2n+2 (x) =
(2n + 3)!
The error of the approximation
x3 x5 x7
sin x x
+

,
3!
5!
7!

|x| 1 ,

is less than 1/9! 0.000003.


2
Corollary 8.4 also shows that, for a xed n, the error may grow
with increasing |x| (as in the above example). This implies that Taylor
polynomials of higher degrees are needed to achieve the same accuracy
for large |x| as for smaller |x| (see Fig. 8.9). To avoid using high-degree
Taylor polynomials to approximate the function at large |x|, one can
use Taylor polynomials about some a close to the range of x in which
the approximation is needed.

59. TAYLOR SERIES

155

59.5. Exercises.

In (1)(5), nd the Maclaurin series for the function and the radius of
convergence.
(1) ln(1+x) (2) tan x (3) sinh x (4) cosh x (5) x6 +2x5 x3 +x3
In (6)(9), nd the Taylor series for the function about a and the radius
of convergence.

(6) cos x , a =
(7) 1/ x , a = 4
(8) sin x , a = /2

(9) (1 + x)2/3 , a = 7

In (10)(13), use the Maclaurin series for basic functions to nd the


Maclaurin series for the function.
x
x sin x
(10) x cos(x2 /2) (11)
(12)
(13) x2 tan1 (x2 )
3
3
4
x
1+x
In (14)(17), use the products and composition of the Maclaurin series
for basic functions to nd the rst three non-vanishing terms of the
Maclaurin series for the function.
(14) sin((cos x)) (15) esin x

(16) tan1 x ln(1 + x) (17) ln(cos x)

(18) Find the rst ve nonvanishing terms of the Maclaurin series


for f (x) = ex / cos x. Hint: Put f (x) = c0 + c1 x + + c4 x4 +
, then use the product of the Maclaurin series to nd the
coecients from ex = f (x) cos x.
In (19)(21), nd the degree of a Taylor polynomial to approximate
the integrand so that the error of approximating the integral does not
exceed 103 .
 1/2
 1 x
 1/2
e 1
1 2
(19)
tan (x ) dx (20)
(1 + x4 )1/4 dx
dx (21)
x
0
0
0
In (22)(24), nd the sum of the series.



(1)n 2n
(1)n 3n
(22)
(23)
62n (2n)!
2n n!
n=0
n=0
(ln 2)2 (ln 2)3

+
2!
3!
2
(25) (i) For the function f (x) = e1/x if x = 0 and f (0) = 0, show
that f (n) (0) = 0 for all n and hence f cannot be represented
as a power series near 0. (ii) Let f (x) = e1/x if x > 0 and
f (x) = 0 if x 0. Is this function analytic everywhere?
(26) Use the identity /6 = sin1 (1/2) to calculate the number
within the error not exceeding 104 .
(24) 1 ln 2 +

156

8. SEQUENCES AND SERIES

(27) Find cos(1 ) within the error not exceeding 106 .


(28) Use
the rst three terms of the binomial series to approximate

3
9 and estimate the error.
In (29)(32), use various methods to nd the Maclaurin series of the
function and investigate its convergence radius.
 x
 x
dt
t2

(29)
e dt
(30)
1 t4
0
0
 x
 x
sin t
tan1 x
dt
(32)
dt
(31)
t
t
0
0

n
(33) Let f (x) =
n=0 cn x , where |n!cn | < M for n = 1, 2, 3, ...
and some number M . Prove that (i) f (x) is dierentiable
innitely many times at any point x = a and (ii) the Taylor
series representation of f (x) about a holds for |x| < .

CHAPTER 9

Further Applications of Integration


60. Arc Length
60.1. The Length of a Curve. We have seen various applications of in-

tegration to the computation of the area of a domain and to the computation of the volume of a solid. It is perhaps more surprising that
we can also use integration to compute the length of a curve between
two given points. This may sound counterintuitive at rst, since in
the applications we have seen so far, integration was used to compute
some parameter of an object that existed in a higher dimension than
the function that was being integrated.
Let f be a function so that, on the interval [a, b], the derivative f 
of f exists and is a continuous function. We would like to know the
length of the curve of f , starting at the point A = (a, f (a)) and ending
at the point B = (b, f (b)).
Intuitively, we can imagine that we lay a rope over the graph of f
between the two endpoints, mark A and B on the rope, then straighten
that rope out, and measure the distance between them.
A more formal denition, which is useful in the actual computation
of the length of the curve, is the following. Cut the interval [a, b]
into n equal parts, using points a = x0 < x1 < < xn = b. Let
Pi = (xi , f (xi )) = f (xi , yi ). Let |Pi1 Pi | denote the length of the
straight line segment from Pi1 to Pi . Then the sum
(9.1)

Kn =

n


|Pi1 Pi |

i=1

is a little bit smaller than the length of the curve since the points
Pi1 and Pi are on the curve and the straight line is the shortest path
between them.
If we keep rening the subdivision of the interval [a, b] by having n
go to innity, then it can be proved that limn Kn exists. We dene
that limit to be the length of the curve of f from A to B. See Figure
9.1 for an illustration. Note that in the case when the graph of f is
a straight line segment between A and B, this denition is just the
length of that segment, so our denition extends our previous notion
of length.
157

158

9. FURTHER APPLICATIONS OF INTEGRATION

Figure 9.1. Arc length as a limit.


Let us now return to (9.1) in order to compute limn Kn . Let
(b a)/n = (xi xi1 )/n = x. Note that then

|Pi1 Pi | = (xi xi1 )2 + (yi yi1 )2

(yi yi1 )2
= (x)2 + (x)2
(xi xi1 )2

(yi yi1 )2
= x 1 +
.
(xi xi1 )2
Now observe that since f  is continuous, the Mean Value Theorem
(Theorem 6.9) implies that there is a real number xi [xi1 , xi ] such
i1
that xyii y
= f  (xi ). Hence, the previous chain of equalities yields
xi1
|Pi1 Pi | = x


1 + f  (xi )2 .

Summing over all i, we get


Kn =

n



1 + f  (xi )2 .

i=1

As n goes to innity, the left-hand side, by denition, converges to the


length of the curve of f between Aand
B, while the right-hand side,
b
being a Riemann sum, converges to a 1 + f  (x)2 dx. Hence, we have
proved the following theorem.
Theorem 9.1. If f  is a continuous function on the interval [a, b],
then the length of the graph of f (x) from the point (a, f (a)) to the point

60. ARC LENGTH

159

Figure 9.2. The curve of f (x) = 23 x3/2 .


(b, f (b)) is equal to

 b
1 + f  (x)2 dx.
L=
a

Example 9.1. Find the length of the curve of f (x) = 23 x3/2 from
(0, 0) to (1, 2/3). See Figure 9.2 for an illustration.

Solution: We have f  (x) = x, so f  is a continuous function on [0, 1],


and therefore Theorem 9.1 applies. Using that theorem, we obtain
 1

1 + ( x)2 dx
L=
0
 1

1 + x dx
=
0

 1
2

=
(1 + x)3/2 

3
0

4 2 2
.
=
3
3
2
Note that the result is remarkably close to the lengthof the straight
line that connects the two points in question, which is 13/3.
We can use our new technique to verify a classic formula.
Example 9.2. Use Theorem 9.1 to compute the circumference of a
circle of radius 1.
Solution: Let us place the center of the unit circle at the origin. Then
the boundary of the circle is the set of points satisfying x2 + y 2 = 1.

160

9. FURTHER APPLICATIONS OF INTEGRATION

Figure 9.3. One quarter of the unit circle.


We want to use Theorem 9.1, so we need a part of the circle where that
satises the vertical line test (so y is a function of f ) and where the
tangent line to the circle is never vertical (so that f  (x) exists). For
instance,
that starts in the point

we
can choose the quarter of the
circle

2
2
x = 2 and ends in the point x = 2 . See Figure 9.3 for an
x
illustration. On that part of the curve, f  (x) = 1x
2 is continuous,
so Theorem 9.1 implies
 2/2
1 + f  (x)2 dx
L=
2/2

2/2


=
=

2/2
 2/2

2/2


1+

x2
dx
1 x2

1
dx
1 x2

2/2


= sin1 x


2/2

= /2.
This implies that the circumference of the full circle is four times this
much, that is, 2.
2

60. ARC LENGTH

161

60.2. Remarks. Recall that in the rst paragraph of this section, we

discussed why it may seem counterintuitive that integration plays a role


in the computation of arc lengths. Now we can see that the purported
contradiction explained there is resolved by the fact that the integrand
in Theorem 9.1 contains f  , not f .
Compared to other formulas we learned in our earlier studies of
integration, it is relatively rare thatthe
formula given by Theorem 9.1
b
can be explicitly computed, since a 1 + f  (x)2 dx is often dicult
to handle. Therefore, we must often resort to approximate integration
while computing arc lengths.
60.3. Exercises.

(1) Find the length of the curve f (x) = x2 /2 between the points
given by x = 0 and x = 1.
(2) Find the length of the curve f (x) = 3x2 /2 between the points
given by x = 0 and x = 1.
2
(3) Find the length of the curve f (x) = x4 ln2x between the points
given by x = 2 and x = 4.
(4) Find the length of the curve f (x) = ln(cos x) between the
points given by x = 0 and x = /4.
(5) Find the length of the curve f (x) = ln(sin x) between the
points a = /4 and b = 3/4.
2
(6) Find the length of the curve f (x) = ln x x8 from x = 4 to
x = 5.
(7) Verify that Theorem 9.1 provides the correct value for the arc
length of f when f is a linear function.

(8) Let f = 14 (x2 +2x)3/2 12 ln(x+1+ x2 + 2x). Find the length


of the curve of f from
2.
x = 1 to x =

1
1
2
(9) Let f = 4 (2x + 4) x + 4x + 3 2 ln(x + 2 + x2 + 4x + 3).
Find the lengthof the curve of f from
x = 1 to x = 2.
1
1
2
(10) Let f (x) = 2 x x 1 2 ln(x + x2 1). Find the length
of the curve of
f from x = 1 to x =2.
1
(11) Let f (x) = 2 x x2 + 1 + 12 ln(x + x2 + 1). Find the length
of the curve of f from x = 1 to x = 2.
(12) Let f and g be two functions that have continuous derivatives
on the interval [a, b] and assume that f  (x) = g  (x) for all
x [a, b]. Prove that the curves of f and g have the same
length between x = a and a = b. Try to nd two dierent
arguments.
(13) Use a method of approximate integration to estimate the length
2
of the curve f (x) = ex as from (0, 1) to (1, e).

162

9. FURTHER APPLICATIONS OF INTEGRATION

(14) Use a method of approximate integration to estimate the length


of the curve f (x) = ex as from (0, 1) to (1, e). Compare the
result with that of the previous exercise.
(15) Compute the exact arc length in the previous exercise by either
using a software package or an appropriate substitution to nd
the needed indenite integral.
(16) Use a method of approximate integration to estimate the length
of the curve f (x) = sin x from (0, 0) to (, 0).
(17) Use a method of approximate integration to estimate the length
of f (x) = x3 from (0, 0) to (1, 1).
(18) Use the result of the previous exercise to estimate the length
of f (x) = cos x from x = to x = .
(19) Find the length of the curve f (x) = ln x from x = 1 to x =
2. You can use a software package to compute the needed
indenite integral.
(20) Use a method of approximate integration to estimate the length
of f (x) = tan x from x = 0 to x = /4.
61. Surface Area
61.1. The Definition of Surface Area. In the last section, we dened the

length of a curve, and deduced a formula for the computation of that


length. Let us now take a curve, say of a function f (x) = y, where
x [a, b] and f  is continuous on [a, b]. Let us rotate this curve around
the horizontal axis, as shown in Figure 9.4. What is the area of the
obtained surface of revolution?
The denition of the area in question, and its computation, will be
quite similar to what we have discussed in the previous section for the
arc length.
Cut the interval [a, b] into n equal parts, using points
a = x0 < x1 < < xn = b.
Let Pi = (xi , f (xi )) = f (xi , yi ) and let li = |Pi1 Pi | denote the length
of the straight line segment from Pi1 to Pi . As we rotate the curve
of f around the horizontal axis, the rotation of the segment Pi1 Pi
results in the lateral surface Sn,i of a truncated cone with slant height
li and radii yi1 and yi . See Figure 9.5 for an illustration. It is then
not dicult to prove that the area of Sn,i is equal to
(9.2)

A(Sn,i ) = (yi1 + yi )li .

As n goes to innity, the sum of the areas of the surfaces Sn,i approximates what we intuitively think of as the area of the surface obtained
by rotating the curve.

61. SURFACE AREA

163

Figure 9.4. A surface obtained by rotating a curve.

Figure 9.5. Approximating a surface of revolution.


In fact, it can be proved that the limit
(9.3)

A(S) = lim

n

i=1

Sn,i = lim

n


(yi1 + yi )li

i=1

exists. We dene this limit to be the area of the surface of revolution


obtained when the curve of f is rotated around the horizontal axis,
with x [a, b].
In order to compute this surface area, recall from the last section
that there exists a real number xi [xi1 , xi ] such that li = |Pi1 Pi | =

164

9. FURTHER APPLICATIONS OF INTEGRATION


x 1 + f  (xi )2 . Also note that, since f is continuous, small changes
in x lead to small changes in f (x) = y, so if n is large enough, then
f (xi ) f (xi ) = yi and f (xi ) f (xi1 ) = yi1 . Therefore, (9.3)
implies
(9.4)

A(S) = lim

n


2f (xi ) x


1 + f  (xi )2 ,

i=1

where x = (b a)/n. Now notice that the last expression obtained


for S(A) is the
limit of a Riemann sum, that is, an integral (of the
function 2f (x) 1 + f  (x)2 ). This means that we proved the following
theorem.
Theorem 9.2. Let f be a function that f  is continuous on the
interval [a, b]. Let S be the surface obtained by rotating the curve y =
f (x), where x [a, b], around the horizontal axis.
Then the area of S is
 b

2f (x) 1 + f  (x)2 dx.
A(S) =
a

Example 9.3. Compute the surface area of a sphere of radius r.


Solution: Such a sphere canbe obtained by rotating the semicircle
given by the equation f (x) = r2 x2 around the horizontal axis. See
Figure 9.6 for an illustration.
Theorem 9.2 then yields

 r
x2
2 r2 x2 1 + 2
dx
A(S) =
r x2
r

 r
r2
=
2 r2 x2
dx
r 2 x2
r
 r
=
2r dx
r
r

= 2rx
r

= 4r .

61.2. Variations. If we rotate our curve about the vertical axis instead

of the horizontal axis, then most of the previous argument remains


valid. The only dierence is that when the point Pi = (xi , yi ) is rotated,
it is rotated in a circle of radius xi , not yi . This leads to the following
theorem.

61. SURFACE AREA

165

Figure 9.6. A sphere as a surface of revolution.


Theorem 9.3. Let f be a function such that f  is continuous on
the interval [a, b]. Let S be the surface obtained by rotating the curve
y = f (x), where x [a, b], about the vertical axis.
Then the area of S is
 b

A(S) =
2x 1 + f  (x)2 dx.
a

Note that the f (x) term in the integrand of Theorem 9.2 is replaced
by x.
Example 9.4. Rotate the curve given by y = f (x) = x2 /2, with
x [0, 1], around the vertical axis. Find the area of the obtained surface.

Figure 9.7. The curve of y = x2 /2 and the surface


obtained by its rotation.

166

9. FURTHER APPLICATIONS OF INTEGRATION

Solution: Theorem 9.3 implies



A(S) =
0

2x 1 + x2 dx

1 2
= 2
(x + 1)3/2
3

2 21
= 2
3
0.6095.

 1



0

Note that another way of writing the result of Theorem 9.2 is




A(S) =

2f (x) 1 +
a

dy
dx

2
dx.

By interchanging the roles of x and y, this implies that, for curves given
by an equation g(y) = x, the following holds.
Theorem 9.4. Let g be a function such that g  is continuous on
the interval [a, b]. Let S be the surface obtained by rotating the curve
x = g(y), where y [a, b], around the vertical axis.
Then the area of S is given by



2g(y) 1 +

A(S) =
a

dx
dy

2
dy.

While the surface area of a cone can be computed by elementary


methods, it is elucidating to compute it with our new method and see
that the result is what we expect it to be.
Example 9.5. Find the surface area of a right cone of base radius
R and height h.
Solution: The base circle of the cone has area R2 . In order to compute the lateral surface, note that the lateral surface can be obtained by
+ R = g(y),
rotating the line segment given by the equation x = Ry
h
where y [0, h], around the vertical axis. Therefore, Theorem 9.4
applies, and for the lateral surface area, it yields

61. SURFACE AREA

167


Ry
R2
+R
A(S) =
2
1 + 2 dy
h
h
0


h
2

R2 + h2 y

= 2R

+
y

h2
2h
0

= R R2 + h2
= Rs,


where s = R2 + h2 is the slant height of the cone.


So the total surface area of the cone is the sum of the area of its
base plus the area of its lateral surface, that is, R2 +Rs = R(R+s).
2
Theorem 9.4 also has a version that applies to curves given as functions of y that are rotated around the horizontal axis.
Theorem 9.5. Let g be a function such that g  is continuous on
the interval [a, b]. Let S be the surface obtained by rotating the curve
x = g(y), where y [a, b], about the horizontal axis.
Then the area of S is given by

 2
 b
dx
2y 1 +
dy.
A(S) =
dy
a
Note that for the computation of some surface areas, we will have
a choice of two theorems discussed in this section. The reader is encouraged to describe the curves whose rotations lead to such surfaces.
61.3. Exercises.

(1) Rotate the curve of y = f (x) = x2 + 5, where x [2, 3], about


the y axis. Find the surface area.
(2) Rotate the curve of y = f (x) = 9x2 +1, where x [2, 3], about
the y axis. Find the surface area.
(3) Rotate the curve of y = f (x) = 43 x3/2 , where x [0, 1], about
the y axis. Find the surface area.
(4) Compute the surface area obtained by rotating the curve y =
ex , for x [0, 1], about the x axis.
(5) Rotate the curve of y = f (x) = x, where x [0, 1], about
the x axis. Find the surface area.
(6a) Rotate the curve of y = f (x) = 3 x, where x [3, 4], about
the y axis. Find the surface area.

168

9. FURTHER APPLICATIONS OF INTEGRATION

(6b) Rotate the curve of y = f (x) = 9x + 1, where x [3, 4],


about the x axis. Find the surface area.
(7) Rotate the curve of y = f (x) = x3 , where x [0, 1], about the
x axis. Find the surface area.
(8) Rotate the curve of y = f (x) = x3 , where x [0, 1], about the
y axis. Find the surface area.
(9) Rotate the curve of x = g(y) = 23 (y + 1)3/2 , where y [2, 3],
about the x axis. Find the surface area.
(10) Rotate the curve of x = g(y) = 12 y 2 , where y [2, 3], about
the x axis. Find the surface area.
(11) Solve Example 9.5 using Theorem 9.3.
(12) Let f be a function such that f  is continuous on [a, b]. Prove
that the area of the surface obtained by rotating the graph of
f from x = a to x = b about the y axis is the same as the
area of the surface obtained by rotating the same segment of
the graph of f1 (x) = f (x) + C about the y axis. Here C is an
arbitrary constant. Try to nd two dierent arguments.
(13) Let f be a function such that f  is continuous on [a, b]. Prove
that the area of the surface obtained by rotating the graph of f
from x = a to x = b about the x axis is the same as the area of
the surface obtained by rotating the graph of f2 (x) = f (x+C)
from x = a C to x = b C about the x axis. Here C is an
arbitrary constant. Try to nd two dierent arguments.
(14) Rotate the curve of y = f (x) = x2 , where x [2, 3], about the
x axis. Find
You may want to use the
surface area. (Hint:
 the
1
2
fact that 1/ 1 + x dx = sinh (x).)
In the remaining exercises of this section, use a method of approximate integration to compute the area of the given surface
of revolution.
(15) f (x) = sin x, x [0, ], rotated about the x axis.
(16) f (x) = sin x, x [0, /2], rotated about the y axis.
(17) f (x) = ln x, x [1, 2], rotated about the x axis.
(18) f (x) = ln x, x [1, 2], rotated about the y axis.
2
(19) g(y) = ey , y [0, 1], rotated about the x axis.
2
(20) g(y) = ey , y [0, 1], rotated about the y axis.
62. Applications to Physics and Engineering
62.1. Center of Mass.
62.1.1. One-Dimensional Systems. Let us assume that we have two

objects of mass m1 and m2 placed on the line of real numbers, at

62. APPLICATIONS TO PHYSICS AND ENGINEERING

169

points x1 and x2 , respectively. We want to nd the point xg such that


if we place a fulcrum under the interval [x1 , x2 ] at xg , the objects at the
endpoints of the interval will balance. See Figure 9.8 for an illustration.
We assume that the interval [x1 , x2 ], or the stick representing it, has
negligible mass.
If m1 = m2 , then we clearly have xg = (x1 + x2 )/2. Otherwise,
we make use of the well-known fact of physics that the interval will
balance if the moments on the two sides of the fulcrum are equal, that
is, when
(9.5)

m1 (xg x1 ) = m2 (x2 xg )

holds. Solving (9.5) for xg , we get


(9.6)

xg =

m 1 x1 + m 2 x2
.
m1 + m2

The point xg of the real line is called the center of mass or center of
gravity of the system described above, that is, the system of an object of
mass m1 at x1 and an object of mass m2 at x2 . The moment of an object
with respect to a point P is the mass of the object times the distance of
the object from P . In particular, in the above system, the two objects
had moments m1 x1 and m2 x2 with respect to the origin. So the total
system had moment m1 x1 + m2 x2 . Note that if we replace the two objects by a simple object of mass m1 +m2 placed at xg , then the moment
of the system about the origin does not change. This is an important
property that only the center of mass has, and therefore we repeat it.
If we concentrate the total mass of the system at the center of mass,
the moment of the system with respect to the origin will not change.
If we consider a system of k distinct objects of mass m1 , m2 , . . . , mk
placed at points x1 , x2 , . . . , xk along the horizontal axis, then we can
use an analogous argument to show that the center of mass of the
system is at

Figure 9.8. Center of mass.

170

9. FURTHER APPLICATIONS OF INTEGRATION

k
(9.7)

xg = i=1
k

m i xi

i=1 mi

62.2. Two-Dimensional Systems.


62.2.1. Discrete Two-Dimensional Systems. Let us now consider the

more general case when the k objects of mass m1 , m2 , . . . , mk are placed


in points (x1 , y1 ), (x2 , y2 ), . . . , (xk , yk ) of the plane. We would like to
nd the center of mass (xg , yg ) of this system. In other words, we
assume that a plate of negligible mass is placed under our system, and
we want to nd the point (xg , yg ) with the property that if we place a
fulcrum under the plate at that point, the plate will balance.
Using methods similar to the one-dimensional case, it can be proved
that the plate will balance if the fulcrum is placed at (xg , yg ) with
k
k
m i yi
i=1 mi xi
and
yg = i=1
.
(9.8)
xg = k
k
m
m
i
i
i=1
i=1
This corresponds to the intuitively appropriate concept that the
plate will balance if
it balances both horizontally and vertically.
The sum Mx = ki=1 mi yi is called the moment of the system with
respect to the x axis. This name is due to the fact that if we tried to
balance the system on the x axis, the larger the number Mx , the more
would 
the weights of the system rotate the plate. Similarly, the sum
My = ki=1 mi xi is called the moment of the system with respect to the
y axis.
62.2.2. Symmetry Lines. Now let us consider the continuous version of
the problem. Let P be a plate and let us try to nd the center of mass
of P . (We no longer assume that the mass of the plate is negligible;
in fact, that mass is the object of our study now.) Let us assume, for
the rest of this chapter, that the mass of P is uniformly distributed
over P . Let us also assume that the density of the material of which
P is made is 1. That is, the mass of a unit square within P is 1. Once
these assumptions are made, it is clear that the center of mass of P is
determined by the purely geometric characteristics of P . In order to
emphasize this, if the assumptions of this paragraph hold, we will call
the center of mass of P the centroid of P .
Sometimes we can nd the centroid of P without computation. A
symmetry line of P is a straight line t such that the image of P when
reected through t is P itself. That implies that the two parts into
which t cuts P are congruent, and the plate balances on the line t.
Consequently, the centroid C of P must be on t, since if we concentrate
the entire mass of P in C, it still has to balance on the line P .

62. APPLICATIONS TO PHYSICS AND ENGINEERING

171

The argument of the previous paragraph shows that the centroid


of P must be on every symmetry line of P . So if P has more than
one symmetry line, then these symmetry lines must all intersect in one
point, namely, in the centroid of P . In this case, we obtain the center
of mass of P as the intersection of any two symmetry lines of P . For
example, we can nd the center of mass of a circle, ellipse, rectangle,
or rhombus in this way.
62.2.3. A Formula for Continuous Two-Dimensional Systems. Let us keep

the conditions from the previous section and let us impose the new condition that P is a domain under a curve; that is, the borders of P are
the vertical lines x = a and x = b, the horizontal axis, and the graph
of the continuous function f (x) = y.
We would like to use formula (9.8) to nd the approximate location
of the centroid of P . Let us cut the interval [a, b] into n equal parts,
using the intermediate points a = x0 < x1 < x2 < < xn = b, and
let x = (b a)/n. The vertical lines x = xi cut P into n vertical
stripes. Let Si be the ith such stripe. The area, and hence the mass,
of Si is close to x f (xi ), where xi is the midpoint (xi1 + xi )/2 of
the interval [xi1 , xi ]. So we are approximating Si by a rectangle Ri .
Let us concentrate the entire mass of Ri in the centroid of Ri , that is,
at (xi , f (xi )/2).
Now we can compute the moment of the obtained system of n objects with respect to the x axis. Note that the mass of Ri is equal to
the area of Ri , that is, xf (xi ). So we have
Mx (n) =

n

i=1

m i yi =

n


x f (xi ) f (xi )/2.

i=1

The right-hand side is a Riemann sum, so, as n goes to innity,


it will converge to the corresponding integral, while the left-hand side
will converge to the moment Mx of the original plate with respect to
the x axis.
This yields
 b
1
(9.9)
Mx =
f (x)2 dx.
2
a
A similar argument using horizontal stripes instead of vertical ones
shows that
 b
(9.10)
My =
xf (x) dx.
a

Finally, now that the moments of P are known, it is straightforward


to compute the coordinates of the center of mass of P . Indeed, the

172

9. FURTHER APPLICATIONS OF INTEGRATION

center of mass is the unique point (xg , yg ) with the property that if
b
the entire mass A = a f (x) dx of P is placed in that point, then the
moments of this one-object system are identical to the moments of P .
In other words, Mx = yg A and My = xg A. Therefore, xg = My /A
and yg = Mx /A, which means that formulas (9.9) and (9.10) imply the
following theorem.
Theorem 9.6. Let f be a function such that f (x) 0 if x [a, b].
Let D be a domain whose borders are the vertical lines x = a and x = b,
the horizontal axis, and the curve of the function f (x) = y. Let A(D)
denote the area of D.
Let xg and yg be the coordinates of the centroid of D. Then we have
b
b 1
xf
(x)
dx
[f (x)]2 dx
and
yg = a 2
.
xg = a
A(D)
A(D)
Example 9.6. Find the coordinates of the centroid of the quarter
of the unit circle that is in the northeastern quadrant.
Note that if we asked the same question for the entire unit circle,
the answer would obviously be that xg = yg = 0, since the centroid
of any domain must be on all symmetry lines. If we asked the same
question for the half of the unit circle that is in the northern half-plane,
then xg = 0 would clearly hold, since the vertical axis is a symmetry
line of that semicircle.
Solution: (of Example 9.6): Note that the domain D in question has
a symmetry line, namely, the line determined by the equation x = y.
So the centroid of D is on that line, that is, xg = yg. Therefore, it
suces to compute one of xg and yg . We have f (x) = 1 x2 = y, so
yg is somewhat easier to compute. Theorem 9.6 yields
1 1
(1 x2 ) dx
yg = 0 2
/4

 1
3
2
x 
= x


3 
0

2 2
=
3
4
=
.
3
So the center of gravity of the quarter of the unit circle in the north4
4
, 3
). See Figure 9.9 for an illustration. 2
eastern quadrant is at ( 3

62. APPLICATIONS TO PHYSICS AND ENGINEERING

173

Note that 4/(3) 0.424. This makes perfect sense since this shows
that the centroid of D is closer to the horizontal axis (the bottom of
the quarter circle) than to the y = 1 line (the top of the quarter circle).
That is reasonable, since the bottom of D is wider than the top of D,
so it constitutes a larger portion of the total weight of D than the top
of D.
Example 9.7. Find the centroid of the domain D whose borders
are the vertical lines x = 0 and x = 1, the horizontal axis, and the
graph of the function f (x) = x2 = y.
Solution: The domain D in question does not have a symmetry line,
so we must use Theorem 9.6 to compute both of xg and yg . We have
b
xf (x) dx
xg = a
A(D)
1
x x2 dx
= 0 1
x2 dx
0
1

x4 /4
=
01

x3 /3
0

3
=
4

Figure 9.9. The centroid of a quarter of the unit circle.

174

9. FURTHER APPLICATIONS OF INTEGRATION

b

and
yg =

1
[f (x)]2
a 2

1

dx

A(D)

(x4 /2) dx
=
1/3
1

5
x /10
0
=
1/3
3
= .
10
So the centroid of D is at (0.75, 0.3). This agrees with our intuition,
since the bottom of D is larger than its top, and the left-hand side of
D is smaller than the right-hand side of D. See Figure 9.10 for an
illustration.
2
0

62.3. Exercises.

(1) Find the centroid of the unit semicircle that lies in the northern
half-plane.
(2) Find the centroid of the unit semicircle that lies in the western
half-plane.
(3) Find the centroid of the plate whose borders are the lines x = 0
and x = /2, the graph of the function f (x) = sin x, and the
horizontal axis.
(4) Find the centroid of the plate whose borders are the vertical
lines x = 1 and x = 2, the horizontal axis, and the graph of
the function f (x) = ln x.

Figure 9.10. Center of mass of the area dened in Example 9.7.

62. APPLICATIONS TO PHYSICS AND ENGINEERING

175

(5) Find the centroid of the plate whose borders are the vertical
lines x = 1 and x = 2, the horizontal axis, and the graph of
the function f (x) = ex .
(6) Find the centroid of the plate whose borders are the vertical
lines x = 0 and x = 1, the horizontal axis, and the graph of
the function f (x) = x3 .
(7) Find the centroid of the plate whose borders are the vertical
lines x = 0 and x = 1, the horizontal axis, and the graph of
the function f (x) = x.
(8) Find the centroid of the plate whose borders are the vertical
lines x = 0 and x =1, the horizontal axis, and the graph of
the function f (x) = x.
(9) Find the centroid of the plate whose vertices are at (0, 0),
(10, 10), and (20, 0).
(10) Find the centroid of the plate whose vertices are at (0, 0),
(10, 10), and (50, 0).
(11) Find the centroid of the plate whose vertices are at (0, 0),
(15, 0), (1, 1), and (8, 1).
(12) Let ABC be any triangle. Let us assign objects of equal mass
to A, B, and C. Let A1 denote the midpoint of the segment
BC, let B1 denote the midpoint of the segment AC, and let
C1 denote the midpoint of the segment AB. Prove that the
lines AA1 , BB1 , and CC1 intersect in one point, centroid of
the system ABC.
(13) Keep the notation of the preceding exercise. Prove that the
center of mass of ABC is the same as the centroid of A1 B1 C1 ,
if we assign objects of equal weight to A1 , B1 , and C1 .
(14) An object consists of two squares. The rst is the square
with vertices (0, 0), (0, 2), (2, 0), and (2, 2), and the other is
the square with vertices (0, 2), (1, 2), (0, 3), and (1, 3). The
density of the material of the small square is twice the density
of the material of the large square. Where is the centroid of
this object?
(15) Let n > 1 be a positive integer. Find the centroid M (n) of the
plate whose borders are the vertical lines x = 1 and x = n,
the horizontal axis, and the graph of the function f (x) = x3 .
(16) Keep the notation of the preceding exercise. Describe the behavior of the point M (n) as n goes to innity.
(17) How does the answer to the two preceding exercises change if
f is replaced by g, where g(x) = x2 ?

176

9. FURTHER APPLICATIONS OF INTEGRATION

Figure 9.11. The demand function p(x) of a commodity.


(18) Solve exercise 15 with f unchanged, but with the two vertical
lines changed to x = 1/n and x = 1. Try to nd an answer
without additional integration.
(19) Find the centroid of the plate whose borders are the horizontal
lines y = 0 and y = 1, the vertical axis, and the graph of the
function x = y.
(20) Find the centroid of the plate whose borders are the horizontal
lines y = 0 and
y = 1, the vertical axis, and the graph of the
function x = y + 1.
63. Applications to Economics and the Life Sciences
63.1. Consumer Surplus. Let us consider the problem of pricing some

merchandise whose value is highly subjective; that is, it is worth more


to some customers than to others. Examples of this could be tickets
for various sporting events, air line tickets to vacation destinations, or
popular books.
Let p(x) be the demand function of this commodity. That is, p(x)
is the price that will result in selling x units of the commodity. Lower
prices usually lead to higher sales, therefore p(x) is usually a decreasing
function as illustrated in Figure 9.11.
The area under the graph of p represents the total revenue the
company could possibly have, if it managed to charge each customer
the maximum price that that customer is willing to pay. Indeed, if the
highest amount anyone is willing to pay for one unit is p(x1 ), and x1
customers are willing to pay that price, then the revenue coming from
these most enthused customers is x1 p(x1 ), which is the area of the
domain under the graph of p that is between the vertical lines x = 0

63. APPLICATIONS TO ECONOMICS AND THE LIFE SCIENCES

177

and y = x1 . We could continue in this way, noting that if the second


highest price that some customers are willing to pay is p(x2 ), and there
are x2 x1 people who are willing to pay this price (not including those
who are willing to pay even p(x1 )), then the revenue from them will
be (x2 x1 )p(x2 ). This is the area of the domain under the graph of p
that is between the lines x = x1 and x = x2 , and so on.
If the seller decides to set one xed price p(z), then the seller will
sell z items, for a total revenue of zp(z) (the area of the rectangle R
bordered by the two coordinate axes and the lines x = z and y = p(z)).
This means that the customers who would have paid an even higher
price for these goods have saved money. Besides losing that potential
revenue, the seller also loses revenue by not getting any purchases from
customers who were willing to pay some amount, but not p(z), for one
unit.
Let xn be the number of items that the seller can sell at the lowest
price at which the seller is still willing to sell these items. It is a direct
consequence of the above discussion that the total amount saved by all
customers who bought the item at p(z) dollars is the area under the
curve of p but above the rectangle R, that is,
(9.11)

n


(p(xi ) p(z))(xi xi1 ).

i=1

If the number n of prices at which various customers are willing to


buy goes to innity, then the Riemann sum in (9.11) approaches the
denite integral
 z
(9.12)
CS =
(p(x) p(z)) dx.
0

In economics, CS is called the customer surplus for the given commodity.


Similarly, the integral

p(x) dx
z

is the amount of missed revenue, that is, the money the company could
have received from buyers who found the product too expensive. Note
that this is the area of the domain under the graph of p, but on the
right of R.
Example 9.8. Tickets for a certain ight are normally priced at
$300, and in an average month, 500 tickets are sold. Research shows
that, for every $10 that the price is reduced, the number of tickets sold

178

9. FURTHER APPLICATIONS OF INTEGRATION

goes up by 20. Find the demand curve and compute the consumer
surplus for these tickets if the price is set at $240.
Solution: If the airline wants to sell x tickets, then the price that the
airline needs to charge is
x 500
x 500
= 300
.
p(x) = 300 10
20
2
Indeed, in order to sell x500 extra tickets, the airline needs to decrease
its price by $10 for each 20-pack of extra tickets.
If the price is set at p(z) = $240, then the last displayed equation
shows that z = 60. Now we can apply formula (9.12) to compute that
the customer surplus is
 60
(p(x) 240) dx
CS =
0

 60 
x 500
=
dx
60
2
0
 60

x
=
310
dx
2
0
= 17, 700.
So customers would save a total of $17,700 in an average month if the
price of the tickets were set at $240.
2
63.2. Survival and Renewal.

Example 9.9. Let us assume that there are currently 30,000 people
in the United States who have a certain illness. Let us also assume
that we know that the fraction of that population who will still have the
illness t months from now is given by the function f (t) = e0.05t . We
also know that every month 1000 new patients will get the illness. How
many people in the United States will have the illness in 20 months?
Solution: Clearly, f (20) = e1 = 0.368 of the people who currently
have the illness will still have it 20 months from now. Now we have
to compute the number of people who will get the illness between now
(t = 0) and 20 months from now (t = 20) and will still be ill 20 months
from now.
Subdivide the interval [0, 20] into n equal subintervals using the
points
0 = t0 < t1 < < tn = 20.
Set 20/n = t. Then, for all i, there are 1000 t people who will get
the illness during the time period [ti1 , ti ). That means that 20 months

63. APPLICATIONS TO ECONOMICS AND THE LIFE SCIENCES

179

from now, in other words, approximately 20 ti1 months from getting


the illness, the fraction of them who will still have the illness will be
f (20 ti1 ). So their number will be about 1000 t f (20 ti1 ).
Summing over all allowed values of i, we get that the total number of
people in the United States who will have the illness 20 months from
now is
n

1000 t f (20 ti1 ).
30, 000 f (20) +
i=1

We recognize that the above sum is a Riemann sum, so, as n goes to


innity, the above expression converges to
 20
D = 30, 000 f (20) +
1000f (20 t) dt
0
 20
1
= 30, 000 e + 1000
e0.05t1 dt
0

11, 036.38 + 12, 642.41


23, 679.
So 20 months from now, 23,679 people in the United States will
still have the illness.
2
Note that the result of the previous example shows that the number
of people in the United States who have the illness will decrease during
the next 20 months. Try to nd an intuitive explanation for that fact
that does not involve integration.
63.3. Exercises.

(1) How much money would customers save in total if the ights
discussed in Example 9.8 were sold for $210 each?
(2) Consider the ights discussed in Example 9.8 and assume that
the tickets are sold at their original price of $300. Compute
the missed revenue for the airline.
(3) Consider the ights discussed in Example 9.8 and assume that
the tickets are sold at a reduced price of $270. Compute the
missed revenue for the airline.
(4) A demand curve is given by p(x) = 600/(x + 10). Find the
consumer surplus when the selling price is $20.
(5) A demand curve is given by p(x) = 1000/(x + 40). Find the
missed revenue when the selling price is $25.
(6) We deposit P dollars into a bank account at an annual interest
rate of r%. Let us assume that the interest is compounded n
times a year. Find a formula for our account balance after t

180

9. FURTHER APPLICATIONS OF INTEGRATION

(7)

(8)

(9)

(10)

(11)

(12)

years. Then use that result to compute our account balance


after t years if the interest is compounded continuously.
We deposit $100,000 into a bank account, where it will earn
an annual interest of 5%. The interest is compounded continuously. Each year, we deposit $2000 into this same account in
a continuous manner. What will our account balance be in 15
years?
Consider the bank account of the preceding exercise, with the
sole dierence that, after 10 years, we increase our annual
deposit to $3000. What will our account balance be in 15
years?
We deposit $100,000 into a bank account, where it will earn
an annual interest of 5%. The interest is compounded continuously. Each year, we withdraw a total of $4000 in a continuous
manner. What will our account balance be in 20 years?
A country currently has a population of 80 million people and
a natural growth rate of 1.5%. The natural growth g of the
population in a given year is computed as the dierence between the number of births and the number of deaths in that
year, while the natural growth rate for that year is g divided
by the size of the population at the beginning of that year.
Let us assume that each year 1.1 million people emigrate
from this country. If the current trends continue, how large
will the population of this country be in 20 years?
A country currently has a population of 100 million people
and a natural growth rate of 1%. See the preceding exercise
for the denition of natural growth rate.
Let us assume that each year 0.5 million people emigrate
from this country. If the current trends continue, how large
will the population of this country be in 25 years?
A country currently has a population of 80 million people and
a natural growth rate of 0.5%. Let us assume that each year
0.35 million people immigrate to this country. If the current
trends continue, how large will the population of this country
be in 20 years?

63. APPLICATIONS TO ECONOMICS AND THE LIFE SCIENCES

181

(13) Tickets to a certain section of the arena for a basketball game


usually cost $50. This results in the sales of 1000 tickets. For
every dollar that the price is dropped, the number of tickets
sold goes up 1%. Find the demand function for these tickets.
(14) Compute the consumer surplus for the tickets discussed in the
previous exercise if the tickets are sold at $40.
(15) Let S(x) be the supply function for a certain commodity. That
is, S(x) is the price that one unit of the commodity has to cost
in order to attract enough sellers to provide x units for sale.
Note that S(x) is an increasing function, since a higher
price is needed to attract more sellers.
Let us assume that the units are sold at a xed price T =
S(t). That means that the sellers who would be willing to
sell at a lower price are making a prot. The total amount of
the prot made by all sellers is sometimes called the producer
surplus. Prove that the producer surplus for this commodity
can be computed by the formula
 t
(T S(x)) dx.
0

(16) Let us assume that the supply function for a certain commodity is given by S(x) = x2 + 2x. Compute the producer surplus
(which we dened in the preceding exercise) if the selling price
is set at $24.
(17) How does the producer surplus change in the situation described in the previous exercise if the selling price is raised to
$35?
(18) Let us assume that the supply function for tickets to ights
between two given cities is given by S(x) = x2 + 10x + 5.
Compute the producer surplus if the selling price is set at
$205.
(19) The gates of a football stadium open at 3 p.m. After t hours,
fans enter the stadium at a rate of f (t) = t3 + 20t2 + 30t +
64 people per hour, where t = 0. How many fans enter the
stadium between 5 p.m. and 6 p.m.?
(20) There are currently 50,000 people in the United States who
take a certain medication. The fraction of that population
who will still take that medication t months from now is given
by the function f (t) = e0.01t . Every month, the medication
will be taken by 500 new patients. How many people in the
United states will take the medication a year from now?

182

9. FURTHER APPLICATIONS OF INTEGRATION

64. Probability
The word probability is often used in informal conversations, even
if it is sometimes not clear what the speaker means by that word. It
turns out that there are two distinct concepts of probability. These two
concepts complement each other in that they are applicable in dierent
circumstances, and use very dierent methods.
64.1. Discrete Probability. Let us say that we are tossing a fair coin

four times. What is the probability that we will get at least three
heads?
This is a situation in which the event that we study, that is, the
sequence of four coin tosses, has only a nite number of outcomes.
Indeed, there are 24 possible outcomes, since each coin toss has two
outcomes (heads or tails), and the coin is tossed four times.
Among these 16 possible outcomes, ve are favorable outcomes,
namely, HHHH, HHHT , HHT H, HT HH, and T HHH. Furthermore, each single outcome (favorable or not) is equally likely to occur,
since the coin is fair, and the result of each coin toss is equally likely
to be heads or tails.
In this situation, that is, when the number of all possible outcomes
is nite, and each outcome is equally likely to occur, dene
Number of favorable outcomes
,
(9.13)
Probability of event =
Number of all outcomes
which, in our example, shows that the probability of getting at least
three heads is 5/16.
Probabilities dened by formula (9.13) are called discrete probabilities. The formula is applicable only when the number of possible
outcomes is nite. If we want to apply this formula in complicated
situations, we need advanced techniques to count the number of all
outcomes and the number of favorable outcomes. The fascinating discipline studying those techniques is called enumerative combinatorics,
and will not be discussed in this book.
64.2. Continuous Probability. Let us say we want to know the probabil-

ity that during the next calendar year the city of Gainesville, Florida,
will have more than 40 inches of precipitation, or we want to know the
probability that Gainesville will have less than 50 inches of precipitation, or that Gainesville will have at least 42 but at most 48 inches of
precipitation.
In this case, formula (9.13) is not applicable, since both the number
of favorable outcomes and all outcomes is innite. Indeed, the amount

64. PROBABILITY

183

of precipitation in Gainesville next year can be any nonnegative real


number. Furthermore, not all outcomes are equally likely. Receiving
very little or very much precipitation is far less likely than receiving
close to the usual amount. We need a totally dierent approach. Our
approach, while dierent from the one in the previous section, shares
some of the most important features of that approach. For instance,
the probability of an event that is certain to happen will be 1, while
the probability of an event that never happens will be 0.
Similar situations occur when we want to know the probability that
a certain device will work for more than t years, or that a randomly
selected person weighs more than p pounds but less than q pounds, or
that the blood pressure of a randomly selected person is below a given
value. The quantities mentioned here are called random variables.
We would like to dene the probability F (a) that the amount X
of precipitation in Gainesville next year will be at most a inches. This
probability will sometimes be denoted by P (X a).
While we do not yet know how to compute F (a), we know that the
function F has to satisfy the following requirements:
(1) We will have F (a1 ) F (a2 ) if a1 < a2 . In other words, F is
increasing. Indeed, if X a1 , then X a2 since a1 < a2 .
(2) We will have lima F (a) = 1, since the amount of precipitation is always nite.
(3) The function F (a) is a continuous function, since a little bit
of change in a will only mean a little bit of change in F (a) =
P (X a). We sometimes refer to this fact by saying that X
is a continuous random variable.
Note that the function F is called the distribution of the random
variable X.
It can be proved that if F has all these properties, then there exists
a unique function f : R R that has the following properties:
a
(a) For all a R, we have f (x) dx = F (a) = P (X a).

(b) The equality f (x) dx = 1 holds.
(c) For all real numbers x, the inequality f (x) 0 holds.
If f is the unique function described by the three properties above,
then f is called the probability density function, or simply density function, of the continuous random variable X.
Note that property (a) above implies that, for all real numbers
a < b, the equality
 b
f (x) dx
(9.14)
P (a X b) =
a

184

9. FURTHER APPLICATIONS OF INTEGRATION

Figure 9.12. The probability P (a X b) as an area.


holds. In other words, P (a X b) is equal to the area of the domain
between the graph of the density function f , the horizontal axis, and
the vertical lines x = a and x = b. See Figure 9.12 for an illustration.
Indeed, we have
P (a X b) = P (X b) P (X a)
= F (b) F (a)

 b
f (x) dx
=

 b

f (x) dx

f (x) dx.

=
a

Example 9.10. Let the continuous random variable X have density


function

0 if x,
f (x) = 6x(1 x) if 0 x 1 ,
0 if x > 1.
Verify that f is indeed a density function and compute the probability P (0.3 X 0.6).
Solution: In order to see that f is indeed a density function, we
must verify that its denite integral, taken over the entire line of real
numbers, is equal to 1. This is not dicult, since f (x) = 0 outside the
interval [0, 1]. This leads to
 2
 1
 1

3


x
1
x 
x x2 dx = 6
f (x) dx = 6

 = 6 = 1.
2
3 
6

0
0

So f is indeed a valid density function.

64. PROBABILITY

185

Figure 9.13. The graph of the function f in Example 9.10.


We can use formula (9.14) to compute the requested probability.
We get
 0.6
P (0.3 X 0.6) =
f (x) dx
0.3
 0.6
=6
(x x2 ) dx
0.3
 2
 0.6
3
x
x 
=6


2
3 
0.3

= 6(0.108 0.036)
= 0.432.

64.2.1. Exponential Distribution. Consider the following density func-

tion. Let be a positive real number and let



0
if x < 0,
(9.15)
f (x) =
x
if 0 x .
e
We see that f is a decreasing function on the interval [0, ). Figure
9.14 shows how the speed at which f decreases depends on the parameter .
It turns out that this density function is a very frequently occurring
one. Therefore, it has a name. It is called the exponential density function with parameter . Using the right constant and under the right
circumstances, it can be used in many scenarios, typically connected
to waiting times. For instance, it could be used to measure the probability that, given a starting moment, a given cell phone will ring in less
than t minutes, or that, at a given location, it will start raining in less

186

9. FURTHER APPLICATIONS OF INTEGRATION

Figure 9.14. = 1 (red), = 1.5 (blue), = 2 (orange).


than h hours, or that, given a random store, a customer will enter in
s seconds. The exponential density function will give a good approximation to compute these probabilities if the mentioned processes take
place at a roughly constant rate. That is, we should choose a part of
the day when that given cell phone receives calls at roughly constant
frequency, a season when it rains at that location at roughly constant
time periods, or a time of day when customers enter that store at a
roughly constant rate.
Example 9.11. The probability that a certain kind of new refrigerator will need a major repair in x years is given by the exponential
density function with parameter = 1/9. What is the probability that
a new refrigerator will not need a major repair for 10 years?
Solution: First, we compute the probability that the refrigerator will
need a major repair in 10 years. Let X denote the number of years
passing before the rst major repair is needed. Then that probability is
 10
f (x) dx
P (X < 10) =

10

1 x/9
dx
e
9
0
10


= ex/9 


0
10/9

=1e
= 0.671.

64. PROBABILITY

187

Therefore, the probability that the refrigerator will not need a major
repair in 10 years is P (X 10) = 1 0.671 = 0.329.
2
64.2.2. Mean. If we want to compute the average weight of a per-

son selected from a given population of n people, we can simply take


the weights a1 , a2 , . . . , an of those people and compute their arithmetic
mean, or average, that is, the real number
a1 + a 2 + + a n
.
A=
n
This could take a very long time if n is a very large number. If the data
are given in a more organized form, we may be able to save some time.
In particular, if we know that there are b1 people in the population
whose weight is x1 , there are b2 people whose weight is x2 , and so on,
then we can compute the average weight of the population as
(9.16)

A=

b1 x 1 + b 2 x 2 + + b k x k
,
b1 + b2 + + bk

since this fraction is the total weight of the population divided by the
number of people in the population.
Now note that pi = bi /(b1 + b2 + + bk ) is just the probability that
a randomly selected person of this population has weight xi . Therefore,
(9.16) can be written as
(9.17)

p 1 x1 + p 2 x2 + + p k xk .

Theoretically, the weight of a person can take innitely many values since the measuring scale can be always be more precise. It is not
dicult to prove that, as k goes to innity, the sum in (9.17) will turn
into a Riemann sum, and the weights xi will be measured by a continuous random variable X, and the probabilities pi will be expressible by
the denite integrals of a density function. This leads to the following
denition.
Definition 9.1. Let f be the density function of the continuous
random variable X. Then the value of

tf (t) dt
(X) =

is called the average value or mean or expected value of X.


Example 9.12. Let X be the continuous random variable whose
density function is the exponential density function with parameter
that we dened in (9.15). Then (X) = 1/.

188

9. FURTHER APPLICATIONS OF INTEGRATION

Solution: Using Denition 9.1, we have



(X) =
tf (t) dt


=
tet dt
 
1

tet et 



=

1
.

2
In other words, the parameter and the mean of an exponential distribution are reciprocals of each other. In view of this, we can reformulate the result of Example 9.11 as follows. If the average time before a
new refrigerator needs a major repair is 9 years, then the probability
that a refrigerator will not need a major repair for 10 years is 0.329.
64.2.3. Normal Distribution. Let be a real number and let be pos-

itive real number. Consider the density function




(x )2
1
.
f (x) = exp
2 2
2
The distribution dened by this density function is called the normal distribution with parameters and . This distribution is denoted
by N (, ). In particular, if = 0 and = 1, then the obtained
distribution N (0, 1) is called the standard normal distribution.
Plotting the graph of f for various values of and , we see that
the graph has a bell curve; its highest point is reached when x = ,
and it increases on the left of that and decreases on the right of that.
The smaller the value of , the steeper is the rise and fall of the graph
of f . See Figure 9.15 for an illustration.
It can be proved that is precisely the mean of N (, ). The
constant is called the standard deviation of N (, ). It measures how
spread out the values of our variable X are. (The precise denition is
that is the square root of the mean of (X )2 .)
Many scenarios are modeled by a normal distribution, such as test
scores, athletic results, or annual snowfall at a given location.
Example 9.13. In an average year, Northtown gets 10 feet of snow,
with a standard deviation of 2 feet. What is the probability that, in a
random year, Northtown gets between 9 and 12 feet of snow if snowfall
is modeled by a normal distribution?

64. PROBABILITY

189

Figure 9.15. = 1 (red), = 1.5 (orange), = 2


(green), and = 2.5 (blue).
Solution: Let X denote the snowfall in a random year in Northtown.
We need to nd the probability P (9 X 12). As snowfall is modeled
by a normal distribution, the given parameters imply that that must
be the distribution N (10, 2). Therefore, by formula (9.14), we have


 12
1
(x 10)2
exp
dx
P (9 X 12) =
8
2 2
9
= 0.5328,
where the denite integral has to be computed by some approximation
2
method (or a software package) since ex has no antiderivative among
elementary functions.
2
64.3. Exercises.

(1) For which value of c will f (x) = cx4 be a density function on


[0, 1]?
c
(2) For which value of c will f (x) = 1+4x
2 be a density function
on [, ]?
(3) Consider the value of c that makes the function f of the preceding exercise a density function. Using that value c, nd
P (2 < x < 2).
(4) Prove that the exponential density function, which we dened
in (9.15), is indeed a density function.
(5) Let X be a random variable whose density function is 0 outside
the interval [0, 1] and satises f (x) = 2x for x [0, 1]. Prove
that f is indeed a density function, and compute the mean
of X.

190

9. FURTHER APPLICATIONS OF INTEGRATION

(6) Let us say that the lifetime of a bicycle tire (measured in


months) has an exponential distribution with = 7. What is
the probability that a tire will last between 5 and 8 months?
(7) What is the probability that the tire of the preceding exercise
will last more than 14 months?
(8) Find the number a of months such that there is exactly 1/2
probability that the tire of the previous exercise will last at
least a months.
(9) Let us assume that the lifetime of a given product, measured
in months, has an exponential distribution with parameter .
What is the probability that the product will last for more
than twice its expected lifetime?
(10) Considering the product of the preceding exercise, what is the
probability that it will last at least k times its expected lifetime, where k is any positive real number?
(11) Considering the product of the preceding exercise, what is
more likely, that its lifetime will be at most 1 months
or that it will be at least + 1 months?
(12) The average score on an exam is 100 points. In order to pass,
a student cannot be more than 2 standard deviations below
the average. If the scores have a normal distribution with a
standard deviation of 6, how large a fraction of the students
will pass the exam?
(13) Using the conditions of Example 9.13, what is the probability
that Northtown will get less than 5 feet of snow in a given
year?
(14) Use a software package to prove that the standard normal
distribution function is indeed a distribution function.
(15) The height of the adult male population of the Netherlands
has a normal distribution with a mean of 73 inches and a
standard deviation of 3 inches. What percentage of the adult
male population is more than 75 inches tall?
(16) Consider the adult male population of the Netherlands given
in the preceding exercise. What percentage of that population
is between 70 and 75 inches tall?
(17) Prove that the mean of the standard normal distribution is
indeed 0.
(18) Let X be the random variable that counts the goals scored
by an oensive soccer player of a certain elite league during
an entire season. An oensive player is considered exceptional
if the number of goals he scores exceeds the average of all

64. PROBABILITY

191

oensive players by at least 3 standard deviations. Let us say


that X has distribution N (33, 3). What percentage of oensive
players is considered exceptional?
(19) For the standard normal distribution, nd the probability that
the value of the random variable is within 3 standard deviations of the mean.
(20) For any normal distribution, nd the probability that the value
of the random variable is within 3 standard deviations of the
mean.

CHAPTER 10

Planar Curves
65. Parametric Curves
Every point in a plane can be dened as an ordered pair of real numbers (x, y) called the rectangular or Cartesian coordinates. A graph of
a function f is the set points in a plane whose coordinates satisfy
the condition y = f (x). The graph gives a simple example of a planar
curve. More generally, a planar curve can be dened as the set of points
whose coordinates satisfy the condition F (x, y) = 0 called the Cartesian equation of a curve. In many instances, an equation F (x, y) = 0
has multiple solutions for every given x. For example, consider the
circle of unit radius:

(10.1)
x2 + y 2 = 1 = y = 1 x2 , x [1, 1].

2
The two solutions represent two semicircles. The graph y =
1x
is the semicircle above the x axis, while the graph y = 1 x2 is
the semicircle below the x axis. The union of the two graphs is the full
circle. This example shows a deciency in describing planar curves by
the graph of a function because the curves cannot always be represented
as the graph of a single function.
On the other hand, (10.1) admits a dierent solution:
(10.2)

x2 + y 2 = 1

x = cos t ,

y = sin t ,

t [0, 2],

which immediately follows from the trigonometric identity cos2 t +


sin2 t = 1 for all values of t.
This representation means that a point of the coordinate plane is
assigned to every value of t [0, 2] by the rule (x, y) = (cos t, sin t).
The coordinates of points of the circle are functions of a third variable
called a parameter. As t changes, the point (cos t, sin t) traces out the
circle of unit radius centered at the origin in the plane. The parameter t has a simple geometrical interpretation. It is the angle counted
counterclockwise from the positive x axis to a ray from the origin on
which the point (cos t, sin t) lies. This observation admits a natural
generalization.
Definition 10.1 (Parametric curves). Let x(t) and y(t) be continuous functions on [a, b]. A parametric curve in the coordinate plane is
193

194

10. PLANAR CURVES

Figure 10.1. Circle: x(t) = cos t, y(t) = sin t.

Figure 10.2. Parametric curve. As t increases from a


to b, the point (x(t), y(t)) traces out a curve in the xy
plane.
the set of points satisfying the conditions, called the parametric equations,
x = x(t) , y = y(t) , t [a, b].
The points (x(a), y(a)) and (x(b), y(b)) are called the initial and terminal points of the curve, respectively.
The graph of a function f is a particular example of a parametric
curve: x = t, y = f (t).

65. PARAMETRIC CURVES

195

Figure 10.3. The spiral x = t cos t, y =


t sin t, t
[0, 2]. The distance from the origin R = x2 + y 2 = t
increases linearly as the angle t, counted counterclockwise from the positive x axis, increases from 0 to 2.
Parametric curves are common in everyday life. The position of a
particle in a plane is dened by its rectangular coordinates (x, y) in the
plane. When the particle moves, its coordinates become functions of
time t so that the parametric curve x = x(t), y = y(t) is the trajectory
of the particle.
Example 10.1. Sketch the curve with the parametric equations x =
t cos t, y = t sin t, t [0, 2].
Solution: A basic approach to visualize the shape of a parametric
curve is to plot points (x(tk ), y(tk )), k = 1, 2, ..., n, corresponding to
successive values of t: t1 < t2 < < tn . For n large enough, a fairly
good picture of the curve emerges. This approach can be followed
here, and the reader is advised to do so, for example, tk = 2k/n,
k = 0, 1, ..., n. However, there is another way in part specic to this
very problem. Note that x2 + y 2 = t2 so that the equations may be
written in the form of the parametric equations of the circle x = R cos t,
y = R sin t, where the radius increases linearly with t, R = R(t) =
t. The parameter t can be viewed as the angle between a ray from
the origin and the positive x axis counted counterclockwise. Thus,
the curve has the following interpretation. As the point (x(t), y(t))

196

10. PLANAR CURVES

rotates about the origin, the distance R between it and the origin
increases linearly with the rotation angle. Such a motion occurs along
an unwinding spiral. In the interval t [0, 2], the spiral makes one
full turn from the initial point (0, 0) to the terminal point (2, 0). 2
65.1. Parametric Curves and Curves as Point Sets. If a curve is dened

as a point set in the coordinate plane, for example, by the Cartesian


equation F (x, y) = 0, then there are many parametric equations that
describe it. For example, the circle (10.1) may also be described by the
following parametric equations:
(10.3)
x2 + y 2 = 1 = x = cos(3 ) , y = sin(3 ) , [0, 2].
What is the dierence between (10.2) and (10.3)? First, note that,
as the parameter t in (10.2) increases, the point (cos t, sin t) traces out
the circle counterclockwise (the initial point (1, 0) moves upward as y =
sin t > 0 for 0 < t < /2). In contrast, the point (cos(3 ), sin(3 ))
does so clockwise with increasing (the initial point (1, 0) moves downward as y = sin(3 ) for 0 < < /6). Second, as t ranges over the
interval [0, 2], the point (cos t, sin t) traces out the circle only once,
while the point (cos(3 ), sin(3 )) winds about the origin three times
because the period of the trigonometric functions involved is 2/3, so
the point returns to the initial point ( = 0) three times when = 2/3,
= 4/3, and = 6/3 = 2. Third, there is a relation between the
parameters t and : t = 3 .
This example illustrates the main dierences between curves dened
as a point set by the Cartesian equation F (x, y) = 0 and parametric
curves.
A parametric curve C is oriented ; that is, the point (x(t), y(t))
traces out C in a particular direction (from the initial to the
terminal point).
A parametric curve may repeat itself (or some of its parts)
multiple times.
Parametric equations describing the same point set in the
plane dier by the choice of parameter; that is, if (x(t), y(t))
and (X( ), Y ( )) trace out the same point set C in the plane,
then there is a function g( ) such that X( ) = x(g( )) and
Y ( ) = y(g( )). The change of the parameter t = g( ) is
called a reparameterization of a curve C.
A good mechanical analogy is the motion of cars along a road. The
road is a point set. A parametric curve describes the actual motion of a

65. PARAMETRIC CURVES

197

particular car along the road. Naturally dierent cars moves dierently
along the very same road.
Example 10.2. Suppose a curve C is described by the parametric
equations x = x(t), y = y(t) if t [a, b]. Find the parametric equations
of C such that the curve is traced out backward, that is, from the point
(x(b), y(b)) to (x(a), y(a)) (the initial and terminal points are swapped).
Solution: One has to nd a new parameter , t = g( ), such that
g(b) = a and g(a) = b. When increases from a to b, the parameter
t decreases from b to a, and the sought-after parametric equations are
obtained by the composition X( ) = x(g( )) and Y ( ) = y(g( )). The
simplest possibility is to look for a linear relation between t and ,
g( ) = c + d . The coecients c and d are xed by the conditions
g(a) = b or b = c + da and g(b) = a or a = c + db. Therefore, by
subtracting these equations, b a = (c + da) (c + db) = (b a)d or
d = 1. By adding these equations with d = 1, b+a = (ca)+(cb)
or c = a + b. Hence, t = (a + b) , so that the parametric equations
of C with reversed orientation are
x = x(a + b ) ,

y = y(a + b ) ,

[a, b].

For example, if C is the circle oriented counterclockwise as in (10.2),


then the same circle oriented clockwise is described by x = cos(2
) = cos , y = sin(2 ) = sin , [0, 2].
2
65.2. The Cycloid. The curve traced by a xed point on the circumference of a circle as the circle rolls along a straight line is called a
cycloid (see Figure 10.4). To nd its parametric equations, suppose
that the circle has a radius R and it rolls along the x axis. Let the
xed point P on the circumference be initially at the origin so that the
center of the circle is positioned at the point (0, R) (on the y axis). Let
CP denote the straight line segment between the center of the circle C
and P . Initially, CP is perpendicular to the x axis. As the circle rolls,
the segment CP rotates about the center of the circle. Therefore, it is
natural to choose the angle of rotation as a parameter. The coordinates of P are functions of to be found. If the circle rolls a distance
D so that its center is at (D, R), then the arc length R of the part
of the circle between P and the touch point T has to be equal to D,
that is, D = R. Let Q be a point on the segment CT such that P Q
and CT are perpendicular. Consider the right-angled triangle CP Q.
Its hypotenuse CP has length |CP | = R, and the lengths of its catheti
are |CQ| = |CP | cos = R cos and |P Q| = |CP | sin = R sin . Let

198

10. PLANAR CURVES

Figure 10.4. Denition of a cycloid. A disk of radius


R is rolling along the x axis. A curve traced out by a
xed point on its edge is called a cycloid.

Figure 10.5. Overall shape of a cycloid.


(x, y) be coordinates of the point P . The parametric equations of the
cycloid are
x = D |P Q| = R R sin = R( sin ).
y = R |CQ| = R R cos = R(1 cos ).
It looks like an upward arc over the interval 0 x 2R, with maximal height ymax = 2R ( = /2), and the arc repeats itself over the
next interval of the length of circumference 2R and so on.
Remark. In 1696, the Swiss mathematician Johann Bernoulli
posed the brachistochrone problem: Find the curve along which a particle will slide (without friction) in the shortest time (under the inuence
of gravity) from a point A to a lower point B not directly beneath A.
The particle will take the least time sliding from A to B if the curve is
a part of an inverted arch of a cycloid.

65. PARAMETRIC CURVES

199

65.3. Families of Curves. Dierent values of R dene dierent cycloids.


In general, if the parametric equations contain a numerical parameter,
then the parametric equations dene a family of curves; each family
member corresponds to a particular value of the numerical parameter.

Example 10.3. Investigate the family of curves with the parametric


equations x = a cos t, y = b sin(2t), t [0, 2], where a and b are
positive numbers.
Solution: Consider rst the simplest case a = b = 1. The function
x(t) = cos t has a period of 2, and y(t) = sin(2t) has a period of
. The initial point is (x(0), y(0)) = (1, 0). As t increases, the point
moves upward so that x(t) decreases (becomes less than 1), while y(t)
increases, reaching its maximum value 1 at t = /4. After that, y(t)
begins to decrease, while x(t) continues to decrease. At t = /2, the
point arrives at the origin and passes through it into the third quadrant
so that x(t) and y(t) continue to decrease. When t = 3/4, y(t) attains
the minimum value 1 and begins to increase for t > 3/4, while
x(t) = cos t is still decreasing toward its minimal value 1, which is
reached at t = , and the curve crosses the x axis moving into the
second quadrant. In the second quadrant < t < 3/2, x(t) increases
toward 0, while y(t) rst reaches its maximum value 1 at t = + /4
(the curve touches the horizontal line y = 1) and then decreases to
0. The curve passes the origin again at t = 3/2 and moves into the
fourth quadrant, where it again touches the horizontal line y = 1 at
t = 3/2 + /4 and at t = 2 it arrives at the initial point. The shape
of the curve resembles the innity sign () embedded into the square
bounded by the lines y = 1 and x = 1 so that it touches each of
the horizontal sides y = 1 twice and each of the vertical sides x = 1
once. If a and b are arbitrary, the transformation x ax stretches
(a > 1) or compresses (a < 1) any geometrical set horizontally in the
coordinate plane. The transformation y by does the same but in the
vertical direction. So the family of curves consists of curves of the
shape stretched to t into the rectangle bounded by the lines x = a
and y = b.
2
65.4. Exercises.

In (1)(9), sketch the curve by plotting its points. Include the arrow
showing the orientation of the curve. Eliminate the parameter to nd
a Cartesian equation of the curve.

200

10. PLANAR CURVES

(1) x = 1 + 2t , y = 3 t (2) x =
(4) x = 1 + 2et , y = 3 et

t , y = 2 t (3) x = t2 , y = t3

(5) x = cosh t , y = sinh t

(6) x = 2 sin t , y = 3 cos t

(7) x = 2 3 cos t , y = 1 + sin t

(8) x = cos t , y = sin(4t)

(9) x = t2 sin t , y = t2 cos t

In (10)(12), sketch the family of parametric curves


a2 b 2
a2 b 2
cos3 t , y =
sin3 t
a
b
(11) x = a cosh2 t , y = b sinh2 t
(10) x =

(12) x = a(sinh t t) , y = a(cosh t 1)


In (13)(16), nd parametric equations of the curve dened by a Cartesian equation and sketch it. Investigate the dependence of the shape
of the curve on the parameter a.

(14)
x2 + y 2 = a tan1 (y/x)
(13) x2/3 + y 2/3 = a2/3
(15) x3 + y 3 = 3axy

(16) (x2 + y 2 )2 = a2 (x2 y 2 )

Hint: Put y = tx in (15) and y = x tan t in (16).


(17) The curves x = a sin(nt), y = b cos t, where n is a positive integer, are called Lissajous gures. Investigate how these curves
depend on a, b, and n.
(18) Consider a disk of radius R. Let P be a point on the disk at
a distance b from its center. Find the parametric equations of
the curve traced out by the point P as the disk rolls along a
straight line. The curve is called a trochoid. Are the equations
well dened if b > R. Sketch the curve for b < R, b = R, and
b > R.
(19) The swallowtail catastrophe curves are dened by the parametric equations x = 2ct 4t3 , y = ct2 + 3t4 . Sketch these
curves for a few values of c. What features do the curves have
in common? How do they change when c increases?
(20) A hypocycloid is the curve traced out by a point on the circle
that is rolling along a xed circle so that it remains within the
latter. Find parametric equations of the hypocycloid if the
radius of the moving circle is a > b, where b is the radius of
the xed circle. Sketch the curve if the ratio b/a is an integer
n = 2 and n = 4.
(21) An epicycloid is the curve traced out by a point on the circle
that is rolling along a xed circle so that it remains out of

66. CALCULUS WITH PARAMETRIC CURVES

201

the latter. Find parametric equations of the epicycloid if the


radius of the moving circle is a and b is the radius of the xed
circle. Sketch the curve if the ratio a = b.
66. Calculus with Parametric Curves
66.1. Tangent Line to a Parametric Curve. Consider a parametric curve

x = x(t), y = y(t), where the functions x(t) and y(t) are continuously dierentiable and the derivatives x (t) and y  (t) do not vanish
simultaneously for any t. Such parametric curves are called smooth.
Theorem 10.1 (Tangent Line to a Smooth Curve). A smooth parametric curve x = x(t), y = y(t) has a tangent line at any point (x0 , y0 ),
and its equation is
x (t0 )(y y0 ) y  (t0 )(x x0 ) = 0,

(10.4)

where (x0 , y0 ) = (x(t0 ), y(t0 )).


Proof. Take a point of the curve (x0 , y0 ) = (x(t0 ), y(t0 )) corresponding to a particular value t = t0 . Suppose that x (t0 ) = 0. Then,
by the continuity of x (t), there is a neighborhood I = (t0 , t0 + )
for some > 0 such that x (t) = 0 for all t I ; that is, the derivative is either positive or negative in I . By the inverse function
theorem (studied in Calculus I), there is an inverse function t = f (x)
that is dierentiable in some open interval that contains x0 . Substituting t = f (x) into the second parametric equation y = y(t), one
obtains that near the point (x0 , y0 ) the curve can be represented as a
part of the graph y = F (x) such that y0 = F (x0 ). The function F is
dierentiable as the composition of two dierentiable functions. The
derivative F  (x0 ) determines the slope of the tangent to the graph, and
the equation of the tangent line reads
y = y0 + F  (x0 )(x x0 ).

(10.5)
By construction,
y = F (x)

y(t) = F (x(t)) for all t I .

Dierentiation of this equation with respect to t by means of the chain


rule yields
y  (t) = F  (x(t))x (t) = F  (x(t)) =

y  (t)
y  (t0 )

(x
)
=
=
F
.
0
x (t)
x (t0 )

Substituting this equation into (10.5) , the latter can be written in


the form (10.4). If x (t0 ) = 0, then y  (t0 ) = 0 by the denition of
a smooth curve so that there is a dierentiable inverse t = g(y) and

202

10. PLANAR CURVES

hence x = G(y) = x(g(y)). Similar arguments lead to the conclusion


that the tangent line to the graph x = G(y) has the form (10.4). The
details are left to the reader as an exercise.
2
The rule for calculating the slope of the tangent line can also be
obtained by means of the concept of the dierential. Recall that the
dierentials of two related quantities y = F (x) are proportional: dy =
F  (x) dx. On the other hand, x = x(t), y = y(t) and therefore dx =
x (t) dt and dy = y  (t) dt. Hence,
dy
F (x) =
=
dx


dy
dt
dx
dt

y  (t)
.
x (t)

These manipulations with dierentials are based on a tacit assumption


that, for a smooth curve x = x(t), y = y(t), there exists a dierentiable
function F such that y = F (x). In the proof of the tangent line theorem, this has been shown to be true as a consequence of the inverse
function theorem. The use of the dierentials establishes the following
helpful rules to calculate the derivatives:
d
=
dx

d
dt
dx
dt

1 d
x (t) dt

and

d
=
dy

d
dt
dy
dt

1 d
.
y  (t) dt

66.2. Concavity of a Parametric Curve. The concavity of a graph y =

F (x) is determined by the sign of the second derivative F  (x). If


F  (x) > 0, the graph is concave upward, and it is concave downward if
F  (x) < 0. If y(t) and x(t) are twice dierentiable, then the concavity
of the curve can be determined by the sign of the second derivative

 
y


2
x
dy
y  x x y 
d dy
1 d dy
=
.
=
= 
=
d2 x
dx dx
x dt dx
x
(x )3
Example 10.4. A curve C is dened by the parametric equations
x = t2 , y = t3 3t.
(i) Show that C has two tangent lines at the point (3, 0).
(ii) Find the points on C where the tangent line is horizontal or vertical.
(iii) Determine where the curve is concave upward or downward.
Solution: (i) Note that y(t) = t(t2 3) = 0 has three solutions t = 0
and t = 3. But the curve has only two
points of intersection with
the x axis, (0, 0) and (3, 0), because x( 3) = 3; that is, the curve is
self-intersecting at the point (3, 0). This explains why the curve may
have two tangent lines. One has x (t) = 2t and y  (t) = 3t2 3 so that

66. CALCULUS WITH PARAMETRIC CURVES

203

x ( 3) = 2
3 andy  ( 3) = 6. So the slopes of the tangent lines

are (y  /x )( 3) = 3, and the equations of the lines read

y = 3 (x 3) and y = 3 (x 3).
(ii) The tangent line becomes horizontal when y  (t) = 3t2 3 = 0
(see Eq. (10.4)). This happens when t = 1. Thus, the tangent
line is horizontal at the points (1, 2). The tangent line is vertical if
x (t) = 2t = 0 or t = 0. So the tangent line is vertical at the origin
(0, 0).
(iii) The second derivative is
1
3

1
1 d dy
1 d 3t2 3
3 d

d2 y
t

=
1
+
.
=
=
=
d2 x
x dt dx
2t dt 2t
4t dt
t
4t
t2
This equation shows that the curve is concave upward if t > 0 (the
second derivative is positive) and the curve is concave downward if
t < 0 (the second derivative is negative).
2
66.3. Cusps of Planar Curves. Consider a curve dened by the Carte-

sian equation x2 y 3 = 0. This equation can be solved for y, y = x2/3 ,


such that dy/dx = 23 x1/3 . For x > 0, the slope of the tangent line
diverges, y  (x) as x 0+ (as x approaches to 0 from the right).
For x < 0, it also diverges, y  (x) as x 0 (as x approaches 0
from the left). The two branches of the curve (x > 0 and x < 0) are
joined at x = 0 and have a common tangent line, which is the vertical
line x = 0 (the y axis) in this case, but the slope suers a jump discontinuity (from to ). So the curve is not smooth at x = 0 and
exhibits a horn like shape near x = 0. Such a point of a planar curve
is called a cusp.
A parametric curve x = x(t), y = y(t) may have cusps even though
both derivatives x (t) and y  (t) are continuous for all t. For example,
consider the parametric curve x = t3 , y = t2 . For all values of t,
x2 y 3 = 0. So this curve coincides with that discussed above and has
a cusp at the origin (t = 0). The derivatives x (t) = 3t2 and y  (t) = 2t
are continuous everywhere, and, in particular, x (0) = y  (0) = 0 at the
cusp point. Despite the continuity of the derivatives, the slope of the
curve is not dened since dy/dx = y  /x is an undetermined form 00 .
A closer investigation shows that the slope y  (t)/x (t) = 23 t1 suers
a jump discontinuity (from to + as t changes from negative
to positive). The denition of a smooth parametric curve requires
that the derivatives x (t) and y  (t) are continuous and do not vanish
simultaneously at any t. This condition eliminates possible cusps that
may occur at points where both derivatives vanish.

204

10. PLANAR CURVES

Figure 10.6. Plot of y = x2/3 . The curve has a cusp at


the origin.
Furthermore, consider the curve x = t2 , y = t3 . The slope dy/dx =
y /x = 32 t is continuous everywhere and, in particular, at t = 0, where
x (0) = y  (0) = 0. Nevertheless, the curve has a cusp at the origin. To
see this, let us investigate the Cartesian equation of this curve x3 y 2 =
0, which can be solved for x, x = y 2/3 . Therefore, the derivative
dx/dy = 23 y 1/3 exhibits a jump discontinuity from to as y
changes from negative to positive. The two branches of the curve (y > 0
and y < 0) have a common tangent line (the horizontal line y = 0),
but at their joining point a cusp is formed. Note also that the rate
dx/dy = x /y  = 23 t1 suers a familiar innite jump discontinuity, thus
indicating a cusp. This example shows that both rates dy/dx = y  /x
and dx/dy = x /y  must be studied to determine whether there is a
cusp at the point where y  = x = 0.


Example 10.5. Find the tangent line to the astroid dened by the
parametric equation x = a cos3 t, y = a sin3 t, t [0, 2] at the points
t = /4. Determine the points where the tangent line is horizontal
and vertical. Is the curve smooth? Specify the regions of upward and
downward concavity. Use the results to sketch the curve.
Solution: The slope of the tangent line at a generic point is
dy
1 d
1
=  y=
3a sin2 t cos t = tan t.
2
dx
x dt
3a cos t sin t
The value t = /4 corresponds to
the point x = a/23/2 , y = a/23/2
because sin(/4) = cos(/4) = 1/ 2, and the slope at this point is

66. CALCULUS WITH PARAMETRIC CURVES

205

Figure 10.7. The curve x = t2 , y = t3 . The derivatives


x (0) = y  (0) = 0 vanish at t = 0. The curve has a cusp
at the point (x(0), y(0)) = (0, 0).
1. So the tangent line is

a
a
a
y = x
or y = x.
2
2 2
2 2
The slope dy/dt = tan t vanishes at t = 0 and t = so the tangent
line is horizontal (y = 0) at the points (a, 0). However, the derivatives
x (t) = 3a cos2 t sin t and y  (t) = 3a sin2 t cos t vanish simultaneously
at t = 0 and t = . The inverse slope dx/dy = 1/(dy/dx) = cot t
exhibits an innite jump discontinuity at t = 0 and t = , so the
curve has cusps at (a, 0) and hence it is not smooth at these points.
The slope dy/dx is innite at t = /2 and t = 3/2. Therefore, the
curve has a vertical tangent line (x = 0) at (0, a). However, the slope
dy/dx = tan t has an innite jump discontinuity at t = /2 and
t = 3/2. So the curve has cusps and is not smooth at (0, a). Note
also that both derivatives x and y  vanish at these points. Thus, the
curve consists of four smooth pieces, and the curve has cusps at the
joining points of its smooth pieces. The second derivative
1
1 d dy
1
d2 y
= 
=
(tan t) =
2
2
dx
x dt dx
3a cos t sin t
3a sin t cos4 t

206

10. PLANAR CURVES

is positive if sin t > 0 (or y > 0) and negative if sin t < 0 (or y < 0).
So the two branches of the curve above the x axis are concave upward,
while the two branches below it are concave downward. The curves
look like a square with vertices (a, 0), (0, a) whose sides are bent
inward toward the origin.
2
66.4. Exercises.

In (1)(4), nd an equation of the tangent line(s) to the curve at the


given point. Sketch the curve and the tangent(s).
(1) x = t2 + t , y = 4 sin t , (0, 0)
(2) x = sin t + sin(2t) , y = cos t + cos(2t) , (1, 1)
(3) x2 + 2y 2 = 3x , (2, 1), (2, 1)

x2
+ y 2 = 1 , ( 2, 1/ 2) , ( 2, 1/ 2)
(4)
4
In (5)(8), investigate the concavity of the curve.
(5) x = t3 12t , y = t2 1

(6) x = sin(2t) , y = cos t

(7) x = t2 ln t , y = t2 + ln t

(8) x = 3 sin t3 , y = 2 cos t3

(9) Investigate the slope of the trochoid x = R b sin , y =


Rb cos in terms of . Find the condition on the parameters
R and b such that the trochoid has vertical tangent lines.
(10) At what points on the curve x = 2t3 , y = 1 + 4t t2 does the
tangent line have slope 1?
(11) Find equations of the tangents to the curve x = 2t3 + 1, y =
3t2 + 1 that pass through the point (3, 4).
(12) Find points on the curve x2 +4y 2 = 2 at which it has a tangent
line parallel to the line y x = 1.
In (13)(18), investigate whether the curve has cusps or not. If it does,
nd their position. Sketch the curve.
(13) x = t3 , y = t3

(14)

x = t5 , y = t2

(15) x = (t2 1)3 , y = (t3 1)2

x n
y n
(16)
+
= 1 , a > 0, b > 0, n > 0
a
b
(17) (x2 + y 2 )2 = a2 (y 2 x2 )
(18) x3 + y 3 = 3axy
In (19) and (20), nd the points of intersection of two curves C1 and
C2 . At each point of intersection, nd the tangent lines to C1 and C2
and determine the angle between these lines.
(19) C1 : x = cos t , y = sin t; C2 : x = 1+cos(2t) , y = sin(2t)

67. POLAR COORDINATES

(20) C1 : x2 + y 2 = a2 ;

207

C2 : x2 + y 2 = 2ay

67. Polar Coordinates


A point on a plane is described by an ordered pair of numbers
(x0 , y0 ) in the rectangular coordinate system. This description implies
a geometrical procedure to obtain the point as the intersection of two
mutually perpendicular lines x = x0 and y = y0 . The set of vertical
and horizontal lines form a rectangular grid in a plane. There are other
possibilities to label points on a plane by ordered pair of numbers. Here
the polar coordinate system is introduced, which is more convenient for
many purposes.
Fix a point O on a plane. A horizontal ray from O is called the polar
axis, and the point O is called the origin or pole. Consider a generic
point P on the plane. Let be the angle between the polar axis and the
ray OP from O through P . The angle is counted counterclockwise
from the polar axis. The position of the point P on the ray OP is
uniquely determined by the distance r = |OP |. Thus, any point P on
a plane is uniquely associated with the ordered pair (r, ), and r, are
called the polar coordinates of P . The coordinate r is called the radial
variable, and is called the polar angle.
All points on the plane that have the same value of the radial variable form a circle of radius r centered at the origin (all points that have

Figure 10.8. Denition of the polar coordinates in a


plane. r is the distance |OP |, and is the angle counted
counterclockwise from the horizontal ray outgoing from
O to the right. The rectangular coordinates of a point P
are related to the polar ones as x = r cos , y = r sin .

208

10. PLANAR CURVES

the same distance from the origin). All points on the plane that have
the same value of the polar angle form a ray (a half-line bounded by the
origin). So a point P with polar coordinates (r, ) is the intersection
of the circle of radius r and the ray that makes the angle with the
polar axis. Concentric circles and rays originating from the center of
the circles form a polar grid in a plane (see Figure 10.9).
To represent all points of a plane, the radial variable has to range
over the interval r [0, ), while the polar angle takes its values in
the interval [0, 2) because any ray from the origin does not change
after rotation about the origin through the angle 2. It is convenient,
though, to let range over the whole real line. Positive values of
correspond to rotation angles counted counterclockwise, while negative
values of are associated with rotation angles counted clockwise. All
pairs (r, ) with a xed value of r and values of dierent by integer
multiples of 2 represent the same points of the plane. For example,
the ordered pairs (r, ) = (1, ) and (1, ) correspond to the same
point. Indeed, both points are on the circle of unit radius. The ray
= is obtained from the polar axis by counterclockwise rotation
of the latter through the angle . But the same ray is obtained by
rotating the polar axis through the angle clockwise; that is, the rays
= and = coincide.
Furthermore, the meaning of the radial variable r can be extended
to the case in which r is negative by agreeing that the pairs (r, ) and
(r, + ), r > 0, represent the same point. Geometrically, the points
(r, ) lie on a line through the origin at the same distance |r| from the
origin but on the opposite sides of the origin. With this agreement on
extending the meaning of the polar coordinates, each point on a plane
may be represented by countably many pairs:
(10.6)

(r, )

(r, + 2n) or (r, + (2n + 1)),

where n is an integer.
67.1. Rectangular and Polar Coordinates. Suppose that the polar axis

is set so that it coincides with the positive x axis of the rectangular


coordinate system. Every point on the plane is either described by
the rectangular coordinates (x, y) or the polar coordinates (r, ). It is
easy to nd the relation between the polar and rectangular coordinates
of a point P by examining the rectangle with the diagonal OP . Its
horizontal and vertical sides have lengths x and y, respectively. The
length of the diagonal is r. The angle between the horizontal side and

67. POLAR COORDINATES

209

the diagonal is . Therefore, cos = x/r and sin = y/r, or


y
.
x
These relations allow us to convert the polar coordinates of a point to
rectangular coordinates and vice versa (see Fig. 10.8).
x = r cos ,

y = r sin

r 2 = x2 + y 2 ,

tan =

Example 10.6. Find the rectangular coordinates of a point whose


polar coordinates are (2, /6). Find the polar coordinates of a point
with rectangular coordinates (1, 1).

3/2 =
Solution:
For
r
=
2
and

=
/6,
one
has
x
=
2
cos(/6)
=
2

3 and y = 2 sin(/6) = 2/2 =1, so (x, y) = ( 3, 1). For x = 1 and


y = 1, one has r2 = 2 or r = 2 and tan = 1. The point (1, 1)
lies in the second quadrant, that is, /2 . Therefore, = 3/4.
Alternatively, one can take = 3/4 2 = 5/4.
2
67.2. Polar Graphs. A polar graph is a curve dened by the equation

r = f () or, more generally, F (r, ) = 0. It consists of all points that


have at least one polar representation (r, ) that satises the equation.
Here polar coordinates are understood in the extended sense of (10.6)
when they are allowed to take any value.

Figure 10.9. Polar grid. Coordinate curves of the polar coordinates. The curves of constant values of r are
concentric circles. The curves of constant values of are
rays outgoing from the origin.

210

10. PLANAR CURVES

The simplest polar graph is dened by a constant function r = a,


where a is real. Since r represents the distance from the origin, the
pairs (|a|, ) form a circle of radius |a| centered at the origin. Similarly,
the graph = b, where b is real, is the set of all points (r, b), where
r ranges over the real axis, which is the line through the origin that
makes an angle b radians with the polar axis. Notice that the points
(r, b), r > 0, and (r, b), r < 0, lie in the opposite quadrants relative to
the origin as the pairs (r, b) and (r, b + ) represent the same point.
In general, the shape of a polar graph can be determined by plotting
points (f (k ), k ), k = 1, 2, ..., n, for a set of successively increasing
values of , 1 < 2 < < n ; that is, one takes a set of rays = k
and marks the point on each ray at a distance rk = f (k ) from the
origin.
Example 10.7. Describe the curve r = 2 cos .
Solution: By converting the polar graph equation to rectangular coordinates, one nds:
r = 2 cos r2 = 2r cos x2 + y 2 = 2x (x 1)2 + y 2 = 1.
The latter equation is obtained by completing the squares. It represents
a circle with center (1, 0) and radius 1. Note also that by looking at
the graph of the cosine function, one can see that the point (2 cos , )
gets closer to the origin when changes from 0 to /2 (the rst quadrant), reaching the origin at = /2. This gives the upper part of
the circle. A similar behavior is observed when changes from 0 to
/2 (the lower part of the circle in the fourth quadrant). In the intervals (, /2) and (/2, ), the radial variable is negative. The
representation (r, ) is equivalent to (r, ). Therefore, the points
(2 cos , ) and (2 cos , + ) = (2 cos( + ), + ) are the same
for [, /2]. But the latter set can also be described by the
pairs (2 cos , ) if [0, /2]. Similarly, the set traced out by the pair
(2 cos , ) for [/2, ] is the same as when [/2, 0]. So the
pair (2 cos , ) traces out the same set (the circle) each time ranges
2
an interval of length .
Example 10.8. Describe the shape of the curve r = , 0.
Solution: The point (, ) lies on the ray that makes an angle with
the polar axis and is a distance r = from the origin. As the ray rotates
counterclockwise about the origin with increasing , the distance of the
point from the origin increases proportionally. So the curve is a spiral
unwinding counterclockwise (see Fig. 10.10).
2

67. POLAR COORDINATES

211

Figure 10.10. Polar curve r = . It is a spiral because


the distance from the origin r increases with the angle
as the point rotates about the origin through the angle
.
67.3. Symmetry of Polar Graphs. When sketching polar graphs, it it is

helpful to take advantage of symmetry, just like when plotting graphs


y = f (x) for symmetric (f (x) = f (x)) or skew-symmetric (f (x) =
f (x)) functions.
(i) If a polar equation is unchanged when is replaced by ,
the curve is symmetric about the polar axis. Note that the
transformation (r, ) (r, ) means that (x, y) (x, y),
which is the reection about the x axis (or the polar axis).
(ii) If a polar equation is unchanged when (r, ) is replaced by
(r, ) or by (r, + ), the curve is symmetric about the origin. Again, these transformations are equivalent to (x, y)
(x, y), which is the reection about the origin.
(iii) If the equation is unchanged under the transformation (r, )
(r, ), then the curve is symmetric about the vertical line
= /2. In the rectangular coordinates, this transformation
is (x, y) (x, y), which is the reection about the y axis.
Example 10.9. Describe the cardioid r = 1 + sin .
Solution: The equation is unchanged under so the curve
is symmetric about the vertical axis (the y axis). It is sucient to
investigate the curve in the interval [/2, /2] (in the fourth and
rst quadrants). Consider a ray that rotates counterclockwise from
= /2 to = /2 (from the negative y axis to the positive y

212

10. PLANAR CURVES

Figure 10.11. The cardioid r = 1 + sin .


axis). When = /2, r = 0. As increases from /2 to 0 (the
fourth quadrant), the distance from the origin r = 1 + sin increases
monotonically from 0 to 1 (r = 1 on the polar axis). In the interval
[0, /2] (the rst quadrant), the distance from the origin r continues to
increase monotonically and reaches its maximal value 2 on the vertical
axis. The cardioid is shown in Fig. 10.11.
2
67.4. Tangent to a Polar Graph. To nd a tangent line to a polar graph

r = f (), the polar angle is viewed as a parameter so that the parametric equations of the graph are
x = r cos = f () cos ,

y = r sin = f () sin .

By the product rule for the derivative,


dy
=
dx

dy
d
dx
d

f  () sin + f () cos
.
f  () cos f () sin

In particular, if the curve passes through the origin, r = 0, the equation


for the slope at the origin is simplied
dy
dr
= tan if
= f  () = 0.
dx
d
Note that if f  () = 0, then the slope is an undetermined form 00
because x () = y  () = 0 for any value of such that f  () = f () = 0.
This means that the curve may have a cusp at the origin and hence is
not smooth.

67. POLAR COORDINATES

213

Example 10.10. Find the slope of the cardioid r = 1 + sin in


terms of . Investigate the behavior of the cardioid near the origin.
Solution: Here f () = 1 + sin and f  () = cos . This leads to the
slope
cos sin + (1 + sin ) cos
cos (1 + 2 sin )
dy
=
=
.
2
dx
cos (1 + sin ) sin
(1 + sin )(1 2 sin )
where the identity cos2 = 1sin2 has been used to transform the denominator. The cardioid passes through the origin as passes through
/2. The slope dy/dx is undetermined because the numerator and
denominator of the ratio vanish at = /2 (both derivatives dx/d
and dy/d vanish). The left and right limits have to be investigated to
see if the slope has a jump discontinuity thus indicating a cusp. The
numerator vanishes because of the factor cos , while the denominator
vanishes because of the factor (1 + sin ). Hence,
lim

(/2)

dy
cos
sin
1
1
=
lim
=
lim
= ,
dx
3 (/2) 1 + sin
3 (/2) cos

where lHospitals rule has been used to resolve the undetermined form
0
and the property that tan as (/2) has been invoked
0
to nd the limit. The cardioid has a vertical tangent line at the origin.
The slope has an innite jump discontinuity, meaning that the cardioid
has a cusp at the origin (see Figure 10.11).
2
67.5. Exercises.

(1) Find rectangular coordinates of a point whose polar coordinates are given:
(r, ) = (1, /3), (1, 5/6), (4, 10/3)
(2) Find polar coordinates of a point whose rectangular coordinates are given:
(x, y) = (1, 1), (1, 1), (3, 4), (2, 0), (0, 2)
In (3)(5), convert the polar graph equation to a Cartesian equation
and sketch the curve.
(3) r = 4 sin

(4) r = tan sec

(5) r = 2 sin 4 cos

214

10. PLANAR CURVES

In (6)(14), sketch the curve with the given polar equation.


(6) r = , 0 (7) r = ln , 1 (8) r2 3r + 2 = 0
(9) r = 4 cos(6) (10) r2 = 9 sin(2) (11) r = 1 + 2 cos(2)
(12) r = 2 + sin(3) (13) r = 1 + 2 sin(3) (14) r2 = 1
(15) Sketch the curve (x2 + y 2 )2 = 4x2 y 2 . Hint: Use polar coordinates.
(16) Investigate the dependence of the shape of the curve r =
cos(n) as the integer n increases. What happens if n is not
an integer?
(17) Show that the curve r = 1 + a sin has an inner loop when
|a| > 1 and nd the range of that corresponds to the inner
loop.
(18) For what values of a is the curve r = 1 + a sin smooth?
In (19) and (20), nd the slope of the tangent line to the given curve
at the point specied by the value of and give an equation of the
tangent line.
(19) r = 2 sin , = /3

(20) r = 1 2 cos , = /6

(21) Show that the curves r = a sin and r = a cos intersect at


right angles.
In (22)(24), investigate the concavity of the polar graph:
(22) r = a > 0

(23) r = a

(24) r = a(1 + cos )

Hint: Use dierentials to express the derivative d2 y/dx2 in polar coordinates similarly to the calculation of the slope in Section 67.4.
68. Parametric Curves: The Arc Length and Surface Area
68.1. Arc Length of a Smooth Curve. Let C be a smooth curve dened

by the parametric equations x = x(t), y = y(t), where t [a, b].


Suppose that C is traversed exactly once as t increases from a to b
and consider a partition of the interval [a, b] such that t0 = a and
tk = t0 + k t, k = 0, 1, 2, ..., n, are the endpoints of the partition
intervals of width t = (b a)/n. Then the points Pk with coordinates
(x(tk ), y(tk )) lie on the curve so that P0 and Pn are the initial and
terminal points, respectively. The curve C can be approximated by a
polygonal path with vertices Pk . By denition, the length L of C is
the limit of the lengths of these approximating polygons as n :
n

(10.7)
L = lim
|Pk1 Pk |,
n

provided the limit exists.

k=1

68. PARAMETRIC CURVES: THE ARC LENGTH AND SURFACE AREA 215

By the mean value theorem, when applied to the functions x(t) and
y(t) on the interval [tk1 , tk ], there are numbers tk and t
k in (tk1 , tk )
such that
xk = x(tk )x(tk1 ) = x (tk ) t ,

yk = y(tk )y(tk1 ) = y  (t
k ) t.

Therefore,


2
2
2
|Pk1 Pk | = (xk ) + (yk ) = (x (tk ))2 + (y  (t
k )) t.
The
sum in (10.7) resembles a Riemann sum for the function F (t) =
(x (t))2 + (y  (t))2 . It is not exactly a Riemann sum because tk = t
k
in general. However, if x (t) and y  (t) are continuous, it can be shown
that the limit (10.7) is the same as if tk and t
k were equal, namely, L
is the integral of F (t) over [a, b].
Theorem 10.2 (Arc Length of a Curve). If a curve C is described
by the parametric equations x = x(t), y = y(t), t [a, b], where x (t)
and y  (t) are continuous on [a, b] and C is traversed exactly once as t
increases from a to b, then the length of C is
 b 
2
2
dx
dy
L=
+
dt.
dt
dt
a

Figure 10.12. The arc length of a smooth parametric


curve is approximated by the length of n straight line
segments connecting points on the curve. The arc length
is dened in (10.7) as the limit n .

216

10. PLANAR CURVES

If C is a graph y = f (x), then x = t, y = f (t), and dx = dt, and


the length is given by the familiar expression
 b

dy 2
1+
dt.
L=
dx
a
It is convenient to introduce the arc length of an innitesimal segment
of a curve (the dierential of the arc length)

 b

ds
ds =
dt.
ds = (dx)2 + (dy)2 = L =
a dt
C

The symbol C means the summation over innitesimal segments of the
curve S (the integral along a curve C) and expresses a simple fact that
the total length is the sum of the lengths of its (innitesimal) pieces.
68.2. Independence of Parameterization. By its very denition, the arc

length is independent of the parameterization of the curve. If a curve


C is dened as a point set, then any parametric equations can be used
to evaluate the arc length. Let C be traced out only once by x = x(t),
y = y(t), where t [a, b], and by x = X( ), y = Y ( ), where [, ].
As noted, there is a relation between the parameters t and , = g(t),
such that g(t) increases from to as t increases from a to b, that
= g  (t) 0, such that x(t) = X(g(t)) and y(t) = Y (g(t)).
is, d
dt
Therefore, the integrals (10.7) corresponding to dierent parametric
equations of the same curve are related by a change of the integration
variable:
 b 

 b 
2
2
dx
dx d 2
dy d 2
dy
+
dt =
+
dt
L=
dt
dt
d dt
d dt
a
a
 b 
2
2
 
2
2
dx
dx
dy
dy
d
+
+
d.
=
dt =
d
d
dt
d
d
a

Thus, the arc length is independent of the curve parameterization and


can be computed in any suitable parameterization of the curve.
A circle of radius R is described by the parametric equations x =
R cos t, y = R sin t, t [0, 2]. Then dx = R sin t dt and dy =
R cos t dt. Hence, ds2 = (R sin t dt)2 +(R cos t dt)2 = R2 (sin2 t+cos2 t) dt2
= R2 dt2 , or ds = R dt, and

 2
 2
L=
ds =
R dt = R
dt = 2R.
C

Example 10.11. Find the length of one arch of the cycloid


x = R( sin ), y = R(1 cos ).

68. PARAMETRIC CURVES: THE ARC LENGTH AND SURFACE AREA 217

Solution: According to the description of the cycloid, one arch corresponds to the interval [0, 2]. The arc length dierential ds is
found as follows:
dx = R(1 cos ) d ,

dy = R sin d,

ds = dx + dy = [(1 cos )2 + sin2 ]R2 d2


2

= [1 2 cos + cos2 + sin2 ]R2 d2 = (2 2 cos )R2 d2 .



ds = 2(1 cos ) R d.

To evaluate the integral of 2(1 cos ), the double-angle identity is
invoked, sin2 (/2) = (1cos )/2. Since 0 /2 when [0, 2],
the sinus is nonnegative, sin(/2)
0, in the integration interval, and
hence, after taking the square root ( u2 = |u|), the absolute value can
be omitted. Thus,
 2
 2
2
sin(/2) d
L=R
4 sin (/2) d = 2R
0
0
2

2
= 2R[2 cos(/2)] = 8R.
0

68.3. Area of a Planar Region. The area under the curve y = f (x)

b
and above the interval x [a, b] is given by A = a f (x) dx, where
f (x) 0. Suppose that the curve is also described by parametric
equations x = x(t), y = y(t), so that the function x(t) is one-to-one.
Then, by changing the integration variable, dx = x (t) dt and

 b
y dx =
y(t)x (t) dt.
A=
a

The new integration limits are found as usual. When x = a, t is either


or , and when x = b, t is the remaining value.
Example 10.12. Find the area under one arch of the cycloid x =
R( sin ), y = R(1 cos ).
Solution: When [0, 2], x [0, 2R] for one arch of the cycloid,
and y() 0. Using the dierential dx found in the previous example,
 2
 2R
2
y dx = R
(1 cos )2 d
A=
0
0
 2
= R2
(1 2 cos + cos2 ) d
0
 2
 2
2
2
1
[1 + 2 (1 + cos(2))] d = R
(1 + 12 ) d = 3R2 ,
=R
0

218

10. PLANAR CURVES

 2
 2
where 0 cos d = 0 and 0 cos(2) d = 0 by the 2 periodicity of
the cosine function.
2
68.4. Surface Area of Axially Symmetric Surfaces. An axially symmetric

surface is a surface symmetric relative to rotations about a line. Such a


line is called the symmetry axis. For example, a cylinder is symmetric
relative to rotations about its axis, a sphere is symmetric relative to
rotations about its diameter, and so on. An axially symmetric surface
is swept by a planar curve when the latter is rotated about a line. A
cylinder of radius R and height h is obtained by revolving a straight line
segment of length h about a line parallel to the segment at a distance
R. A sphere of radius R is obtained by revolving a circle of radius R
about its diameter.

Figure 10.13. A surface is obtained by rotation of a


smooth curve about a vertical line. If ds is the arc length
of an innitesimal segment of the curve at a point P
and R is the distance of the point P from the rotation
axis, then the surface area swept by the curve segment
is dA = 2R ds (the surface area of a cylinder of radius
R and height ds).

68. PARAMETRIC CURVES: THE ARC LENGTH AND SURFACE AREA 219

Let ds be the arc length of an innitesimal segment of a smooth


curve C positioned at a point (x, y). If the distance between the point
(x, y) and the symmetry axis is R(x, y), then the area dA of the part
of the surface swept by the curve segment when the latter is rotated
about the symmetry axis is the area of a cylinder of radius R(x, y) and
height ds:
dA = 2R(x, y) ds.
The total surface area is the sum of areas of all such parts of the surface
 b

ds
dt,
R(x, y) ds = 2
R(x(t), y(t))
(10.8)
A = 2
dt
a
C
where x = x(t), y = y(t), a t b are parametric equations of C.
Here it is again assumed that the point (x(t), y(t)) traces out the curve
C only once as t increases from a to b. In particular, if the symmetry
axis coincides with the x axis, then R(x, y) = |y| (the distance of the
point (x, y) to the x axis) and


 b

dx 2
dy 2
|y| ds = 2
|y(t)|
+
dt.
A = 2
dt
dt
a
C
Example 10.13. Find the area of the surface obtained by revolving
one arch of the cycloid x = R( sin ), y = R(1 cos ) about the x
axis.
Solution: The dierential of the arc length of the cycloid has been
computed in Example 10.11. Since y(t) 0 here, the absolute value
may be omitted and

 2

A = 2
y ds = 2
R(1 cos ) 2(1 cos ) R d
0
C
 2

2
3
2
sin (/2) d = 16R
sin3 u du
= 8R
0
0

 1
2
2
2
= 16R
(1 cos u) sin u du = 16R
(1 z 2 ) dz
0

1
64R2

.
= 16R2 (z z 3 /3) =
3
1

where the double-angle identity has been used again, sin2 (/2) = (1
cos )/2, and then two successive changes of the integration variable
have been done to evaluate the integral, u = /2 [0, ] and z =
cos u [1, 1].
2

220

10. PLANAR CURVES

68.5. Exercises.

In (1)(6), nd the arc length of the curve.


(1) x = 2 + 3t2 , y = 1 2t3 between the points (2, 1) and (5, 1).
(2) x = 3 sin t sin(3t), y = 3 cos t cos(3t), 0 t .
(3) x = t/(1 + t), y = ln(1 + t) between the points (0, 0) and
(2/3, ln 3).
2
2
(4) x = ca cos3 t, y = cb sin3 t, c2 = a2 b2 .
(5) x = cos4 t, y = sin4 t.
(6) x = a(sinh t t), y = a(cosh t 1), 0 t T .
(7) Find a suitable parameterization of the curve x2/3 +y 2/3 = a2/3
and use it to calculate the arc length of the curve.
In (8)(13), nd the area of the region enclosed by the curve(s). If
necessary, nd parametric equations of the curve rst.
(8)
(9)
(10)
(11)
(12)
(13)

x = a cos3 t , y = a sin3 t (the astroid).


x = a cos t , y =b sin t (an ellipse).
x = t2 2t , y = t, and the y axis.
x = 2t t2 , y = 2t2 t3 .
2
2
x = ca cos3 t, y = cb sin3 t, c2 = a2 b2 .
a sin2 t
.
x = a cos t , y = 2+sin
t

In (14)(22), nd the area of a surface generated by rotating the given


curve about the specied axis. Sketch the surface.
(14)
(15)
(16)
(17)
(18)
(19)
(20)
(21)
(22)
(23)

x = a cos3 t, y = a sin3 t (about the x axis).


x = a cos3 t, y = a sin3 t (about the axis y = x).
x = t3 , y = t2 , 0 t 1 (about the x axis).
x = et t, y = 4et/2 , 0 t 1 (about the y axis).
x2 + y 2 = a2 (about the y axis).
x2/3 + y 2/3 = a2/3
x2 + (y b)2 = a2 , b a (about the x axis)
(x/a)2 + (y/b)2 = 1, 0 < b a (about the x axis)
(x/a)2 + (y/b)2 = 1, 0 < b a (about the y axis)
Let V be the volume a solid
 bounded2 by an axially symmetric
surface. Show that V = C [R(x, y)] ds, where C is the curve
whose revolution about the symmetry axis gives the boundary
surface and R(x, y) is dened in (10.8). Find the volume of
the solid bounded by the surface described in Example 10.13.

69. AREAS AND ARC LENGTHS IN POLAR COORDINATES

221

69. Areas and Arc Lengths in Polar Coordinates


69.1. Area of a Planar Region.

Theorem 10.3 (Area of a Planar Region in Polar Coordinates).


Let a planar region D be bounded by two rays from the origin = a,
= b and a polar graph r = f (), where f () 0, that is,
D = {(r, ) | 0 r f () , a b}.
Then the area of D is
1
A=
2

[f ()]2 d.
a

Proof. Consider a partition of the interval [a, b] by points k =


a + k , k = 0, 1, ..., n, where = (b a)/n. Let mk and Mk
be the minimum and maximum values of f on [k1 , k ]. Recall that a
continuous function f always attains its maximum and minimum values
on a closed interval. The area Ak of the planar region bounded by
the rays = k1 , = k and the polar graph r = f () is not less than
the area of the disk sector with radius r = mk and angle and is not
greater than the area of the disk sector with radius r = Mk and angle
. The area of a sector of a disk with radius R and angle radians
is A = 12 R2 . Therefore,
1 2
1
mk Ak Mk2 ,
2
2
and the total area A of the planar region in question satises the inequality
n
n
n

1 2
1 2
L
U
An =
m A
M = An ,
A=
Ak ,
2 k=1 k
2 k=1 k
k=1
which is true for any n. Let F () = 12 [f ()]2 . The function F is continuous on [a, b]. Then 12 m2k and 12 Mk2 are the minimum and maximum
values of F on the partition interval [k1 , k ]. This shows that the
lower and upper bounds, ALn and AUn , are lower and upper sums for
the function F on [a, b]. By the denition of the denite integral and
integrability of a continuous function (see Calculus I), the upper and
b
lower sums converge to the integral of F over [a, b], ALn a F d and
b
b
AUn a F d as n . The conclusion of the theorem, A = a F d,
follows from the squeeze principle.
2
Example 10.14. Find the area enclosed by one loop of the four-leaf
rose r = cos(2).

222

10. PLANAR CURVES

Solution: Note that r = 1 when = 0, which is the maximal value of


r. The function cos(2) has two roots = /4 that are the nearest
to = 0. Hence, one loop corresponds to the interval [/4, /4].
The area is


1 /4
1 /4
2
cos (2) d =
[1 + cos(4)] d
A=
2 /4
4 /4
/4

1

= .
= [ + 14 sin(4)]
4
8
/4
2
Let D be a planar region that lies between two polar graphs r =
f () and r = g() such that f () g() 0 if [a, b] and 0 <
b a 2; that is, D is the set of points whose polar coordinates
satisfy the inequalities:
D = {(r, ) | 0 g() r f () , a b}.
Then the area of D is given by




1 b
1 b

1 b
2
2
[f ()]2 [g()]2 d.
[f ()] d
[g()] d =
A=
2 a
2 a
2 a
Example 10.15. Find the area of a region D bounded by the cardioid r = 1 + sin and the circle r = 3/2 that lies above the polar axis
(in the rst and second quadrants).
Solution: The polar graphs r = 1 + sin = f () and r = 3/2 = g()
are intersecting when f () = g() or 1 + sin = 3/2 or sin = 1/2.
Since the region D lies in the rst two quadrants, that is, 0 ,
the values of for the points of intersection have to be chosen as =
/6 = a and = /6 = b. Therefore,
D = {(r, ) | 3/2 r 1 + sin , /6 5/6},
and hence the area of D is


1 b
1 b 5
2
9
A=
[(1 + sin ) 4 ] d =
[ 4 + 2 sin + sin2 ] d
2 a
2 a
 b
1
=
[ 54 + 2 sin + 12 (1 cos(2))] d
2 a

1 b 3
=
[ 4 + 2 sin 12 cos(2)] d
2 a
5/6 93 2

1
3
1
= 2 [ 4 2 cos 4 sin(2)]
=
.
8
/6
2

69. AREAS AND ARC LENGTHS IN POLAR COORDINATES

223

Remark. When nding points of intersection of two polar graphs,


r = f () and r = g(), by solving the equation f () = g(), one
has to keep in mind that a single point has many representations as
described in (10.6). So some of the pairs (f (), ), where ranges
over solutions of the equation f () = g(), may correspond to the
same point. To select distinct points, all pairs (f (), ) satisfying the
intersection condition can be transformed by means of (10.6) so that
r [0, ) and [0, 2). In this range of polar coordinates, there is
a one-to-one correspondence between points on a plane and pairs (r, )
with just one exception when r = 0; all the pairs (0, ) correspond to
the origin of the polar coordinate system.
69.2. Arc Length. Suppose that a curve C is traversed by the point
(r, ) = (f (), ) only once as increases from a to b. Choosing
as a parameter, the curve is described by the parametric equations
x = r cos , y = r sin , where r = f (). To nd the arc length of C,
one has to nd the relation between the arc length dierential ds and
d. One has

dr

dr
cos r sin d ,
dy =
sin + r cos d
dx =
d
d

Therefore,

dr
2
dr
2 
d2
ds2=dx2 + dy 2 =
cos r sin +
sin + r cos
d
d

dr 2

2
2
2
2
2
=
(cos + sin ) + r (cos + sin ) d2
d


dr 2
=
+ r2 d2 .
d
The arc length of the curve C is

L=


ds =

ds
d =
d

 b
r2 +
a

dr 2
d

d.

where r = f () and b > a.


Example 10.16. Find the length of the cardioid r = 1 + sin .
Solution: One has

dr 2
2
r +
= (1 + sin )2 + (cos )2 = 2(1 + sin ),
d

224

10. PLANAR CURVES

where the trigonometric identity sin2 + cos2 = 1 has been used. The
cardioid is traversed once if [, ]. Therefore, the length is
 /2

1 + sin d = 2 2
1 + sin d,
L= 2

/2

since the cardioid is symmetric about the vertical line (the y axis).
This integral can be evaluated by the substitution
u = 1 + sin [0, 2]


2
so
that
du
=
cos

d,
where
cos

=
1

sin

=
1 (u 1)2 =

u(2 u). Hence,

2
 2
 2 du

u


L=2 2
du = 2 2
= 4 2 2 u  = 8.
0
2u
u(2 u)
0
0
2
69.3. Surface Area. If a surface is obtained by rotating a polar graph

r = f () about a line, the (10.8) can be used to nd the area of the


surface where the distance R(x, y) and the arc length dierential ds
have to be expressed in the polar coordinates with r = f ().
Example 10.17. Find the area of the surface obtained by rotating
the cardioid r = 1 + sin about its symmetry axis.
Solution: The symmetry axis of the cardioid is the y axis. So the
distance from the y axis to a point (x, y) is R(x, y) = |x|. The surface
can be obtained by rotating the part of the cardioid that lies in the
fourth and rst quadrants, that is, x 0 or [/2, /2]. Since
x = r cos , the surface area is
 /2

ds
d
|x| ds = 2
r cos
A = 2
d
/2
C

 /2

dr 2
= 2
r cos r2 +
d.
d
/2
The derivative ds/d has been calculated in the previous example.
Therefore,
 /2
 1
3/2
A = 2 2
(1 + sin ) cos d = 2 2
(1 + u)3/2 du
/2

(1 + u)5/2
= 2 2
5/2

1
32

,
 =
5
1

where the substitution u = sin has been made to evaluate the


integral.
2

70. CONIC SECTIONS

225

69.4. Exercises.

In (1)(4), sketch the curve and nd the area that it encloses.


(1) r = 4 cos(2) (2) r = a(1 + cos )
(3) r = 2 cos(2) (4) r2 = 4 cos(2)
In (5)(7), sketch the curve and nd the area of one loop of the curve.
(5) r = 9 sin(3) (6) r = 1+2 sin (inner loop) (7) r = 2 cos sec
In (8) and (9), nd the area of the region that lies inside the rst curve
and outside the second curve. Sketch the curves.
(8) r = 2 sin , r = 1 (9) r = 3 cos , r = 1 + cos
In (10)(13), nd the area of the region bounded by the curves. Sketch
the region.
(10) r = 2 , r = , [0, 2] (11) r2 = sin(2) , r2 = cos(2)
(12) r = 2a sin , r = 2b cos , a, b > 0
(13) r = 3 + 2 cos , r = 3 + 2 sin
(14) Find the area inside the larger loop and outside the smaller
loop of the limacon r = 1/2 cos .
In (15)(17), sketch the curve and nd its length.
(15) r = 2a sin

(16) r = , [0, 2] (17) r = a + cos , a 1

In (18)(21), nd the area of the surface obtained by rotating the curve


about the specied axis. Sketch the surface.
(18) r = a > 0, about a line through the origin.
(19) r = 2a cos , a > 0, (i) about the y axis and (ii) about the x
axis.
(20) r2 = cos(2) about the polar axis.
(21) = a and 0 r R, where 0 < a < /2 and R > 0, about
the polar axis.
70. Conic Sections
Consider two intersecting lines in space, L1 and L2 . A surface swept
by the line L2 when it is rotated about the line L1 is a circular double
cone. The line L1 is the symmetry axis of the cone. The point of intersection of the lines is called the vertex of a cone. Any plane that does
not pass through the vertex intersects the cone along a curve. It appears that all such curves fall into three types as shown in Figure 10.14.
If the curve of intersection is a loop, then it is an ellipse. If the plane
is parallel to the line L2 , then the curve is a parabola. If the plane

226

10. PLANAR CURVES

is parallel to the axis of the cone, then the curve is a hyperbola. The
curves of intersection of a plane and a cone are called conic sections,
or conics. They have a pure geometrical description, which will be
presented here.
Remark. A trajectory of any massive object in the solar system
(e.g., comet, asteroid, planet) is a conic sectionthat is, a parabola,
hyperbola, or ellipse. This fact follows from Newtons Law of Gravity
and will be proved in Calculus III.
70.1. Parabolas. A parabola is the set of points in a plane that are

equidistant from a xed point F (called the focus) and a xed line
(called the directrix). Let P be a point in a plane. Consider the line
through P that is perpendicular to the directrix and let Q be the point
of their intersection. Then P lies on a parabola if |F P | = |QP |. This
condition is used to derive the equation of a parabola.
A particularly simple equation of a parabola is obtained if the coordinate system is set so that the y axis coincides with the line through
the focus and perpendicular to the directrix. The origin O is chosen
so that F = (0, p) and hence the parabola contains the origin O, while
the directrix is the line y = p parallel to the x axis (the origin is

Figure 10.14. Conic sections are curves that are intersections of a cone with various planes. The shape of
a conic section depends on the orientation of the plane
relative to the cone symmetry axis.

70. CONIC SECTIONS

227

Figure 10.15. Left: Geometrical description of a


parabola as a set of points P in a plane that are equidistant from a xed point F , called the focus, and a xed
line called the directrix (a horizontal line in the gure).
Right: A circular paraboloid is the surface obtained by
rotating a parabola about the line through its focus and
perpendicular to its directrix.
at distance
|p| from F and from the directrix). If P = (x, y), then
|F P | = x2 + (y p)2 , the point Q has the coordinates (x, p), and
|P Q| = (y + p)2 . An equation of the parabola with focus (0, p) and
directrix y = p is
|F P |2 = |P Q|2

x2 + (y p)2 = (y + p)2

x2 = 4py.

In the 16th century, Galileo showed that the path of a projectile that is
shot into the air at an angle to the ground is a parabola. The surface
obtained by rotating a parabola about its symmetry axis is called a
paraboloid. If a source of light is placed at the focus of a paraboloid
mirror, then, after the reection, the light forms a beam parallel to the
symmetry axis. This fact is used to design ashlights, headlights, and
so on. Conversely, a beam of light parallel to the symmetry axis of a
paraboloid mirror will be focused to the focus point after the reection,
which is used to design reecting telescopes.
70.2. Ellipses. An ellipse is the set of points in a plane, the sum of

whose distances from two xed points F1 and F2 is a constant. The


xed points are called foci (plural of focus). Let P be a point on a
plane. Then P belongs to an ellipse if |P F1 | + |P F2 | = 2a, where a > 0

228

10. PLANAR CURVES

Figure 10.16. Left: An ellipse is the set of points in a


plane, the sum of whose distances from two xed points
F1 and F2 (the foci) is a constant. Right: A circular
ellipsoid is the surface obtained by rotating an ellipse
about the line through its foci.
is a constant (the factor 2 is chosen for convenience to be seen later).
Evidently, |F1 F2 | < 2a; otherwise, no ellipse exists.
A particularly simple equation of an ellipse is obtained when the
coordinate system is set so that the foci lie on the x axis and have
|c| < a. Let
the coordinates F1 = (c, 0) and F2 = (c, 0), where

2
2
P = (x, y)
be a point in a plane. Then |P F1 | = (x + c) + y and
|P F2 | = (x c)2 + y 2 . The point P is on an ellipse if
|P F1 | + |P F2 | = 2a |P F2 | = 2a |P F1 |,
(10.9)

|P F2 |2 = (2a |P F1 |)2 = 4a2 4a|P F1 | + |P F1 |2 ,


16a2 |P F1 |2 = (4a2 + |P F1 |2 |P F2 |2 )2 .

These transformations serve only one purpose, that is, to get rid of
the square roots. Note that now all the distances are squared. So
|P F1 |2 |P F2 |2 = (x + c)2 + y 2 (x c)2 y 2 = 4cx. The substitution
of the latter into the condition (10.9) yields
16a2 [(x+c)2 +y 2 ] = (4a2 +4cx)2

(a2 c2 )x2 +a2 y 2 = a2 (a2 c2 ).

By dividing both sides of this equation by a2 (a2 c2 ), an equation of


an ellipse with foci (a, 0) becomes
x2 y 2
+ 2 = 1,
a2
b
2
2
2
where b = a c so that a b > 0. The ellipse intersects the x axis
at (a, 0) and the y axis at (0, b) (called the vertices of an ellipse).
The line segment joining the points (a, 0) is called the major axis.
If the foci of an ellipse are located on the y axis, then x and y are
swapped in this equation, and the major axis lies on the y axis. This
shows that the restriction a b can be dropped in the ellipse equation.
In particular, an ellipse becomes a circle of radius a if a = b.

70. CONIC SECTIONS

229

One of Keplers laws is that the orbits of the planets in the solar
system are ellipses with the Sun at one focus.
70.3. Hyperbolas. A hyperbola is the set of all points in a plane, the

dierence of whose distances from two xed points F1 and F2 (the foci)
is a constant. For any point P on a hyperbola, |P F1 | |P F2 | = 2a
(as the dierence of the distances can be negative). Let the foci be at
(c, 0). Following the same procedure used to derive an equation of an
ellipse, an equation of a hyperbola with foci (c, 0) is found to be
x2 y 2
2 = 1,
a2
b
2
2
2
where c = a + b . The details are left to the reader as an exercise.
This equation shows that x2 /a2 1 for any y, that is, x a or
x a. A hyperbola therefore has two branches. The branch in
x a intersects the x axis at x = a, while the branch in x a
does so at x = a. The points (a, 0) are called vertices. Furthermore,
in the asymptotic region |x| , a hyperbola has slant asymptotes
y = (b/a)x. Indeed,


b|x|
b|x|
a2
b|x|

a2
x2

1
=

y = b
2
2
2
a
a
x
a
2x
a

as |x| . Here the linearization 1 + u 1 + u/2 has been used to


obtain the asymptotic behavior for small u = a2 /x2 0.
If the foci of a hyperbola are on the y axis, then, by reversing the
roles of x and y, it follows that the hyperbola
y 2 x2
2 =1
a2
b
2
2
2
has foci (0, c), where c = a + b , vertices (0, a), and slant asymptotes y = (a/b)x.
70.4. Shifted Conics. Consider a curve dened by a quadratic Carte-

sian equation
Ay 2 + Bx2 + y + x + = 0.
Suppose that A = 0 and B = 0. By completing the squares, this
equation can be transformed to the standard form

2
2
2
2
+B x
=
+
=d
A y
2A
2B
4A 4B
or
(y y0 )2 (x x0 )2
+
= 1,
A/d
B/d

230

10. PLANAR CURVES

Figure 10.17. Left: A hyperbola is the set of all points


in a plane, the dierence of whose distances from two
xed points F1 and F2 (the foci) is a constant. Top
right: A circular hyperboloid of one sheet is the surface
obtained by rotating a hyperbola about the line through
the midpoint of the segment F1 F2 and perpendicular to
it (the vertical line in the left panel). Bottom right: A
circular hyperboloid of two sheets is the surface obtained
by rotating a hyperbola about the line through its foci
(the horizontal line in the left panel).

where x0 = /(2B) and y0 = /(2A), provided d = 0. Depending


on the signs of A/d and B/d, this equation describes either an ellipse
or a hyperbola as if the origin was moved to the point (x0 , y0 ). If
A/d and B/d are both negative, then the equation has no solution.
If either A or B vanishes, but not both, then the quadratic Cartesian
equation describes a parabola (the details are left to the reader as an
exercise). If A = B = 0, the the equation describes a straight line.
If d = 0, solutions
of the equation form a set of two straight lines,

y y0 = (B/A)(x x0 ), through the point (x0 , y0 ), provided
AB < 0. When solutions of the Cartesian equation form a hyperbola
(d = 0, AB < 0), these lines are its slant asymptotes.

70. CONIC SECTIONS

231

70.5. Conic Sections in Polar Coordinates. The following theorem of-

fers a uniform description of conic sections.


Theorem 10.4 (Conic Sections). Let F be a xed point (called the
focus) and L be a xed line (called the directrix) in a plane. Let e be a
xed positive number (called the eccentricity). The set of points P in
the plane whose the ratio of the distance from F to the distance from
L is the constant e is a conic section. The conic is
(1) an ellipse if e < 1
|P F |
(2) a parabola if e = 1
.
e=
|P
L|
(3) a hyperbola if e > 1
Proof. Set the coordinate system so that F is at the origin and
the directrix is parallel to the y axis and d units to the right. Thus,
the directrix has the equation x = d > 0 and is perpendicular to the
polar axis. If the point P has polar coordinates
(r, ) and rectangular

2
2
coordinates (x, y), then |P F | = r = x + y and |P L| = d x =
dr cos . The condition |P F | = e|P L| yields the equation r = e(dx).
By squaring it, one infers a quadratic Cartesian equation
x2 + y 2 = e2 (d x)2 (1 e2 )x2 + y 2 + 2e2 dx e2 d = 0,
which has been investigated in the preceding section. If e = 1, then
the equation describes the shifted parabola y 2 = 2d(x 1/2). When
e = 1, by completing the squares, this equation is brought to the
standard form

y2
e2 d2
e2 d 2
+
=
.
x+
1 e2
1 e2
(1 e2 )2
If e < 1, then all the coecients are positive, and the equation describes
a shifted ellipse
(x x0 )2 y 2
b2
e2 d2
e2 d
2
2
+
=
1
,
a
=
,
b
=
,
x
=
= c,
0
a2
b2
1 e2
1 e2
e2 1
where c is the distance from the origin to the foci of the ellipse, c2 =
a2 b2 . The eccentricity is then e = c/a. Similarly, if e > 1, then the
coecients have opposite signs, and the equation describes a shifted
hyperbola
c
(x x0 )2 y 2
2 = 1 , e = , c 2 = a2 + b 2 .
2
a
b
a
2
In the beginning of the proof, the polar equation for conic sections
was given as r = e(d r cos ). If the directrix is chosen to be to the
left of the focus as x = d, then cos is replaced by cos in the polar

232

10. PLANAR CURVES

equation. If the directrix is chosen to be parallel to the polar axis as


y = d, then the conic sections are r = e(d y) = e(d r sin ). These
equations can be solved for r to obtain conic sections as polar graphs.
Corollary 10.5 (Conics in Polar Coordinates). A polar equation
of the form
ed
ed
or
r=
1 e cos
1 e sin
represents a conic section of eccentricity e. The conic section is an
ellipse if e < 1, a parabola if e = 1, and a hyperbola if e > 1.
r=

70.6. Exercises.

In (1)(9), classify the conic section. Find the vertices, foci (or focus),
directrix, and asymptotes (if the curve is a hyperbola). Sketch the
curve.
(1) y 2 = 16x

(2) x2 = 4y

(3) y + 12x 2x2 = 18

(5) 9x2 18x + 4y 2 = 27

(4) x2 + 4y 2 = 16

(6) x2 + 3y 2 + 2x 12y + 10 = 0
(8) y 2 2y = 4x2 + 3

(7) 4x2 9y 2 = 36

(9) y 2 4x2 + 2y + 16x + 3 = 0

(10) A long-range radio navigation system uses two radio stations,


located at points A and B along the coastline, that transmit simultaneous signals to a ship located at point P in the
sea. The onboard computer converts the time dierence in
receiving these signals into a distance dierence |P A| |P B|.
This locates the ship on one branch of a hyperbola. Suppose
that station B is located D miles from station A. A ship receives the signal from B microseconds (s) before it receives
the signal from A. The signal travels with the speed of light,
c = 980 ft/s. How far o the coastline is the ship? If the
coordinate system is set so that the line AB coincides with the
x axis and A is at the origin, nd the coordinates of the ship
as functions of .
In (11)(13), classify the conic section. Find the eccentricity, an equation of the directrix, and sketch the conic.
(11) r =

8
4 + sin

(12) r =

10
2 5 cos

(13) r =

1
3 + 3 cos

(14) Show that the conic sections r = a/(1 cos ) and r = b/(1 +
cos ) intersect at right angles.

70. CONIC SECTIONS

233

In (15)(20), nd the polar equations for the curve.


(15) x2 + 4y 2 = 16
(17) y 2 + 6x = 0

(16) x2 4y 2 = 9
(18) x = 4 cos t , y = 9 sin t

(19) x = a cosh t , y = b sinh t

(20) x = a sec t , y = b tan t

(21) The orbit of Halleys Comet, last seen in 1986 and due to return in 2062, is an ellipse with eccentricity 0.97 and one focus
at the Sun. The length of its major axis is 36.18 AU. An astronomical unit (AU) is the mean distance between the Earth
and the Sun, about 93 million miles. Find a polar equation
for the orbit of Halleys Comet. What is the maximal and
minimal distance from the comet to the Sun?

Vous aimerez peut-être aussi